0% found this document useful (0 votes)
120 views

Dynamics SuccessClap

This document provides an overview of dynamics as it relates to UPSC mathematics. It discusses concepts like velocity, acceleration, angular velocity, angular acceleration, and their relationships. Specifically, it examines: 1) How to determine the position, velocity and acceleration of a particle moving in a plane using polar coordinates. 2) Definitions of angular velocity as the rate of change of the angle a particle makes with a fixed line, and angular acceleration as the rate of change of angular velocity. 3) Relationships between angular and linear vectors, and how to calculate the rate of change of a unit vector. 4) How angular velocity relates to linear velocity for a particle moving along a curve using its radius and

Uploaded by

Musavir Khaliq
Copyright
© © All Rights Reserved
Available Formats
Download as PDF, TXT or read online on Scribd
0% found this document useful (0 votes)
120 views

Dynamics SuccessClap

This document provides an overview of dynamics as it relates to UPSC mathematics. It discusses concepts like velocity, acceleration, angular velocity, angular acceleration, and their relationships. Specifically, it examines: 1) How to determine the position, velocity and acceleration of a particle moving in a plane using polar coordinates. 2) Definitions of angular velocity as the rate of change of the angle a particle makes with a fixed line, and angular acceleration as the rate of change of angular velocity. 3) Relationships between angular and linear vectors, and how to calculate the rate of change of a unit vector. 4) How angular velocity relates to linear velocity for a particle moving along a curve using its radius and

Uploaded by

Musavir Khaliq
Copyright
© © All Rights Reserved
Available Formats
Download as PDF, TXT or read online on Scribd
You are on page 1/ 159

SuccessClap

Best Coaching for UPSC MATHEMATICS

UPSC MATHEMATICS STUDY MATERIAL


BOOK- 07 Dynamics
------------------------------------------------------------
Important Links
Checklist for UPSC Mathematics: Click Here
Strategy for UPSC Mathematics: Click Here
22 Weeks Study Plan: Click Here
Download Question Bank Questions: Click Here
Our Courses: Click Here

Website: www.SuccessClap.com
Download Android App: Click Here
Join our Telegram Group: https://t.me/SuccessClap
For Query and Guidance WhatsApp: 9346856874
Table of Contents
01 Velocity Acceleration 2
02 Tangential Normal Velocity Acceleration 15
03 SHM 23
04 Motion on Smooth Rough Plane 47
05 Motion in Resisting Motion 67
06 Particle motion varying mass 98
07 Central Orbit 106
08 Planatery Motion 127
09 Motion in 3D 145
Velocity and Acceleration Along
Radial and Transverse Directions
1.1 INTRODUCTION
A particle is said to be in motion relative to surroundings if it changes its position at different
times relative to these objects. If the particle is free to move in a straight line we say that the motion
is rectilinear and particle has only one degree of freedom. If the particle is free to move in a plane
along any curve we say that the motion is in a plane and the particle has two degree of freedom. If
the particle is free to move in space we say that the particle has three degrees of freedom.

ap
Here we shall discuss the motion of a particle when it has two degrees of freedom, i.e., the
motion of the particle is in a plane.
To determine the position of a moving point in a plane we must know any of the following :
(a) its co-ordinates with respect to two axes fixed in the plane (say x and y axes).

l
sC OR
(b) its distance from a point and angular distance from a straight line, both fixed in the plane
(say pole and initial line in case of polar coordinates).
OR
(c) its arcual distance from a point fixed on its path and the angle, its direction of motion makes
with a straight line fixed in the plane (say s and ψ ).
es
We shall discuss all the three case separately.
1.2 ANGULAR VELOCITY
If a point P moves in a plane and if O be a fixed point and
cc

OX be a fixed line throguh O in the plane, then the angular


velocity of P about O (or of the line OP) is the rate of change Q (r+δr, θ+δθ)
of the angle XOP.
Let P be the position of the moving particle at any time t
Su

δt

P (r, θ)
t+

and let ∠ POX = θ. Let Q be the position of the particle at time r


δθ
(t + δt ) and let ∠ QOX = θ + δ θ. θ
O X
Clearly, in time δ t , the angle turned through by the particle
about O = δ θ.
δθ
∴ Average rate of changing of the angle about O =
δt
∴ The angular velocity of the point P
δθ d θ ɺ
lim = =θ
δt → 0 δt dt
1.3 ANGULAR ACCELERATION
Angular acceleration of the point P is the rate of change or increase of its angular velocity.
Hence, we have
d  d θ  d 2θ ɺɺ
Angular acceleration = = = θ.
dt  dt  dt 2
SuccessClap: Best Coaching for UPSC Mathematics : For Info- 9346856874
Checkout ->22 Weeks Study Plan, Videos, Question Bank Solutions, Test Series

692 Dynamics
Units. Since the angle is measured in radians, hence the unit of angular velocity is radians per
second.
The unit of angular acceleration will be radian per second square.
1.4 RATE OF CHANGE OF UNIT VECTOR
→ Y
Let r̂ denote the unit vector OA such that OA = 1 and n
∠ AOX = θ, where OX and OY are mutually perpendicular A
fixed lines in the plane.
π/2
Let i, j be the unit vectors along OX and OY respectively. r
Then
rˆ = (cos θ) i + (sin θ) j π/2+θ
X
d dθ dθ O
∴ (rˆ ) = (− sin θ) i + (cos θ) j
dt dt dt

= [(− sin θ) i + (cos θ) j] ...(i)
dt

ap
π
Also let n̂ is an unit vector perpendicualr to OA. Then it will make an angle + θ with OX.
2
Hence we will have
 π   π 

l
nˆ = cos  + θ  i + sin  + θ  j
 2   2 
sC
= (− sin θ) i + (cos θ) j ...(ii)
∴ From (i) and (ii), we get
d dθ
(rˆ ) = (nˆ ) ...(iii)
dt dt
es
Here n̂ is in the sense in which θ increases.
Also from (ii),
d dθ dθ
(nˆ ) = (− cos θ) i + (− sin θ) j
dt dt dt
cc


= − [(cos θ) i + (sin θ) j]
dt
∴ From (i), we get
d  dθ 
(nˆ ) = −   rˆ
Su

 dt 
...(iv)
dt
1.5 RELATION BETWEEN ANGULAR AND LINEAR VECTORS
If v be the velocity of a point P moving in a plane curve and
(r , θ) its polar co-ordinates referred to fixed point O in the plane,
then the angular velocity of P about O is equal to vp / r 2 , where P (r, θ)

p is the perpendicular drawn from O to the tangent at P. φ


Let radius vector OP makes an angle φ with the tangent at P
to the given curve. θ ψ
O
i.e., ∠ OPL = φ 90°
Q L
Let OL = p be the perpendicular from O to tangent at P. Then
from ∆ POL, we have
p = r sin φ
Also, we know that
SuccessClap: Best Coaching for UPSC Mathematics : For Info- 9346856874
Checkout ->22 Weeks Study Plan, Videos, Question Bank Solutions, Test Series

Velocity and Acceleration Along Radial and Transverse Directions 693

dθ or d θ sin φ p ...(i)
r = sin φ = = 2
ds ds r r
[∵ p = r sin φ]
∴ Angular velocity of P
d θ d θ ds p
= = . = 2 .v
dt ds dt r
vp
∴ Angular velocity of P = 2
r
1.6 RADIAL AND TRANSVERSE VELOCITIES AND ACCELERATIONS
(POLAR COORDIANTES)
If a particle moves in a plane curve and if at timer t the position of the particle be at (r , θ),
referred to O as pole and OX as initial line, then the resolved part of velocity at P along the radius

ap
vector OP in the sense of r increasing is called the radial velocity and the resolved part of the
velocity at P along a line through P but at right angles to OP in the sense in which θ increases is
called the transverse velocity. Similarly radial and transverse accelerations are defined.
Radial and Transverse Velocities

l
Let the particles be moving along the curve APQ and it is
sC
at P and Q at time t and t + δ t respectively. The polar co- Q (r+δr, θ+δθ)
ordinates of P and Q with respect to O as pole and OX as initial
line are (r , θ) and (r + δr , θ + δθ) respectively. R
P (r, θ)
Draw a perpendicular QR on OP from Q.
Obviously the displacements of the moving point along and
es
perpendicular to the radius vector OP are PR and QR O X
A
respectively during interval δt.
Radial velocity at P
displacement along OP
cc

= lim
δ t→ 0 δt
PR
= lim
δt →0 δt
Su

OR − OP
= lim
δ t→ 0 δt
(r + δ r ) cos δ θ − r
= lim
δ t→ 0 δt
 (δ θ)2 
(r + δ r ) 1 − + ... − r
 2! 
= lim expanding cos δ θ.
δ t→ 0 δt
(r + δ r ) .1 − r
= lim , neglecting higher powers of δ θ.
δ t→ 0 δt
δ r dr
= lim = = r,
ɺ in the direction of OP
δ t→ 0 δ t dr
Transverse velocity
displacement perpendicular to OP
= lim
δ t→ 0 δt
SuccessClap: Best Coaching for UPSC Mathematics : For Info- 9346856874
Checkout ->22 Weeks Study Plan, Videos, Question Bank Solutions, Test Series

694 Dynamics
RQ
= lim
δ t→ 0 δ t
(r + δ r ) sin δθ
= lim
δ t→ 0 δt
 (δθ)2 
lim (r + δ r ) δ θ − + ...
δ t→ 0  3! 
=
δt
(r + δr ) δθ
= lim , neglecting higher power of δ θ
δ t→ 0 δt
r δθ
= lim , neglecting δ r . δ θ as it is very small.
δ t→ 0 δ t

=r = r θɺ in the sense of θɺ increasing.
dt

ap
Radial and Transverse Accelerations
Let the velocities along and perpendicular to radius vector at P and Q are u, v, u + δu and
v + δv respectively. Then
v + δv

l
u + δu
sC Q
v u

P
es
δθ
θ
O X
cc

Radial acceleration at P along the direction of r increasing


change of velocity along OP during an interval δt
= lim
Su

δ t→ 0 δt
(u + δu ) cos δθ − (v + δv) sin δθ − u
= lim
δ t→ 0 δt
(u + δu ) .1 − (v + δv) δθ − u
= lim
δ t→ 0 δt
Expanding cos δθ and sin δθ and neglecting higher powers of δθ.
δu − v δθ
= lim
δ t→ 0 δt
du dθ
= −v
dt dt
dr dθ
Since u = radial velocity = and v = transverse velocity = r
dt dt
d  dr   d θ   d θ 
∴ Radial acceelration at P = − r
dt  dt   dt   dt 
SuccessClap: Best Coaching for UPSC Mathematics : For Info- 9346856874
Checkout ->22 Weeks Study Plan, Videos, Question Bank Solutions, Test Series

Velocity and Acceleration Along Radial and Transverse Directions 695


2
d 2r  dθ 
−r r − r θɺ 2
 = ɺɺ
2
dt  dt 
And transverse acceleration at P
change in velocity perrpendicular to OP in time δt
= lim
δ t→ 0 δt
[(u + δu ) sin δθ + (v + δv) sin (90 − δθ)] − v
= lim
δ t→ 0 δt
(u + δu ) sin δθ + (v + δv) cos δθ − v
= lim
δ t→ 0 δt
(u + δ θ) (δθ) + (v + δv) (1) − v
= lim , expanding sin δθ, cos δθ
δ t→ 0 δt
and neglecting higher powers of δθ
u δθ + δv d θ dv dr r dθ

ap
= lim =u + , where u = ; v =
δt→ 0 dt dt dt dt dt
dr d θ d  d θ  dr d θ d 2 θ d θ dr
= . + r. = . +r + .
dt dt dt  dt  dt dt dt 2 dt dt

l
d 2θ dr d θ 1d  2 dθ  1 d 2 ɺ
=r +2 = r = ( r θ ),
dt 2
.
dt dt
sC
in the sense in which θ increases.
r dt  dt  r dt

EXAMPELS
1. A particle describes the curve r = ae mθ with a constant velocity. Find the components of
es
velocity and acceleration along the radius vector and perpendicular to it.
Sol. The given path is
r = aem θ ...(i)
2 2
cc

 dr   dθ 
=   + r
 dt 
Also velocity of particle
 dt 
= constant = v (say)
2 2
 dr   dθ 
Su

2
or  dt  +  r dt  = v ...(ii)
From (i), we get
dr dθ dθ
= ame mθ . = mr
dt dt dt
d θ  1  dr
r =
dt  m  dt
or ...(iii)
∴ From (ii) and (iii), we get
2 2
 dr   1 dr  2
 dt  +  m . dt  = v
   
 dr    1  
2 2
2
or  dt  1 +   =v
  m  
2
 dr  v 2 m2
or  dt  = 2
m +1
SuccessClap: Best Coaching for UPSC Mathematics : For Info- 9346856874
Checkout ->22 Weeks Study Plan, Videos, Question Bank Solutions, Test Series

696 Dynamics
dr vm
=
or dt ...(iv)
(m 2 + 1)
∴ From (iii), we get
dθ v
r =
dt ...(v)
(m 2 + 1)
Components of velocity along and perpendicular to the radius vector are given by (iv) and (v).
From (iv) on differentiating, we get
d 2r
=0
dt 2
∴ Component of acceleration along radius vector
2 2
d 2r  dθ   dθ 
= 2 −r  =−r 
dt  dt   dt 
2
 
 1 v
= −   

ap
 r   m2 + 1 
 
v2
=
r (m 2 + 1)

l
and transverse component of acceleration
sC
1 d  2 dθ 
= r
r dt  dt 
 
1 d vr
=  
r dt  (m2 + 1) 
es
 
v dr v2 m
= =. 2
r (m 2 + 1) dt r ( m + 1)
cc

2. If the curve is an equiangular spiral r = aeθ cot α and if the radius vector to the particle
has constant angular velocity, show that the resultant acceleration of the particle makes an angle
2α with the radius vector and is of magnitude v 2 / r , when v is the speed of the particle.
Sol. The curve is given by
Su

r = aeθ cot α ...(i)


Also, we are given that angular velocity

= constant = ω (say) ...(ii)
dt
From (i), we get
dr dθ
= a cot α . eθ cot α = r ω cot α, from (i) and (ii)
dt dt
Again differentaiting, we get
d 2 r dr
2
= ω cot α = r ω 2 cot 2 α
dt dt
∴ Radial acceleration
2
d 2r  dθ 
= 2 − r   = (r ω 2 cot 2 α ) − r ω 2
dt  dt 
= rω 2 (cot 2 α − 1) ...(iii)
and transverse acceleration
1 d  2 dθ  1 d 2 1 dr dr
=  r  = ( r ω ) = . 2rω = 2ω
r dt  dt  r dt r dt dt
SuccessClap: Best Coaching for UPSC Mathematics : For Info- 9346856874
Checkout ->22 Weeks Study Plan, Videos, Question Bank Solutions, Test Series

Velocity and Acceleration Along Radial and Transverse Directions 697


= 2ω (rω cot α ) = 2rω 2 cot α ...(iv)
If β be the angle which the resultant acceleration makes with the radius vector, then
transverse acceleration
tan α =
radial acceleration
2r ω 2 cot α
= , from (iii) and (iv)
r ω 2 (cot 2 α − 1)
(2 cos α / sin α ) sin 2 α 2 sin α cos α
= 2 2
=
cos α − sin α cos 2α
sin 2α
= = tan 2α
cos 2α
or β = 2α
Also, speed of the particle

ap
 dr  2  d θ  2 
=   +  r 
 dt   dt  

l
= [{(rω cot α ) 2 + (r ω ) 2 }] = rω cosec α = v (given)
sC
Again from (iii) and (iv), resultant acceleration.

= [{(radial acceleration)2 + (transverse acceleration)2 }]

= [{r 2ω 2 (cot 2 α − 1)}2 + (2r ω 2 cot α ) 2 ] = [(r 2 ω 4 cosec 4 α )]


es

= rω 2 cosec2 α = v 2 / r , from (iv)


3. The velocity of a particle along and perpendicular to the radius vector are λr and µθ.
Find the path and show that the acceleration along and perpendicular to the radius vector are
cc

 µ
λ 2 r − µ 2θ2 / r and µθ  λ +  respectively.
r
Sol. Given that
Su

dr
= λr ...(i)
dt

and r = µθ ...(ii)
dt
Differentiating (i), we get
d 2r dr
2
=λ = λ2r ...(iii)
dt dt
Dividing (i) by (ii), we get
dr λr µ dr d θ
= or =
r d θ µθ λ r2 θ
Integrating, we get
µ
− = log θ − log c, where c, is constant of integration
λr
µ  θ
or = log   or θ = ce − µ / λr
λr  c
This is the equation of path.
Now radial acceleration
SuccessClap: Best Coaching for UPSC Mathematics : For Info- 9346856874
Checkout ->22 Weeks Study Plan, Videos, Question Bank Solutions, Test Series

698 Dynamics
2
d 2r  dθ 
= − r   = λ 2 r − r (µθ / r )2 , from (ii) andf (iii)
dt 2  dt 
2 2
µ θ
= λ2r −
r
and transverse acceleration.
1 d  2 d θ  1 d  2 µθ  from (ii)
= r = r
r dt  dt  r dt  r 
1 d µ  dθ dr 
= (µ r θ) =  r +θ 
r dt r  dt dt 
 µ
=   (µθ + θ . λr ) from (i) and (ii)
 r
µ 
= µ  + λ .
r 

ap
4. If the radial and transverse velocity of a particle are always proportional to each other,
then
(a) Show that the path is an equiangular spiral.
(b) If in addition, the radial and transverse accelerations are always proportional to each

l
other, show that the velocity of the particle varies as some power of the radius vector.
sC
Sol. (a) We are given that
dr  dθ 
= k r  ...(i)
dt  dt 
1
or dr = k d θ
es
r
Integrating, log r − log c = k θ
where log c is constant of integration
or log (r / c) = k θ or r = cek θ ,
cc

which is an equiangular spiral.


(b) If in addition to (i), we have
2
d 2r  dθ  1 dθ 
− r   = λ .  r2 
2   r dt 
Su

dt dt
2 2
or
d r  1 dr  λ d  2 1 dr 
2
−r   = r  , from (i)
dt  kr dt  r dt  kr dt 
2
d 2r 1  dr  λ d  dr 
or − 2  dt 
= r
dt 2
k r kr dt  dt 
λ  d 2 r dr dr 
= r + . 
kr  dt 2 dt dt 
2
 λ  d 2r 1  1 λ   dr 
or 1 −  2 =  2 + k   dt 
k dt r k
2
d 2r B  dr 
or A 2
=  
dt r dt
where A and B are constants.
2
d 2 r µ  dr 
or =   , where µ = B / A.
dt 2 r  dt 
SuccessClap: Best Coaching for UPSC Mathematics : For Info- 9346856874
Checkout ->22 Weeks Study Plan, Videos, Question Bank Solutions, Test Series

Velocity and Acceleration Along Radial and Transverse Directions 699


d 2 r / dt 2 µ dr
or =
(dr / dt ) r dt
Integrating,
 dr 
log   = µ log r + log D, where D is constant
 dt 
dr
or = D.r µ
dt
dr
Substituing the value of in (i), we get
dt

D. r µ = kr
dt

or r = α . r µ , where α = D / k .
dt
∴ The resultant velocity

ap
2 2
 dr   dθ 
=   + r = (r µ .D) 2 + (α r µ )2
 dt   dt 
= r µ (D2 + α2 ) ∴ velocity ∝ r µ .

l
5. A small bead slides with constant speed v on a smooth wire in the shape of the cardioid
sC dθ  v  θ
r = a (1 + cos θ). Show that the value of is   sec and that the radial component of the
dt  2a  2
acceleration is constant.
Sol. The path of the bead is
r = a (1 + cos θ) ...(i)
es
dr dθ
∴ = a ( − sin θ) ...(ii)
dt dt
2
d 2r  dθ  d 2θ
and = ( − a cos θ )   − ( a sin θ ) ...(iii)
cc

dt 2  dt  dt 2
∴ Speed of the bead
2 2
 dr   dθ 
=   +r
 dt   dt 
Su

2 2
 dθ   dθ 
=  ( − a sin θ) +  a (1 + cos θ) 
 dt   dt 

= a [sin 2 θ + (1 + cos θ) 2 ]
dt

= a 2 (1 + cos θ)
dt
 θ  dθ
=  2a cos  = v (given)
 2  dt
dθ  v   θ
i.e., =   sec   ...(iv)
dt  2a   2
Now, radial acceleration of the bead
2
d 2r  dθ 
= 2 −r 
dt  dt 
SuccessClap: Best Coaching for UPSC Mathematics : For Info- 9346856874
Checkout ->22 Weeks Study Plan, Videos, Question Bank Solutions, Test Series

700 Dynamics
  dθ 
2
d 2θ   dθ 
2
= ( − a cos θ)   − (a sin θ) 2  − a (1 + cos θ)   ,
  dt  dt   dt 
from (i) and (iii)
2
 dθ  d 2θ
= − a (1 + 2 cos θ)   − (a sin θ) 2 ,
 dt  dt
2
 v θ  v 1 θ θ dθ 
= − a (1 + 2 cos θ) .  sec  − a sin θ  sec tan ,
 2a 2  2 a 2 2 2 dt 
from (iv)
2 2
 v θ 1 θ  v θ
= − a (1 + 2 cos θ) .  sec  − a tan sin θ  sec 
 2a 2 2 2  2a 2
 v2  θ 1 θ 
= − a  2  sec 2  (1 + 2 cos θ) + tan sin θ 
 4a  2 2 2 

ap
1  v2  θ θ
=−   sec2 (1 + 2 cos θ) + sin
2

4 a 2 2
1  v2  θ  1 
=−   sec2  (1 + 2 cos θ) + 2 (1 − cos θ) 

l
4 a 2  
sC
1  v2 
=−   sec2
θ 3 
(1 + cos θ) 

2 2
4 a 
3  v2   2 θ  2 θ
=−    sec   cos 
es
4 a 2 2
2
 3 v
=−  = constant .
 4 a
cc

6. One end of a rod describes a plane curve and the rod always passes through a fixed point
in the plane of the curve. If the angular velocity of the rod is constant, show that the transverse
acceleration of every point of the rod is the same at the same
instant. What curve must the end describe to make this
A
Su

acceleration the same at every instant ?


Sol. Let the co-ordinates of the end A be (r , θ) referred to
fixed point O as pole. P
where P is any point on the rod AB. Then OP = r − d .
The transverse acceleration at P θ
d  O X
1 2 dθ 
=  (r − d ) . 
( r − d ) dt  dt  B
Also angular velocity of the rod

= = constant = ω (say)
dt
1 d
∴ The transverse acceleration of P = [(r − d ) 2 ω ]
(r − d ) dt
1 dr dr
2 (r − d ) ω . = 2ω .
(r − d ) dt dt
This is free from d and hence will be same for every point of the rod at the same instant.
If this acceleration be same at every instant also, then
SuccessClap: Best Coaching for UPSC Mathematics : For Info- 9346856874
Checkout ->22 Weeks Study Plan, Videos, Question Bank Solutions, Test Series

Velocity and Acceleration Along Radial and Transverse Directions 701


dr dr
2ω = constant or = λ (say)
dt dt
dr dθ
∴ We have = λ and =ω
dt dt
Dividing these two, we get
dr λ
= = k say or dr = k d θ
dθ ω
Integraing, we get r = k θ + c.
This is the required equation of the curve.
7. A straight smooth tube revolves with angular velocity ω in a horizontal plane about one
extremity which is fixed, if at zero time a particle inside it be at a distance a from a fixed end and
moving with velocity V along the tube, show that its distance at time t is
V 
a cosh ωt +   sinh ωt R
 ω
Sol. Let initially OX be the position of the tube and A that of the

ap
P
particle inside it.
Let P be the position of the particle at any time t. r

Let OP = r and ∠ POX = θ. θ


O X
Since the particle is moving with constant velocity V along the A

l
tube, i.e., along the radius vector, hence the radial acceleration of the
sC
particle will be zero throughout the motion. Hence,
2
d 2r  dθ 
−r  =0 ...(i)
dt  dt 
Its solution is
es
r = c1 cosh ωt + c2 sinh ωt ...(ii)

where c1 and c2 are constant and = ω.
dt
cc

Initially t = 0 and r = a,
∴ from (ii) c2 = a.
Differentiating (ii)
dr
= c1ω sinh ωt + c2 ω cosh ωt.
Su

dt
dr
Initially t = 0, = V.
dt
Hence V = c2 ω or c2 = V / ω
∴ From (ii) we get
r = a cosh ωt + (V / ω) sinh ωt
This gives the distance of the particle from O at time t.
8. An insect crawls at a constant rate u along the spoke of a cart wheel of radius a, the cart
moving with velocity v. Find the acceelrations along and
perpendicular to the spoke. A
P
Sol. Let the initial position of spoke be OA and that of insect
be O. At any time t, let the position of spoke be OP and that of insect θ
Q v
be Q. r v
Let ∠ AOP = θ and OQ = r. O
Since the insect is crawling at a constant rate u along the spoke,
we have
SuccessClap: Best Coaching for UPSC Mathematics : For Info- 9346856874
Checkout ->22 Weeks Study Plan, Videos, Question Bank Solutions, Test Series

702 Dynamics
r = ut ...(i)
The cart is moving with constant velocity v , hence the acceleration of the point O is zero and
dθ v v
= = (where a is the radius of wheel)
dt OP a
∴ Acceleration of the insect along OP
= radial acceleration
2
d 2r  dθ 
= −r 
dt 2  dt 
 v
2
− rv 2  d 2 r d  dr  du 
=−0−r  = ∵ 2 =   = = 0
 a a  dt dt dt dt 
Acceleration perpendicular to spoke.
= Transverse acceleration
1 d  2 dθ  1 d  2 v 

ap
= r = r 
r dt  dt  r dt  a 
2 dr 2uv dv
= v = ∵ = 0.
a dt a dt

l
EXERCISES
sC
1. A point P describes an equiangular spiral r = aeθ cot α with constant angular velocity about
the pole O. Find its acceleration and show that its direction makes the same angle with the
tangent at P as the radius vector OP makes with the tangent.
es
2. A particle describe an equiangular spiral r = aeθ in such a manner that its acceleration has
no radial component. Prove that its angular velocity is constant and that the magnitude of
the velocity and acceleration is each proportional to r.
3. If the angular velcotiy of a point moving in a plane curve be constant about a fixed origin,
cc

show that its transverse acceleration varies as its radial velocity.


4. If the radial and transverse velocities of a particle are always equal, then prove that the
particle describes an equiangular spiral.
5. A particle moves in a circular path of radius a, so that its angular velocity about a fixed
Su

point in the circumference is constant and equal to ω. Show that the resultant acceleration
2
of the particle at evey point is constant magnitude 4aω .
6. A point P describes, with a constant angular velocity about O, the equiangular spiral
r = aeθ , O being the pole of the spiral. Obtain the radial and transverse acceleration of P.
[Ans. 0, 2ω 2 r ]
7. A particle moves along a circle r = 2a cos θ in such a way that its acceleration towards the
origin is always zero. Show that the transverse acceleration varies as the fifth power of
cosec θ.
8. A particle P describes a curve with constant velocity and its angualr velocty about a given
fixed point O varies inversely as its distance from O. Show that the curve is an equiangular
spiral.
9. The velocities of a particle along and perpendicular to a radius vector from a fixed origin are
λ µ
λ r 2 and µ θ 2 . Show that the equation to the path is = + c and the components of
θ 2r
θ4 θ3
accelerations are 2λ 2 r 3 − µ 2 and λµ r θ2 + 2µ 2 .
r r
SuccessClap: Best Coaching for UPSC Mathematics : For Info- 9346856874
Checkout ->22 Weeks Study Plan, Videos, Question Bank Solutions, Test Series

Velocity and Acceleration Along Radial and Transverse Directions 703


10. A point P describes, with a constant angular velocity about O, the equiangular spiral
r = eθ , O being the pole of the spiral. Obtain the radial and transverse accelerations of P.

[Ans. 0, 2ω 2 r ]
11. A ring which can slide on a thin long smooth rod rests at a distance d from one end O. The
rod is then set revolving uniformly about O in a horizontal plane. Show that in space the
ring describes the curve r = d cos hθ.
12. Show that the path of a point P which possesses two constant velocities u and v, the first of
which is in a fixed direction and the other is perpendicular to radius OP drawn from a fixed
point O, is a conic section and the other is perpendicular to radius OP drawn from a fixed
point O, is a conic whose focus is O and whose eccentricity is u / v.
13. The acceleration of a point moving in a plane curve is resolved into two components, one
parallel to the initial line and the other along the radius vector. Prove that these components

ap
are
1 d 2ɺ cot θ d 2
− (r θ) and r − rθɺ 2
. (r θ) + ɺɺ
r sin θ dt r dt

l
14. A point moves on a parabola 2a = r (1 + cos θ) in such a manner that the component of
sC
velocity at right angles to the radius vector from the focus is constant. Show that accelera-
tion of the point is constant in magnitude.
15. A point describes a circle of radius a with a uniform speed v, show that the radial and trans-
verse accelerations are − (v 2 / a ) cos θ and − (v 2 / a ) sin θ if a diameter is taken as initial
es
line and one end of its diameter as pole.
16. If a rod which always passes through the origin rotates with uniform angular velocity ω,
while one end describes the curve r = a + beθ , show that the radial acceleration of any
cc

point of the rod is the same at every instant, and the radial velocity is the same at every
point at a given instant.

❑❑❑
Su
SuccessClap: Best Coaching for UPSC Mathematics : For Info- 9346856874
Checkout ->22 Weeks Study Plan, Videos, Question Bank Solutions, Test Series

Tangential and Normal Velocities


and Accelerations
2.1 TANGENTIAL AND NORMAL VELOCITIES AND ACCELERATIONS
Tangential and normal velocities and accelerations are defined as the resolved parts of the
velocities and accelerations along the tangent and normal respectively.
Tangential and Nurmal Velocities Q A
Let a moving point moves along the curve APQ. At time t, let it

ap
ε N
comes to a point P and at time t + δt at point Q.
Let the co-ordinates of P and Q be ( s, ψ ) and ( s + δs, ψ + δψ) P
respectively. From Q, let us draw a perpendicular on the tangent at P r
A

l
which is QN and let the ∠ QPN = ε. Join chord PQ, then ψ
O X
Tangential velocity at P
sC
displacement along the tangent at P in time δt
= lim
δt → 0 δt
PN
es
= lim
δ t → 0 δt
(chord PQ) . cos ε
= lim
δt → 0 δt
cc

chord PQ arc PQ
= lim . cos ε
δ t → 0 are PQ δt
δs  chord PQ 
= lim 1. . cos ε ∵ lim = 1 and arc PQ = δs 
Su

δt → 0 δt  δ t → 0 arc PQ 
Now as Q tends to P, ε will be zero and cos ε tends to 1
δs ds
∴ tangentail velocity at P = lim =
δt → 0 δt dt
Similarly,
Normal velocity at P
displacement along the normal at P in time δt
= lim
δt → 0 δt
QN
= lim
δ t → 0 δt
(chord PQ) sin ε
= lim
δt → 0 δt
chord PN arc PQ
= lim . . sin ε
δ t → 0 arc PQ δt
SuccessClap: Best Coaching for UPSC Mathematics : For Info- 9346856874
Checkout ->22 Weeks Study Plan, Videos, Question Bank Solutions, Test Series

Tangential and Normal Velocities and Accelerations 705


Here as Q tends to P, ε tends to zero, and sin ε tends to zero.
∴ Normal velocity at P = O
Remark : Please note that in this case that total velocity is along the tangent.
Tangential and Normal Accelerations : Here let the
velocity at P be v along the tangent which is making an angle
v + δv
ψ with the initial line and the velocity at Q be v + δv along
the tangent at Q, making an angle ψ + δψ with OX. v

δψ
∴ Tangential Acceleration at P
Change in velocity along the tangent at P in time δt
= lim P
δt → 0 δt
S
A
(v + δv) cos δψ − v
= lim
δt → 0 δt ψ y + δy

ap
(v + δv) .1 − v
= lim
δt → 0 δt
[∵ cos δψ = 1, neglecting higher powers of δψ]

l
δv dv
= lim =
δt → 0 δt dt
sC
d  ds  d 2 s
=  = = ɺɺs
dt  dt  dt 2
es
d 2 s d  ds  ds dv
Also =   = v
dt 2 ds  dt  dt ds
Again, Normal Acceleration at P
cc

change in velocity along the normal at P in time δt


= lim
δt → 0 δt
(v + δv) sin δψ
= lim
Su

δt → 0 δt
sin δψ δψ
= lim (v + δv) .
δt → 0 δψ δt

=v.
dt
d ψ ds v 2
=v. . =
dt dt ρ
EXAMPLES
1. A point describes the cycloid s = 4a sin ψ with uniform speed v. Find its acceleration at
any point.
Sol. The equation of the cycloid is
s = 4a sin ψ
Also given that
ds
= v = constant
dt
SuccessClap: Best Coaching for UPSC Mathematics : For Info- 9346856874
Checkout ->22 Weeks Study Plan, Videos, Question Bank Solutions, Test Series

706 Dynamics
d 2s
∴ =0
dt 2
Hence tangential acceleration = 0
Differentiating eqn. (i) with respect to ψ, we get
ds
= 4a cos ψ, or ρ = 4a cos ψ

∴ The resultant acceleration
  2  2  2  2  
 d s v 
=    +    

  dt 
2
   
ρ

  v2  
2
v2
= 0 +    =
  4a cos ψ   4a (1 − sin 2 ψ )

v2 v2

ap
= =
 s2  [(16a 2 − s 2 )]
4a  1 − 2 
 16a  
2. A point moves in a plane curve, so that its tangential and normal accelerations are equal

l
and the angular velocity of the tangent is constant. Find the curve.
Sol. Given that
sC dv v 2
v = ...(i)
ds ρ

and = constant = ω (say). ...(ii)
es
dt
dv v v dψ
From (i) = = =v
ds ρ (ds / d ψ ) ds
or dv = v d ψ
cc

dv
or = dψ
v
Integrating, we get
log v = ψ + log c
Su

where log c is the constant of integration.


v
or log = ψ or v = ceψ
c
ds ds d ψ
or = ceψ or = ceψ
dt d ψ dt
ds c ψ
or = e , from (i)
dψ ω
c
ds = eψ d ψ
ω
Integrating, we get
c
s =   eψ + k
 ω
where k is a constant of integration.
or s = Aeψ + B.
where A and B are arbitrary constants. This is the required intrinsic equation of the curve.
SuccessClap: Best Coaching for UPSC Mathematics : For Info- 9346856874
Checkout ->22 Weeks Study Plan, Videos, Question Bank Solutions, Test Series

Tangential and Normal Velocities and Accelerations 707


3. A particle is describibng a plane curve. If the tangential and normal accelerations are
each constant throughout the motion, prove that the angle ψ, through which the direction of
motion turns in time t is given by ψ = A log (1 + Bt ).
Sol. Given that
d 2s
=k ..(i)
dt 2
v2
and = λ, ...(ii)
ρ
where k and λ are constants.
Integrating (i), we get
ds
= kt + c ...(iii)
dt
where c is constant of integration.
From (ii), we get
v2 ( ds / dt ) 2
=λ or =λ

ap
(ds / d ψ ) ( ds / d ψ )
ds d ψ
or . =λ
dt dt

(kt + c) = λ, from (iii)

l
or
dt

or
sC
dψ =
λ
(kt + c)
dt
Integrating, we get
λ
ψ =   log (kt + c) + log µ,
es
k
where log µ is a constant of integration.
Let ψ = 0, where t = 0, then
0 = (λ / k ) log c + log µ
cc

λ λ
∴ ψ = (kt + c) − log c
k k
λ  kt + c 
= log  
Su

k  c 
λ  kt 
or ψ =   log 1 + 
k  c
or ψ = A log (1 + Bt ),
where A = λ / k and B = k / c.
4. A particle describes a curve (for which s and ψ vanish simultaneously) with uniform speed
v2c
v. If the accleration at any point s be , find the intrinsic equation of the curve.
s2 + c2
Sol. Given that :
ds
= v (constant),
dt
d 2s
so, =0
dt 2
∴ Acceleration at any point
SuccessClap: Best Coaching for UPSC Mathematics : For Info- 9346856874
Checkout ->22 Weeks Study Plan, Videos, Question Bank Solutions, Test Series

708 Dynamics
2
 d 2 s   v2  v2
=  2 +  =
 dt   ρ  ρ
Now, it is given that
v2 v2c
Acceleration at any point = 2
ρ s + c2
1 c dψ c
or = 2 or = 2
ρ s + c2 ds s + c2
1 ds
or dψ = 2
c s + c2
Integrating, we get
 1 1 −1  s 
  ψ + A = tan  
c c c
where A is constant at integration.

ap
when ψ = 0, s = 0 (given), ∴ A = tan −1 0 = 0.
 1  1 −1
Hence,   ψ =   tan ( s / c)
c c

l
or sC s = c tan ψ.
5. A curve is described by a particle having a constant acceleration in a direction inclined
at a constant angle to the tangent. Show that the curve is an equiangular spiral.
Sol. Let at any instant particle be at P. The direction of tangential and normal acceleration at P
are as shown in the figure. The resultant acceleration makes a v2
constant angle α (say) with the tangent at P.
es
ρ
vδv
Hence
ds
2
v /ρ α
tan α =
(v dv / ds ) P
cc

dv ds dv
or v = (ρ tan α ) = tan α .
ds d ψ ds
1
Su

or dv = cot α d ψ.
v
Integrating, we get
log v = (cot α ) ψ + log c,
where log c is integration constant.

∴ v = ceψ cot α ...(ii)


Also given that acceleration is constant,
Therefore, we have
2
 v2   dv 
2
  +  v  =k ...(iii)
 ρ ds 
∴ from (i) and (iii), we get
2 2
 dv   dv 
tan 2 α  v  +  v  = k 2
 ds   ds 
SuccessClap: Best Coaching for UPSC Mathematics : For Info- 9346856874
Checkout ->22 Weeks Study Plan, Videos, Question Bank Solutions, Test Series

Tangential and Normal Velocities and Accelerations 709


2
 dv  2 2
or  v  sec α = k
ds
dv
or v = k cos α
ds
or v dv = k cos α ds
Integrating, we get
v 2 = (2k cos α ) s + c1
where c1 is constant of integration.
or v = [(2ks cos α + c1 )]
∴ From (ii) and (iv), we get
[(2ks cos α + c1 )] = ce ψ cot α
or 2ks cos α + c1 = c 2 e2ψ cot α
This is the intrinsic equation of the equiangular spiral.

ap
6. A particle moves in a catenary s = c tan ψ. The direction of its accleration at any point
makes equal angles with the tangent and the normal to the path at that point. If the speed at the
vertex (where ψ = 0) be u, show that the velocity and acceleration at any other point ψ are
2 2ψ 2
ueψ and { (2 / c )} u e cos ψ .

l
Sol. Since the direction of the resultant acceleration makes equal angles with the tangent and
sC
the normal hence the tangential and normal accelerations are equal.
dv v 2
i.e., v = ...(i)
ds ρ
dv 1 dψ
es
or = ds = . ds = d ψ
v ρ ds
Integrating, we have
log v = ψ + log k , where log k is constant of integration.
cc

When ψ = 0, v = u, so we have
∴ log v = ψ + log u
or log (v / u ) = ψ or v = ueψ ...(ii)
This gives velocity at any point.
Su

Now, the equation of catenary is


s = c tan ψ
ds
∴ radius of curvature ρ = = c sec2 ψ ...(ii)

∴ Resultant acceleration
 2  2 2
 dv  v 
=  v  +   
  ds  ρ
  

 2  2  2  2 
 v v  v2
=   +    = 2
 ρ   ρ 

ρ

u 2 e2 ψ
= 2 , from (ii) and (iii)
c sec 2 ψ
= ( 2 / c) u 2 e2 ψ cos 2 ψ.
SuccessClap: Best Coaching for UPSC Mathematics : For Info- 9346856874
Checkout ->22 Weeks Study Plan, Videos, Question Bank Solutions, Test Series

710 Dynamics

7. A particle is moving in a parabla p 2 = ar with uniform angular velocity about the focus,
prove that its normal acceleration at any point is proportional to the radius of curvature of its
path at that point.
Sol. The equation of the parbala
p 2 = ar. ...(i)
Differentiating,
dp dp a
2p = a or =
dr dr 2 p
dr r 2 pr
∴ Radius of curvature ρ = r = = . ...(ii)
dp a / 2 p a

Angular velocity = = constant = ω (say)
dt
d θ vp vp
Also, = 2 or ω = 2 ...(iii)

ap
dt r r
ωr 2
or v= .
p
v2

l
∴ Normal acceleration = ...(iv)
ρ


sC
Normal acceleration v 2
= ...(v)
Radius of Curvature ρ2
ω2r 4 / p2
= , from (ii) and (v)
es
4 p2r 2 / a2
r 2ω2 a2 r 2ω2 a2
= = , from (i)
4 p4 4a 2 r 2
cc

ω2
=
= constant = k (say)
4
∴ normal acceleration α radius of curvature.
Su

EXERCISES
1. The rate of the change of direction of velocity of a particle moving in a cycloid is constant.
Prove that acceleration must be constant in magnitude.
2. A particle describes a circle of radius r with a uniform speed v, show that its acceleration at
any point of the path is v 2 / r and is directed towards the centre of the circle.
3. Prove that acceleration of a point moving in a curve with unifrom speed is ρψ 2 .
4. A particle describes cycloid with uniform speed. Prove that normal acceleration at any point
varies inversely as the distance from the base of the cycloid.
5. A particle describes a plane curve with a constant speed and its acceleration is constant in
magnitude. Prove that the path is circle.
6. If tangential and normal acceleration components of a particle be equal. Prove that its velocity
varies as e ψ .
7. The tangential acceleration of a particle moving along a circle of radius a is λ times the
normal acceleration. If the speed at a certain time is u, prove that it will return to the same
 a  −2 πλ
point after a time   (1 − e ).
λu
SuccessClap: Best Coaching for UPSC Mathematics : For Info- 9346856874
Checkout ->22 Weeks Study Plan, Videos, Question Bank Solutions, Test Series

Tangential and Normal Velocities and Accelerations 711


8. A point moves in a plane curve so that its tangential acceleration is constant and the
magnitude of the tangential velocity and normal acceleration are in a constant ratio. Find the
intrinsic equation of the curve.
9. The velocity of a point moving in a plane curve varies as the radius of curvature. Show that
the direction of motion revolves with constant angular velocity.

 ds ds dψ 
Hint. v = kρ or =k or d ψ = k dt or = k = const.
 dt dψ dt 
10. If the tangential and normal acceleration of a particle describing a plane curve be constant
throughout, prove that the radius of curvature at any point is given by ρ = ( at + b) 2 .
11. A particle moves in a plane in such a manner that its tangential and normal accelerations are
always equal and its velocity varies as exp (tan −1 s / c), s being the length of arc of the

ap
curve measured from a fixed point on the curve. Find the path. [Ans. s = c tan ψ ]
12. The direction of the acceleration of a particle moving in a cycloid makes with the normal an
angle equal to that which the tangent to the cycloid at the point makes with tangent at the
vertex and is in the same sense. Prove that the tangent at the point turns uniformly, and that

l
the magnitude of the acceleration is constant.
sC
13. One point describes the diameter AB of a circle with constant velocity and another the semi-
circumference AB from rest with constant tangential acceleration. They start together from A
and arrive together at B. Show that velocities at B are π :1.
es
14. A particle, projected with a velocity u is acted on by a force which produces a constant
acceleration f in the plane of the motion inclined at a constant angle α with the direction of
motion. Obtian the intrinsic equation of the curve described and show that the particle will
be moving in the opposite direction to that of projection at time
cc

u
(eπ cot α − 1).
tan α

❑❑❑
Su
SuccessClap: Best Coaching for UPSC Mathematics : For Info- 9346856874
Checkout ->22 Weeks Study Plan, Videos, Question Bank Solutions, Test Series

Simple Harmonic Motion and


Elastic Strings
3.1 SIMPLE HARMONIC MOTION
A particle is said to execute Simple Harmonic Motion, if it moves in a straight line such that
its acceleration is always directed towards a fixed point in the line and is proportional to the
distance of the particle from the fixed point. Simple Harmonic Motion is abbreviated as S.H.M.
Let O be the fixed point and the particle starts from rest
x

ap
from the point A such that PA = a. Let after time t the particle
A' P' O P A
be at P, such that OP = x.

dx d2x
and 2 will be the velocity and the acceleration of the particle at time t acting in

l
Then
dt dt
sC
the direction in which x increases.
Since in this case the acceleration is directed towards the fixed point O and is proportional to
the distance of the particle from O, hence the equation of motion will be
d2x
m 2 = −λx
es
dt
d2x  λ
or = − µ x, where µ =   > 0
dt 2  m
cc

dv
or v = − µ x. ...(i)
dx
The differential equation (i) represents S.H.M. and is used generally as definition of S.H.M. If
the equation of motion of a particle is of the form of (i) then we can at once say that the particle is
Su

executing S.H.M.
Integrating (i) with respect to x, we get
1 2 1
v = − µ x 2 + A,
2 2
where A is constant of integration. Initially,
at A, x = a and v = 0
1 1
∴ 0 = − µa 2 + A or A = µa 2
2 2
1 2 1 2 2
∴ v = µ (a − x ).
2 2
or v 2 = µ (a 2 − x 2 ) ...(ii)
dx
or = − µ {(a 2 − x 2 )} ...(iii)
dt
the negative sign is to be taken here as the particle is moving towards O, i.e.,; x decreases as t
increases.
From (iii), we have
SuccessClap: Best Coaching for UPSC Mathematics : For Info- 9346856874
Checkout ->22 Weeks Study Plan, Videos, Question Bank Solutions, Test Series

Simple Harmonic Motion and Elastic Strings 713


dx
− = {(µ )} t + B
{( a 2 − x 2 )}
Integrating, we get
cos −1 ( x / a ) = {(µ )} t + B ...(iv)
where B is a constant of integration
At A, x = a and t = 0, so from (iv), we get B = 0, and hence
cos −1 ( x / a ) = (µ ) t
or x = a cos { (µ ) t} ...(v)
Equation (i), (iii) and (v) give the acceleration, velocity and the position of the particle at time t.
Nature of Simple Harmonic Motion
From (ii), v = 0 at x = ± a, thus if A′ is a point on the other side of O such that OA = OA′ = a,
the particl comes to rest at A′ also. When x = 0, v = ± a µ , i.e., at O the velocity is a µ .
π

ap
From (v) we have at x = 0, cos (µ ) t = 0 ∴ (µ ) t = ,
2
π
i.e., time from A to O is equal to .
2 (µ )
At x = − a,

l
sC
cos (µ ) t = − 1,

(µ ) t = π, i.e., time from A to A′ is


π
which is double the time from A to O.

µ
d2x π
es
At A, 2
= − µa and due to this attraction the particle moves towards O and in time
dt 2 µ
the particle reaches O with velocity a µ . At O, the attraction ceases but due to velcoity a µ in
the negative direction, the particle crosses O and moves towards the point A′. As soon as the
cc

particle crosses O it is attracted towards O and due to this, its velocity decreases and becomes zero
at A′. At A′ although the velocity of particle becomes zero, but due to attraction it again moves
towards O and reaches O with velocity a µ . Due to this velocity the particle moves from O towards
Su

A and again stops at A where its velocity becomes zero. Thus the motion is repeated again and
again. Thus the motion is repeated again and again. Thus the motion is oscillatory i.e., from A to
A′ and back to A and so on.
Period. The time from A to A′ is known as the period of the motion and this is given by
π 2π
T = 2. =
µ µ
The frequency is the number of complete oscillations in one second, so that if n be the frequency
and T the periodic time then,
1 µ
n= =
T 2π
The distance a (= OA), i.e., the distance of the centre from one of the position of rest is called
the amplitude.
From (i), we have
d2x
+ µx = 0.
dt 2
The most general solution of this equation is
x = a cos { (µ ) t + ξ}.
SuccessClap: Best Coaching for UPSC Mathematics : For Info- 9346856874
Checkout ->22 Weeks Study Plan, Videos, Question Bank Solutions, Test Series

714 Dynamics
In this relation the quantity ξ is called the epoch and the angle ( (µ ) t + ξ) is called argument.
The particle is at its maximum distance at time t0 , where
(µ ) t0 + ξ = 0
ξ
i.e., t0 = − .
µ
The phase at a point is defined as the time that has elapsed since the particle was at its maximum
distance in the positive direction, i.e.,
ξ (µ ) t + ξ
t − t0 = t + = ,
µ µ
this is phase at time t.
3.1.1 Geometrical Representaion of S.H.M.
If a particle describes a circle with constant angular velocity, the foot of the perpendicualr
from the particle on a diameter moves with S.H.M.

ap
Let a point P moves along the circumference of circle with centre O, P
with uniform speed. Let P describes equal arcs in equal times. Let the
rate of description of ∠ POA is ω.
ωt
Let N be the foot of the perpendicular drawn from P on the diameter B A

l
sC O x N
AOA′.
Let the particle starts from A and reaches P after time t. Then
∠ AOP = ωt.
If ON = x and OP = a, then from ∆ OPN , we have
es
x = a cos ω t ...(i)
dx
Hence, = − aω sin ωt ...(ii)
dt
cc

d2x 2 2
and = − aω cos ωt = − ω x ...(iii)
dt 2
Equations (ii) and (iii) represent the velocity and acceleration of N. Also, as P moves along the
Su

circumference of the circle, N oscillates from A to A′ and back to A. Thus the motion of N is periodic.
The periodic time will be the time taken by P in moving once along the whole circumference of the
circle, i.e., the time taken by P to turn through an angle 2π with the uniform rate of ω.

∴ Periodic time of N =
ω
EXAMPLES
1
1. Show that the particle executing S.H.M. requires th of its period to move from the position
6
of maximum displacement to one in which the displacement is half the amplitude.
Sol. The equation of motion of the particle executing S.H.M. is
d2x
= − µx ...(i)
dt 2

then the period = = T (say) ...(ii)
µ
SuccessClap: Best Coaching for UPSC Mathematics : For Info- 9346856874
Checkout ->22 Weeks Study Plan, Videos, Question Bank Solutions, Test Series

Simple Harmonic Motion and Elastic Strings 715


From (i), we get
2
 dx  2 2
  = µ (a − x ),
dt
where a is amplitude.
dx
or = − µ [(a 2 − x 2 )]
dt
negative sign indicates that particle is moving towards the centre.
dx
or dt = −
µ [( a 2 − x 2 )]
Hence, the time T, taken by the particle in moving from the position of maximum displacement,
1
i.e., x = a to position when the displacement is half the amplitude, i.e., x = a will be
2

ap
a/2
1 a/2 dx 1  −1  x  
T1 =
µ
∫a =  cos   
µ  a a
[(a 2 − x 2 )]

l
1  −1  1  −1  1 π 
=
µ  2
sC
cos   − cos (1)  =

 − 0
µ 3
π 1
= = T, from (ii)
3 µ 6
es
2. Show that if the displacement of a particle moving in a straight line is expressed by the
equation x = a cos nt + b sin nt , a S.H.M. whose amplitude is ( a 2 − b 2 ) and period is 2 π / n.
Sol. We have
cc

x = a cos nt + b sin nt ...(i)


dx
= − an sin nt + bn cos nt
dt
Su

d2x
and 2
= − an 2 cos nt − bn2 sin nt
dt
= − n 2 (a cos nt + b sin nt )

d2x
or 2
= − n2 x
dt
d2x
This equation is of the form = − µx,
dt 2
hence represents S.H.M., where µ = n 2
2π 2π 2π
∴ Time period = = = .
µ n2 n

Also, amplitude of the motion is the value of x when velocity (dx / dt ) is zero.
SuccessClap: Best Coaching for UPSC Mathematics : For Info- 9346856874
Checkout ->22 Weeks Study Plan, Videos, Question Bank Solutions, Test Series

716 Dynamics

dx
∴ by = 0, we get − an sin t + bn cos nt = 0
dt
b
or tan nt = b / a. This gives sin nt =
a 2 + b2

a
and cos nt =
a + b2
2

∴ Amplitude, from (i)


a b a 2 + b2
= a. + b. =
a 2 + b2 a 2 + b2 (a 2 + b2 )

= (a 2 + b 2 ).

ap
3. At the ends of three successive seconds the distances of a point moving with S.H.M. from its
mean position measured in the same direction are 1, 5 and 5. Show that the period of a complete

oscillation is .

l
−1  3 
cos  
 5
sC
Sol. If x be the distance of the particle from its mean position at time t, then for S.H.M.
x = a cos µ t ...(i)
where µ is the intensity and a the amplitude of the S.H.M.
es
According to the given problem
1 = a cos { µ (T − 1)} ...(ii)
cc

5 = a cos ( µ T ) ...(iii)

and 5 = a cos { µ (T + 1)} ...(iv)


considering three successive seconds as T − 1, T and T + 1.
Su

Adding (ii) and (iv), we get


1 + 5 = a [cos { µ (T − 1)} + cos { µ (T + 1)}]

or 6 = a [2 cos ( µ T ) cos µ ] or 3 = 5 cos ( µ ), from (iii)

3  3
or cos µ = or µ = cos −1  
5  5
∴ The period of a complete oscillation
2π 2π
= =
µ  3
cos −1  
 5
4. A point executes S.H.M. such that in two of its positions the velocities are u, v and the
v2 − u 2
corresponding accelerations α, β. Show that the distance between the positions is and
α −β
SuccessClap: Best Coaching for UPSC Mathematics : For Info- 9346856874
Checkout ->22 Weeks Study Plan, Videos, Question Bank Solutions, Test Series

Simple Harmonic Motion and Elastic Strings 7 1 7

{(v 2 − u 2 ) (α 2 v 2 − β 2 u 2 )}1/ 2
amplitude of the motion is .
(β 2 − α 2 )
Sol. Let at distances x1 and x2 from the centre the velocities be u and v and accelerations be
α and β respectively.
Let µ be the intensity and a the amplitude of S.H.M., then we have
d2x
= − µx ...(i)
dt 2
2
 dx  2 2
and   = µ (a − x ) ...(ii)
dt
From (i), we have
α = − µx1 ...(iii)
and β = − µx2 ...(iv)

ap
From (ii), we get
u 2 = µ (a 2 − x12 ) ...(v)
and v 2 = µ ( a 2 − x22 ) ...(vi)
Adding (iii) and (iv), we get α + β = − µ ( x1 + x2 )

l
v 2 − u 2 = µ ( x12 − x22 )
and from (v) and (vi),
sC
v2 − u 2 µ ( x12 − x22 )
Dividing, we get =
α+β − µ ( x1 + x2 )
= x2 − x1
es
v2 − u 2
i.e., the required distance between the positions = x2 − x1 =
α+β
Again, from (iii) and (iv), we get
β 2 − α 2 = µ 2 ( x22 − x12 )
cc

v2 − u 2 µ ( x12 − x22 ) 1
∴ 2 2
= 2
=−
β −α µ ( x22 − x12 ) µ
2  α2 
Su

u
From (v) a2 = + x12 = (u 2 / µ ) +  2  , From (iii)
µ µ 
u 2 (v 2 − u 2 ) α 2 (v 2 − u 2 ) 2
=− +
(β 2 − α 2 ) (β 2 − α 2 )
(v 2 − u 2 )
= [ − u 2 (β 2 − α 2 ) + α 2 (v 2 − u 2 )]
(β 2 − α 2 ) 2
= (v 2 − u 2 ) (v 2 α 2 − u 2β 2 ) /(β 2 − α 2 )2
{(v 2 − u 2 ) (v 2 α 2 − u 2β 2 )}1/ 2
or amplitude = a = .
(β 2 − α 2 )
5. A particle is performing a S.H.M. of period T about a centre O and it passes through a
point P where OP = b with velocity v in the direction OP prove that time which elapses before it
returns to P is
T 1  vT 
  tan  .
π 2 πb 
SuccessClap: Best Coaching for UPSC Mathematics : For Info- 9346856874
Checkout ->22 Weeks Study Plan, Videos, Question Bank Solutions, Test Series

718 Dynamics
Sol. Given that OP = b. Let us take a point Q on the line OP, such that OQ = x. Then the
velocity at Q is given by
dx
= µ [(a 2 − x 2 )] ...(i)
dt
where OA = a is the amplitude of S.H.M.
Now, it is given that velocity at P = v, i.e., at x = b. Hence from (i) we get
v = µ [(a 2 − b 2 )] ..(ii)
Also, from (i), we get
dx
= (µ ) dt
[(a − x 2 )]
2
Integrating, we get
sin −1 ( x / a ) = (µ ) t + c,
where c is an arbitrary constant.
Let at t = 0, x = 0 then c = 0.

ap
∴ sin −1 ( x / a ) = (µ ) t
or x = a sin { (µ ) t} ...(iii)
Let t ′ be the time taken in moving from O to P, then from (iii),

l
b = a sin ( µ t ′ )
sC
 1  −1  b 
or t′ =   sin   . ...(iv)
 µ a
Also, time taken from O to A
es
1 1  2π  π
= (period) =  = .
4 4  µ 2 µ
∴ Time taken in moving from P to A
π
cc

= − t′
2 µ
∴ Required time = 2 × time taken in moving from P to A
 π   π 1 b 2 π b
=2 − t ′ = 2  − sin −1  = − sin −1 
Su


2 µ  2 µ µ a µ 2 a

Also, time period T =
µ
1 T
∴ = ...(v)
µ 2 π
π −1 b
Also, let − sin =θ ...(vi)
2 a
b π 
∴ = sin  − θ = cos θ
a 2 
[(a 2 − b 2 )] v
∴ tan θ = = from (ii)
b b µ
vT
= , from (vi)
2πb
SuccessClap: Best Coaching for UPSC Mathematics : For Info- 9346856874
Checkout ->22 Weeks Study Plan, Videos, Question Bank Solutions, Test Series

Simple Harmonic Motion and Elastic Strings 719


 vT 
θ = tan −1 
or  2πb 
1
Substituting the values of and θ.
µ
π −1 b 
i.e.,  − sin  in (v), we get
2 a
2T  vT  T  vT 
the required time = tan −1   = tan −1  .
2π  2πb  π  2πb 
6. If in a S.H.M. u, v, w be the velocities at distances a, b, c from a fixed point on the straight
line which is not the centre of the force; show that the period T is given by the equation
u 2 v 2 w2
4π 2
(b − c) (c − a) (a − b) = a b c .
T2 1 1 1

ap
Sol. Let O be the centre of force and O′ the fixed
point from which the distances of the points A, B and C
are a, b and c respectively, velocities at A, B, C are u, v O O A B C
and w. Let OO ′ = k . Then the distances of A, B and C

l
from O will be a + k , b + k and c + k respectively.
sC
We know that velocity V at a distance x from the centre of force is given by
V 2 = µ ( A2 − x 2 ),
where µ is the intensity and A the amplitude of S.H.M.
∴ at A we get
es
u 2 = µ { A2 − ( a + k ) 2 }
u2
or = A2 − a 2 − k 2 − 2ak
µ
cc

 u2 
or  + a 2  + 2ak + (k 2 − A2 ) = 0 ...(i)
 µ 
Similarly, at B and C, we get
 v2 
Su

 + b 2  + 2bk + (k 2 − A2 ) = 0 ...(ii)
 µ 
w 2 
and  + c 2  + 2ck + (k 2 − A2 ) = 0 ...(iii)
 µ 
Eliminating 2k and (k 2 − A2 ) from (i), (ii) and (iii), we get
u2
+ a2 a 1
µ
v2
+ b2 b 1 =0
µ
w2
+ c2 c 1
µ
u2 a 1 a2 a 1
1 2
or v b 1 + b2 b 1 =0
µ 2
w c 1 c2 c 1
SuccessClap: Best Coaching for UPSC Mathematics : For Info- 9346856874
Checkout ->22 Weeks Study Plan, Videos, Question Bank Solutions, Test Series

720 Dynamics

u2 a 1 a2 a 1
or v2 b 1 = − µ b2 b 1
w2 c 1 c2 c 1

u2 a 1
v2 b 1 = µ (b − c) (c − a) (a − b)
or
w2 c 1

2π 4π 2
Also, periodic time T = or µ=
µ T2
u2 v2 w2
4π 2
∴ a b c = 2 (b − c) (c − a) (a − b)
T

ap
1 1 1
7. A particle oscillates with S.H.M. of amplitude a and periodic time T. Find the expression of
the velocity v in terms of (i), a, T and x; (ii) a, T and t and also prove that
T 2 2π 2 a 2

l
∫ 0
v dt = sC
T
.

Sol. If µ be the intenstiy of S.H.M. we have


v 2 = µ (a 2 − x 2 ) ...(i)
x = a cos (µ ) t ...(ii)
es
and T = 2π / π ...(iii)
(i) From (iii), µ = 4π 2 / T 2
4π 2
v2 = (a 2 − x 2 ).
cc

∴ From (i) 2
T
(ii) From (i) and (ii)
v 2 = µ [ a 2 − a 2 cos 2 (µ ) t ]
Su

2 2
= µ a sin (µ ) t
2
4µ  2πt 
= a 2 sin 2 
T 2  T 
T 2
(iii) ∫
0
v dt

T 4π 2  2πt 
=∫ a 2 sin 2  dt
0 T 2  T 
2a 2 π 2 T  2πt 
=
T 2 ∫0 2 sin 2 
 T 
dt

2a 2 π 2 T 4πt 
=
T 2 ∫0 1 − cos T 
 dt
T
2a 2 π 2  T 4πt 
= 2 
t− sin 
T 4π T 0
SuccessClap: Best Coaching for UPSC Mathematics : For Info- 9346856874
Checkout ->22 Weeks Study Plan, Videos, Question Bank Solutions, Test Series

Simple Harmonic Motion and Elastic Strings 721


2a 2 π 2  T  
2 
= T− sin 4π − 0
T  4π  
2 2
2a π
= .
T
8. A small bead P can slide on a smooth wire AB, being acted upon by a force per unit mass
equal µ / CP 2 , where C is outside AB. Show that time of a small oscillation about its position of
equilibrium is (2π / µ ) b3 / 2 , where b is the perpendicular distance of C from AB.
Sol. Let O be the foot of the perpendicular from C on AB and let O be the origin. Let P be the
position of bead at time t and OP = x, CP = r.
O P
µ E A
Here, the force along PC = , hence
2
r
Component of force along PO
µ µx µ/r2
= 2 cos θ = 3 B

ap
r r
−3 / 2
µx µx  x2 
= 2 = 1 + 2 
(b + x 2 )3 / 2 b3  b 
µx  2 

l
3 x
= 3 1 − + ...
b 
sC 2 b 2

µx x
= 3 , neglecting higher powers of .
b b
Hence the equation of motion of bead will be
es
d2x µx
2
.=−
dt b3
This is the standard form of S.H.M. and its period
cc

2π 2π 3 / 2
= = b .
3
{ (µ / b )} µ
9. A particle P moves in a straight line OCP being attracted by a force mµ. PC, always
Su

directed towards C whilst C moves along OC with a constant acceleration f . If initially C was at
rest at the origin O, then P was at a distance c from O moving with velocity V, prove that the
distance of P from O at any time t is
 f  V f 1 2
 µ + c cos µ t + µ sin µ t − µ + 2 ft .

Sol. Intially, let O and C are coincident. Let after time t, CP = x. Since C is itself moving with
constant acceleration f, we consider the motion of P relative to C. For this purpose we impose the
constant acceleration f upon the whole system in the opposite direction. Let at any instant CP = x,
T T
x
O C P O C P

then the particle P will be attracted towards C with a force µx per unit of mass.
Eqation of motion of P relative to C is
d2x  f
= − µx − f = − µ  x −  ...(i)
dt 2  µ
SuccessClap: Best Coaching for UPSC Mathematics : For Info- 9346856874
Checkout ->22 Weeks Study Plan, Videos, Question Bank Solutions, Test Series

722 Dynamics
Solution to this differential equation will be
f
x − = A cos µ t + B sin µ t ...(ii)
µ
Differentiating, we get
dx
= − A µ sin µ t + B µ cos µ t ...(iii)
dt
dx
Given conditions are, at t = 0, x = c, = V . Hence we have from (ii) and (iii),
dt
f V
A=c− ,B=
µ µ
Therefore,
f  f V
x = +  c −  cos µ t + sin µ t ...(iv)
µ  µ µ
Equation (iv) gives the distance of P from the point C after time t, but in time t the point C itself

ap
1 2
will move a distance ft . Hence the distance of P from O at any instant is equal to
2
f 1 2  f V
+ ft +  c −  cos µ t + sin µ t.

l
µ 2  sC µ µ
EXERCISE
1. In a S.H.M., at what distance from the centre will the velocity be half of the maximum.

1
[Ans. ± a 3]
es
2
2. The speed v of a point P which moves in a straight line is given by the relation v 2 = a − bx 2 ,
where x is the distance of the point P from a fixed point on the path, a and b being con-
stants. Show that motion of P is simple harmonic and determine its amplitude and period.
cc

[Ans. (a / b), 2πb]


3. A particle moves in a straight line and its velocity at a distance x from the origin is
k [(a 2 − b 2 )], where k and a are constants. Prove that the motion is simple harmonic and
Su

find the amplitude and the periodic time of the motion.


 2π 
 Ans. a, k 
 
4. A particle is moving with S.H.M. and while making an excursion from one position of rest to
the other, its distance from the middle point of its path at three consecutive seconds are

observed to be x1 , x2 , x3 ; prove that the time of a complete oscillation is , where
θ
 x + x3 
θ = cos −1  1 .
 2 x2 
5. A body is attached to one end inextensible string and the other end moves in a vertical line
with S.H.M. of amplitude a, taking n complete oscillations per second, show that the string
will not remain tight during the motion unless n 2 < ( g / 4π 2 a).
6. Show that in a S.H.M. of amplitude a and period T, the velocity v at a distance, x from the
centre is given by the relation v 2T 2 = 4π 2 (a 2 − x 2 ).
SuccessClap: Best Coaching for UPSC Mathematics : For Info- 9346856874
Checkout ->22 Weeks Study Plan, Videos, Question Bank Solutions, Test Series

Simple Harmonic Motion and Elastic Strings 723


7. The speed v of a particle moving along the axis OX is given by the relation
v 2 = n 2 (8ax − x 2 − 12a 2 ). Prove that the motion is simple harmonic, with amplitude 2a
π  π
and that the time taken from x = 4a to x = 6a is . What is the periodic time.  Ans. 2n 
2n  
8. The speed v of the point P which moves in a line is given by the relation v 2 = a + 2bx − cx 2 ,
where x is the distance of the point P from a fixed point on the path and a, b, c are con-
stants. Show that the motion is simple harmonic if c is positive and determine the period.
 2π 
 Ans. 
 c
9. If time t be regarded as a function of velocity v, prove that the rate of decrease of accelera-
3 d 2t
tion is given by f , f being the acceleration.
dv 2

ap
10. In a S.H.M., if the velocities at distance b and c from the centre of force be respectively u
and v , then prove that the frequency n of oscillation is given by
4π 2 n 2 (b 2 − c 2 ) = v 2 − u 2 .
11. A particle moves with S.H.M. in a straight line. In the first second after starting from rest, it travels a
distance a and in the next second it travels a distance b in the same direction. Prove that the amplitude

l
of the motion is 2a 2 /(3a − b).
sC
12. A body moving in a straight line OAB with S.H.M. has zero velocity when at the points A and B
whose distances from O are a and b respectively, and has velocity v when half way between them.
Show that the complete period is π (b − a ) / v.
13. A particle of mass m is attached towards a fixed point O with a force (m / µ ) times the distance from
es
O. If initially it is projected towards O with a velocity v from a point distant c from O, find the
amplitude of its oscillations.
  2  
1/ 2
 Ans. v + c 2 
cc

   
 µ 
 

14. In a S.H.M. of period if the initial displacement be x0 and the initial velocity u0 , prove
Su

ω
that
(i) Amplitude = {x02 + (u02 / ω 2 )}
  1  u 
(ii) Position at time t = {x02 + (u02 + ω 2 )} cos t −   tan −1  0  
  ω   ωx0  
 1 −1  u0 
(iii) Time to the position at rest =   tan 
 ω  ω x0 
15. A particle of mass m is attached to a light wire which is stretched tightly between two fixed
points with a tension T. If a, b be the distances of the particle from the two ends, prove that
the period of small transverse oscillation of mass m is

{T (a + b) / mab}
SuccessClap: Best Coaching for UPSC Mathematics : For Info- 9346856874
Checkout ->22 Weeks Study Plan, Videos, Question Bank Solutions, Test Series

724 Dynamics

16. A particle starts from rest under an acceleration k 2 x directed towards a fixed point and
after time t another particle starts from the same position under the same acceleration. Show
 1 
that the particle will collide at time (π / k ) + t  after the start of the first particle provided
 2 
t < 2π / k .
17. Assuming that the gravity inside the earth varies as the distance from its centre, show that a
train, starting from rest and moving under gravity only, would take the same time to travors
smooth straight airless tunnel between any two points of the earth’s surface. Find the time.
1
[Ans. 42 minutes nearly]
2
3.2 ELASTIC STRINGS AND SPRINGS (HOOKE’S LAW)
According to Hooke’s Law,
stress ∝ strain or stress = λ (strain)
where λ is called modulus of elasticity.

ap
In case of elastic strings or springs the stress is the tension and strain is the increase in length
per unit legnth or the increase in natural length divided by the natural length.
Thus if a is natural length and l is the extended length then tension T of the string is given by
l−a

l
T =λ
sC .
a
3.2.1 A Horizontal Elastic String
One end of an elastic string whose modulus of elasticity is λ and whose natural length is a,
is tied to a fixed point on a smooth horizontal table, and the other end is tied to a mass m lying
es
on the table. The particle is pulled to a distance where the extension of the string becomes b, and
then let go. Discuss the motion and find the period of one complete oscillation.
Let O be the fixed point and OA the natural length i.e., OA = a. The particle is pulled out to a
point B and then released.
cc

Let P ( AP = x) be the position of the particle at time t, then by Hooke’s law, the tension T in
the string will be
Su

x m
B' A' O T A T P B

x
T =λ ...(i)
a
The equation of motion of m will be
d2x x
m 2
= −T = − λ
dt a

d2x λ
or 2
=− x ...(ii)
dt am
This show that the motion is simple harmonic so long as there is extension in the string, i.e., x
is not equal to zero. The period of the S.H.M. is
SuccessClap: Best Coaching for UPSC Mathematics : For Info- 9346856874
Checkout ->22 Weeks Study Plan, Videos, Question Bank Solutions, Test Series

Simple Harmonic Motion and Elastic Strings 725

2π  am  
= 2π    .
 λ    λ  
 
 am  

1  am   π  am  
Hence time of describing BA will be . 2π    =  
4  λ   2  λ 
dv λ
From (ii), we have v =− x
dx am
1 2 λ x2
Integrating, v =− +C
2 am 2
where C is constant of integration.

ap
λ 1 2
At B, x = b, v = 0, hence C = . b
am 2
λ
Therefore v2 = (b 2 − x 2 ) ...(iii)

l
sCam
Since the particle is moving from B towards O, i.e., x decreases as t increases, we get

dx  λ 
= −  (b 2 − x 2 )  ...(iv)
dt   am 
es
when the particle reaches the point A, x = 0 and hence from (i) and (iv) we have T = 0 and

 λ  
v = −    b. Hence the S.H.M. ceases and the particle moves towards O with constant velocity
 am  
cc

 λ  
   b till it reaches the point A′ on the other side of O where OA′ = OA = a.
 am  
Su

 λ  
Thus the particle moves from A to A′ with constant velocity    b and time taken to
 am  
describe this path

2a 2a  am  
= =  
 λ   b  b 
  b
 am  
As soon as the particle goes beyond A′, the string becomes again extended so that tension
comes into play and the motion is again simple harmonic till the particle reaches the point B′,
where AB = A′ B ′ = b. Here the velocity becomes zero and particle stops. The particle will then
retrace its path under S.H.M. till it will reach to A′ and then again under constant velocity
 λ  
   b till the point A and then again under S.H.M. till the point B where particle will stop.
 am  
SuccessClap: Best Coaching for UPSC Mathematics : For Info- 9346856874
Checkout ->22 Weeks Study Plan, Videos, Question Bank Solutions, Test Series

726 Dynamics
This motion will again be repeated.
Obviously, the motion from B to A, A′ to B′, B′ to A′ and A to B is simple harmonic and the

 λ  
total time for this motion is equal to 2π    .
  am  

 am  
The motion from A to A′ and A′ to A in under constant velocity    b and the total
 λ 

2a  am  
time for this motion is 2 .   .
b  b  
Thus the complete motion is oscillatory and the time for one complete oscillation is
 am   4a  am    am    2a 

ap
2π    +    = 2     π + .
 λ   b  λ    λ   b

EXAMPLES

l
1. A light elastic string of modulus λ is stretched to double its natural length and is tied to
sC
two fixed points distant 2a apart. A particle of mass m, tied to its middle point is displaced in the
line of the string through a distance equal to half its distance from the fixed point and released.
 am    λa  
Prove that the time of a small oscillation is π    and the maximum velocity is   
 λ    m 
es
whose λ is modulus of elasticity.
Sol. Let the string be tied to two fixed points A and A′ . Let O, the middle point of A A′ be the
origin. Let the particle tied at O is displaced to B, where OB = a / 2. Let P be the position of the
cc

particle m at time t and then tensions in the string be T1 and T2 along PA and PA′. Let OP = x.
T2 T1
A' B' O xP B A
Su

Then the equation of motion of P will be


d2x
m = T1 − T2
dt 2
a − x − a/2  2x 
Now, T1 = λ . = λ 1 + 
a/2  a
since a − x is extended length and a / 2 natural length of this portion. Similarly
a + x − a/2  2x 
T2 = λ = λ 1 + 
a/2  a
d2x  2x   2x  
Hence from (i) m = λ 1 −  − 1 + 
 a  a 
2
dt
d2x 4λ
or 2
=− x. ...(ii)
dt am
SuccessClap: Best Coaching for UPSC Mathematics : For Info- 9346856874
Checkout ->22 Weeks Study Plan, Videos, Question Bank Solutions, Test Series

Simple Harmonic Motion and Elastic Strings 727


It is clear from equation (ii) that the motion of the particle is simple harmonic and particle
oscillates between B and B′ and its periodic time is

 am    am  
2π    = π   .
  4λ    λ  
dv 4λ
Now, from (ii) v =− x
dx am
1 2 2λ
Integrating, v =− x2 + C,
2 am
4a
where C is the constant of integration. At B, x = a / 2, v = 0; therefore C = . Hence
m
4λ  a 2 
v2 =  − x2  ...(iii)
am  4 

ap
From (iii) it is clear that the velocity is maximum at x = 0 and then its maximum value will be
 λa  
vmax =    .
 m  
2. One end of elastic string whose modulus of elasticity is λ and whose natural length is a,

l
sC
is tied to a fixed point on a smooth horizontal table, and the other end is tied to a mass m lying
on the table. The particle is pulled to a distance where the extension of the string becomes b and
then let go. Describe the character of motion and show that the period of complete oscillation is

 2a   ma  
2 π +    .
es
 b   λ  
Sol. Let O be the fixed point on the table, to which one end of the string is tied. OA = a is the
natural length of the string. A particle of mass m is tied to the string at A. Let the particle is now
cc

pulled to B where AB = b and then released.


Let P is the position of the particle at any time t, where AP = x. When the particle is at P, the
extended length of the string = OP = a + x.
Su

T
B A O Ax P E

∴ Tension in the string


(a + x) − a λx
AO = λ = .
a a
The only force acting on the particle at P is the tension. Hence the equation of motion will be
d2x λx
m 2
=−
dt a
Negative sign indicates that the tension is in the direction of x decreasing.
d2x λ
Hence 2
x=− ...(i)
dt am
This represent S.H.M. whose time period
SuccessClap: Best Coaching for UPSC Mathematics : For Info- 9346856874
Checkout ->22 Weeks Study Plan, Videos, Question Bank Solutions, Test Series

728 Dynamics

 ma  
= 2π    ...(ii)
 λ  
Again from (i), we get
dv λ d2x dv
v =− x, as 2 = v
dx am dt dx
Integrating, we get
λ 2
v2 = − x + C,
ma
where C is the constant of integration. At B, x = b, v = 0
λ 2 λ 2
∴ 0=− b +C or C= b
ma ma

ap
Substituting the value of C, we get
λ
v2 = (b 2 − x 2 ) ...(iii)
ma
When the particle reaches A, then x = 0. Then its velocity,

l
sC
v = 
 λ  
 b
 ma  
At A the tension on the particle will be zero, which means that the particle is not influenced by
es
 λ  
any force. So it moves with velocity    b and will pass through O; and go to the left of O
 ma  
to the point A′. When it reaches A′ the string again becomes stretched and the tension comes
cc

into play and the motion once again becomes S.H.M. to B. The particle comes to instantaneous rest
at B′, moves back to A′ under S.H.M. From A to A′ the motion is under uniform velocity while
from A to B it is simple harmonic.
Hence, time of one complete oscillation
Su

= 4 [time form B to O] = 4 [time from B to A + time from A to O]


1 distance AO 
= 4  × periodic time + 
 4 uniform velocity 

 
 
1  ma   a 
= 4 ×  × 2π    + 
4  λ    λ   
   b
  ma   

π  ma  a  ma  
=4   +  
 2 λ  b λ  

 2a   ma  
= 2  π +    .
 b λ  

SuccessClap: Best Coaching for UPSC Mathematics : For Info- 9346856874
Checkout ->22 Weeks Study Plan, Videos, Question Bank Solutions, Test Series

Simple Harmonic Motion and Elastic Strings 729


3. A particle is performing S.H.M. in the line joining two points A and B on a smooth plane
and is connected with these points by elastic strings of natural lengths a and a′ the moduli of
elasticity being λ and λ′ respectively. Show that the periodic time is

 m 
2π   
 (λ / a ) + (λ′ / a ′ )  
Sol. Let OA and OB be the elastic strings whose natural lengths x
are a and a′ respectively. Let O be the position of equilibrium of A O P B
mass m, λ and λ′ being the moduli of elasticity of the strings OA and OB respectively.
Then in the position of equilibrium tension in the string OA = tension in the string OB
λ λ′
i.e., l= l′ ...(i)
a a′
where l and l′ are the extensions in OA and OB.

ap
Let at time t , P be the displaced position of the particle of mass m, such that OP = x. Then
 λ′ 
tension in the string PB =   (l ′ − x) acting in the direction PB and tension in the string
 a′ 
PA = (λ / a) (l + x), acting in the direction PA. Hence the equation of motion of the particle will be

l
d2x  λ′   λ  λ ′l ′ λl   λ ′ λ 
m
dt
sC
=   (l ′ − x) −   (l + x) = 
 a′ 
2  a  a′
−  − +  x
a   a′ a 
  λ λ′  
d2x   a + a ′  
or   from (i)
=−  x,
dt 2 m
es
 
 
This is the standard form of S.H.M. Hence the required time period
 
cc

 
2π  m 
= = 2π  
 λ λ′     λ + λ′  
 +     a a ′  
 a a′ 
Su

m
EXERCISES
1. A particle of mass m executes simple harmonic motion in the line joining the points A and B
on a smooth table and is connected with these points by elastic strings whose tensions in
 mll ′  
equilibrium are each T. Show that the time of an oscillation is 2π    , where
 T (l + l ′ )  
l , l′ are the extensions of the string beyond their natural lengths.
2. An elastic string of natural length (a + b) where a > b modulus of elasticity λ has a par-
ticle of mass m attached to it at a distance a from one end, which is fixed to a point A of a
smooth horizontal plane. The other end of the string is fixed to a point B so that the string is
just unstretched. If the particle be held at B and then released, show that it will oscillate to
and fro through a distance b ( a + b ) / a in a periodic time π ( a + b ) [(m / λ )].
3. Two light elastic strings are fastened to a particle of mass m and their other ends to fixed
points so that the strings are taut. The modulus of each is λ, the tension T and lengths a
and b. Show that the period of one oscillation along the line of the string is
SuccessClap: Best Coaching for UPSC Mathematics : For Info- 9346856874
Checkout ->22 Weeks Study Plan, Videos, Question Bank Solutions, Test Series

730 Dynamics

 mab 
2π   
 (T + b) (a + b)  
4. Prove that the work done against the tension in stretching a light elastic string is equal to
the product of its extension and the mean of the initial and final tension.
5. Two particles of masses m1 and m2 are tied to the end of an elastic string of natural length
a and modulus λ. They are placed on a smooth table so that the string is just taut and m2
is projected with any velocity directly away from m1. Prove that the string will become slack
after the lapse of time
 am1m2 
π 
 λ ( m1 + m2 ) 
6. Two masses m1 and m2 are connected by a spring of such a strength that when m1 is held
fixed m2 performs n complete vibrations per second. Show that if m2 be held fixed, m1 will

ap
m 
make n  2  , and if both be free, they will make n {(m1 + m2 ) / m1} vibrations per sec-
 m1 
ond, the vibrations in each case being in the line of spring.

l
3.3 VERTICAL ELASTIC STRING
sC
A light elastic string of natural length a and modulus of elasticity λ is O O O
suspended by one end, to the other end is tied a particle of mass m, the
particle is slightly pulled down and released. Discuss the motion. a a a
Let O be the fixed point at which upper end of the string is tied. Let
es
OA = a be the natural length of the string. When weight mg is tied at the
A A A
lower end A then let b the extension in the string in equilibrium, where AB = b.
b T0 b
Let T0 be the tension in this case. Then
B B
b mg x T
cc

T0 = mg = λ ...(i)
a P
Now, the particle is pulled vertically down wards to a point C ( BC = c) C
mg
and then released. Since the tension in the string when particle is at C is greater
Su

than the weight of the particle; hence it will move upwards.


Let P ( BP = x) be the position of the particle after time t and then let T be the tension in the
string, then by Hooke’s Law,
b+ x x
T =λ. = mg + λ ...(ii)
a a
[from (i)]
and equation of motion will be
d2x
m 2 = mg − T
dt
x
= mg − mg − λ [from (ii)]
a
d2x λ
or 2
=− x
dt am
g
or = − x, [from (i)] ...(iii)
b
dv g
v =− x
dx b
SuccessClap: Best Coaching for UPSC Mathematics : For Info- 9346856874
Checkout ->22 Weeks Study Plan, Videos, Question Bank Solutions, Test Series

Simple Harmonic Motion and Elastic Strings 731


Integrating, we get
v2 g x2
=− + A,
2 2 2
where A is constant of integration.
At C, x = c, v = 0, hence
g c2
A=
b 2
g
∴ v = (c 2 − x 2 )
2
...(iv)
b
The motion given by (iii) is simple harmonic, having B, the position of equilibrium as the centre
of oscillation. Now, let us consider the following two cases :
Case I. If c ≤ b, the motion given by (iii) is purely simple harmonic, centre of oscillation B,
with amplitude c and period
2π [(b / g )].

ap
Case II. If c > b but less than [(b 2 + 4ab)], the particle in its upwards motion goes above
A, but at A the string becomes slack and tension becomes zero. Hence S.H.M. ceases and the particle
rises against gravity till the velocity becomes zero.

l
sC  g 
From (iv) the velocity at A is u =   (c 2 − b 2 )  in the upward direction and the particle
 b 

u 2 c 2 − b2
rises up to the height = above O.
es
2g 2b
This will be true if the height risen above O is not greater than a, the natural length of the
string, otherwise the motion again becomes S.H.M. Condition that motion above O may not be
S.H.M. is
cc

c2 − b2
< 2a, i.e., c < [(b 2 + 4ab)]
2b
Remarks : (i) In the case of a spring the law of compression is same as law of extension. Thus
the tension operates even when particle rises above A. Hence in this case equation (ii) holds good
Su

through out the motion. The period of motion will be 2π [(b / g )].
(ii) While solving problems on vertical elastic strings the position of equilibrium must be obtained
first.
EXAMPLES
1. An elastic string without weight, of which the unstretched length is l and modulus of
elasticity is the weight of n oz, is suspended by one end and a mass of m oz, is attached to the
other end. Show that the time of a small vertical oscillation is
  ml  
2π     .
  ng  
Sol. Let O be the fixed point, l natural length and b and extension when m oz mass is attached
to the string. The string is then pulled down to a small distance and then released. Let when weight
is at P at time t, then extension in the string is b + x. Then we have
b
T0 = mg = λ
l
SuccessClap: Best Coaching for UPSC Mathematics : For Info- 9346856874
Checkout ->22 Weeks Study Plan, Videos, Question Bank Solutions, Test Series

732 Dynamics

b
or mg = ng (since λ = ng ) ...(i)
l
Equation of motion will be O O
2
d x
m = mg − T ...(ii)
dt 2
b+x ng A A
But T = ng . = mg + x, [from (i)] b T0 b
l l B B
Hence from (ii) x T
P
d2x ng
m 2
= mg − mg − x
dt l

d2x ng
=−
x.

ap
or 2
dt ml
Hence the motion is S.H.M. and the period of motion

2π   ml  
= = 2π     .

l
[( ng / ml )]   ng  
sC
2. A light elastic string of natural length l has one end fixed at a point A; and the A
other attached to a stone, the weight of which in equilibrium would extend the string to
B
a length l1. Show that if the stone be dropped from rest at A, it will come to an
es
C
instantaneous rest at a depth [(l12 − l 2 )] below the equilibrium position.
P
Sol. AB = l is the natural length of the string. Let m be the mass of the stone and C
its position of equilibrium. According to given condition
cc

AC = l1 or BC = (l1 − l )
D
 λ
Then at C, mg =   (l1 − l ) ...(i)
l
Su

where λ is the modulus of elasticity.


Let the stone be dropped from rest at A, the motion from A to B will be due to gravity only as
there will be no tension. If v be the velocity of the particle when it reaches B, then
v = [(2 gl )] ...(ii)
At B, the string becomes taut and for the motion below B, the tension of the string comes into
play. Let P be any displaced position of the stone. Let CP = x. Then at P the forces acting on the
 λ
stone being (i) its weight mg acting vertically downwards and (ii) tension   (l1 − l + x) in the
l
string acting vertically upwards. Hence the equation of motion will be
 d2x λ
m  2  = mg − (l1 − l + x)
 dt  l

 λ
= mg − mg −   x, from (i)
l
SuccessClap: Best Coaching for UPSC Mathematics : For Info- 9346856874
Checkout ->22 Weeks Study Plan, Videos, Question Bank Solutions, Test Series

Simple Harmonic Motion and Elastic Strings 733

d2x λ gx
or 2
=− .x =− , from (i)
dt ml (l1 − x)
dv gx
or v =−
dx (l1 − l )
Integrating,
v2 g x2
=− + A.
2 (l1 − l ) 2
At B, v = [(2 gl )] and x = − (l1 − l )
g
∴ 2 gl = − (l1 − l ) 2 + A
(l1 − l )
or A = 2 gl + g (l1 − l )
 g  2

ap
v 2 = 2 gl + g (l1 − l ) −  x
 l2 − l 
∴ ...(iii)
If the particle comes to instantaneous rest at D, such that CD = d , then from (iii), we get
gd 2

l
0 = 2 gl + g (l1 − l ) −
(l1 − l )
or
sC
d 2 = (l12 − l 2 ) or d = [(l12 − l 2 )].
3. A smooth light pulley is suspended from a fixed point by
a spring of natural length l and modulus of elasticity ng. If O 2T
es
masses m1 and m2 hang at the ends of a light inextensible string
passing round the pulley, show that the pulley executes simple
harmonic motion about a centre whose depth below the point A A
f
of suspension is l (1 + 2M / n), where M is the harmonic mean
T
cc

between m1 and m2 . T
T
Sol. Let O be the fixed point which the pulley is suspended. f
Let λ be the modulus of elasticity, then m2g
λ = ng ...(i) m1g
Su

Let us consider the motion of masses m1 and m2 . Let f be m2g m1 g


the acceleration of the system and T the tension in the string
round the pulley. Then the equation of motion will be
m1 g − T = m1 f
and T − m2 g = m2 f
which give
2m1m2
T= g = Mg , where
(m1 + m2 )
2m1m2
M = .
m1 + m2
Hence pressure on the pulley due to the masses m1 and m2 = 2T = 2Mg.
Now, the problem reduces to the motion of a mass 2M hanging from one end of an elastic
spring of natural length l whose other end is fixe at O.
Let B be its position of equilibrium such that AB = d (say),
Then at B,
weight of the mass 2M = Tension in the spring
i.e., 2Mg = (λ / l ) d = (ng / l ) d from (i)
SuccessClap: Best Coaching for UPSC Mathematics : For Info- 9346856874
Checkout ->22 Weeks Study Plan, Videos, Question Bank Solutions, Test Series

734 Dynamics
2lM
or d=
n
In case of vertical spring whose one end is fixed we know that the motion is simple harmonic
about the posiion of equilibrium of the mass attached to the other end.
Hence required depth = OB = OA + AB
2lM   2M  
=l+d =l+ = l 1 +   .
n   n 
EXERCISES
1. A light elastic string of natural length l is hung by one end and to the other end are
tied successively particles of masses m1 and m2 . If t1 and t2 be the periods and c1 , c2
the statical extensions corresponding to these two weights, prove that
g (t12 − t22 ) = 4π 2 (c1 − c2 ).
2. A heavy particle is attached to one end of an elastic string, the other end of which is fixed.
The modulus of elasticity of the string is equal to the weight of the particle. The string is

ap
drawn vertically down till it is four times its natural length and then let go. Show that the
 a  4 
particle will return to this point in time  g   3 π + 2 3  where a is the natural length of

l
the string. sC
3. A heavy particle attached to a fixed point by an elastic string hangs freely stretching the
s t r i n g b y e. It is drawn by an additional distance f and then let go. Determine the
a q u a n t i t y

height to which it will rise if f 2 − e2 = 4ae; e being unstretched length of the string.
[Ans. f + e + 2a ]
es
4. A mass m hangs from a fixed point by a light spring and is given small vertical displacement.
Show that the motion is simple harmonic. If l is length of the spring when the system is in
equilibrium and n the number of oscillations per second, show that the natural length of the
spring is l − ( g / 4π 2 n 2 ).
cc

5. A light elastic string whose natural length is a has one end fixed to a point O and to the
other end is attached a wieght which in equilibrium would produce an extension e. Show
that if the weight be let fall from rest at O, it will come to stay instantaneously at a point
distant (2ae + e2 ) below the position of equilibrium.
Su

6. A heavy particle of mass m is attached to one end of an elastic string of natural length l
whose other end is fixed at O. The particle is then let fall from rest at O. Show that, part of
the motion is simple harmonic, and that if the greatest depth of the particle below O is
1
l cot 2 (θ / 2), the modulus of elasticity of the string is mg tan 2 θ and that the particle
2
attains this depth in time (2l / g ) [1 + (π − θ) cot θ], where θ is a positive acute angle.
7. One and of a light elastic string of natural length a and modulus 2mg is attached to a point
O and the other end to a particle of mass m. The particle initially held at rest at O, is al-
lowed to fall. Find the greatest extension of the string and show that the particle will reach
O again after a time t equal to
 2a 
(π + 2 − tan −1 2)  
 g
8. A heavy particle of mass m is attached to one end of an elastic string of natural length l
feet, whose modulus of elasticity is equal to the weight of the particle and the other end is
fixed at O, the particle is let fall from rest at O. Show that a part of the motion is simple
harmonic and that the greatest depth of the particle below O is (2 + 3) l feet. show that
this depth is attained in time
SuccessClap: Best Coaching for UPSC Mathematics : For Info- 9346856874
Checkout ->22 Weeks Study Plan, Videos, Question Bank Solutions, Test Series

Simple Harmonic Motion and Elastic Strings 735

(l / g ) { 2 + π − cos −1 (1/ 3)} seconds.


9. A particle of mass m is attached to one end of an elastic string of natural length a and modulus
of elasticity 2 mg, whose other end is fixed at O. The particle is let fall from A, when A is
vertically above O and OA = a. Show that its velocity will be zero at B, where OB = 3a;
calculate also time from A to B.
 1  a  
 Ans. −1  1  
   4 2 + π + 2 sin    
 2  2g    3  

10. A heavy particle is attached to an inextensible string to a fixed point from which the particle
is allowed to fall freely. When the particle is in its lowest position the string is of twice its
natural length. Prove that the modulus is four times the weight of the particle and find the
time during which the string is extended beyond its natural length.
  a  
 Ans.
1 −1  1  
 g   π − 2 cos  3   
 2   

ap
11. The bodies of mases M and M ′ are attached to the lower end of an elastic string whose
upper end is fiexd and hang at rest, M ′ falls off; show that distance of M from the upper

l
end of string at time t is a + b + c cos { ( g / b) t}, where a is the unstretched length of the
sC
string, b and c the distances by which it would be stretched when supporting M and M ′
respectively.
12. A heavy particle is attached to one end of a fine elastic string, the other end of which is
fixed. The unstretched length of the string is a and its modulus of elasticity is n times the
weight of the particle, is pulled vertically downwards till the length of the string is a′ and
es
is then let go from rest. Show that the time it returns to this position is
 a
2 (π − θ + θ ′ + tan θ − tan θ ′ )  
 ng 
cc

where θ and θ′ are positive acute angles given by


na ′
sec θ = − n − 1, sec2 θ ′ = sec2 θ − 4n.
a
13. A heavy particle is attached to one point of a uniform light elastic string. The ends of the
Su

string are attached to two points in a vertical line. Show that the period of a vertical oscilla-
tion in which the string remains taut is 2π (mh / 2λ ), where λ is the coefficient of elastic-
ity of the string and h the harmonic mean of the unstretched lengths of the two parts of the
string.
14. Two particles of masses M and 2M are connected by an inextensible string passing over a
smooth peg. From the particle M another equal particle hangs by an elastic string of natrual
length a and modulus Mg. The system is initially supported with the string vertical, the first
being taut and the second at its natural length and then released. Show that the motion is
S.H.M. with period π (3a / g ) and the extension of the second string at time t is
a [1 − cos {2t ( g / 3a )}]
❑❑❑
SuccessClap: Best Coaching for UPSC Mathematics : For Info- 9346856874
Checkout ->22 Weeks Study Plan, Videos, Question Bank Solutions, Test Series

Motion on Smooth and Rough Plane


4.1 MOTION ON A SMOOTH PLANE CURVE
A particle is compelled to move on a smooth plane curve under the action of given forces in
the plane, to find the motion.
Let P be the position of a particle of mass m at time t Y Y
and let the arc AP be s, where A is some fixed point on the
curve. Let the components of the force acting at P be X X cosψ

ap
and Y parallel to the axes OX and OY respectively. Let R Y cosψ
be the reaction at P. Let the tangent at P makes an angle
ψ with the x-axis. R Y cos
ψ
In the problems of this type one should use tangential ψ ψ

l
X
and normal accelerations. The impressed forces must be
sC P xs
in ψ
resolved along the tangent and normal and equated to the
effective forces in those directions. Hence, the effective
s
2 2 A
d s v ψ
forces are m and m . The impressed forces are
2 ρ
es
O T X
dt
X, Y and normal reaction R.
The equations of motion of the particle along the tangent and normal to the curve at P are
cc

d 2s
m = X cos ψ + Y sin ψ ...(i)
dt 2
v2
m = R − X sin ψ − Y cos ψ
Su

and ...(ii)
ρ
where ρ is the radius of curvature of the curve at P. These two equations determine the motion
of the particle.
4.2 MOTION ON A SMOOTH PLANE CURVE UNDER GRAVITY
To determine the motion of a particle on a smooth vertical plane curve under gravity.
(i) When the particle is moving outside the restraining
ψ
curve, so that the normal reaction offered by the curve is away
from the centre of curvature of the curve. Let ψ be the angle
R
which the tangent to the curve at any point makes with the
horizontal line. The impressed forces are the weight of the
particle and the reaction of the curve and they must be equal
ψ
d 2s v2 mv2
to the effective forces m along the tangent and m ρ
dt 2 ρ mg
along the normal, if s is measured from the highest point of the
2
curve. Hence, we have, the tangential effective force m d 2s
dt
SuccessClap: Best Coaching for UPSC Mathematics : For Info- 9346856874
Checkout ->22 Weeks Study Plan, Videos, Question Bank Solutions, Test Series

Motion on Smooth and Rough Plane 737

d 2s
m = mg sin ψ ...(i)
dt 2
The normal effective force
mv 2
= mg cos ψ − R ...(ii)
ρ
(ii) When the particle is moving inside the curve, so that R 2
m ds 2
mv 2 dt
is in the same direction . If s is measured from the lowest
ρ
point on curve in this case, then we have R mv2
d 2s ρ
m = − mg sin ψ ...(iii)
dt 2 P
mv 2
and = R − mg cos ψ ...(iv)

ap
ρ
Equations (i) and (ii) or (iii) and (iv) are quite sufficient to mg
determine the motion, as the case may be. ψ

EXAMPLES

l
sC
1. A particle slides on the curve x = 2 {a ( y − a)} with a velocity due to a fall from the
horizontal x-axis, the y-axis being vertically downwards. Find the pressure on the curve at any
time and time of sliding from y = b to y = c.
Solution. The equations of motion will be ψ
es
d 2s R
m = mg sin ψ ...(i)
dt 2
mv 2
and = mg cos ψ − R ...(ii)
cc

ρ mv2
ψ

Since the velocity of the particle is due to the fall from ρ


the horizontal x-axis, we have mg

v = ( 2 gy) 2
m d 2s
Su

To find the pressure R from (ii) let us find out ρ and ψ. dt


Now from the equation of the curve; so have
x 2 = 4 a ( y − a)
dy x d2y 1
∴ = and =
dx 2a 2 2a
dx
R| F dy I 2 U| 3/ 2
F 1 + x 2 I 3/ 2
S|1 + GH dx JK V| GH 4a2 JK (4ay)3/2 2 y 3/2
ρ= T W
Now, = = =
2 2
d y / dx 1 / 2a 4a 2 a1/ 2
dy x
Also, = tan ψ =
dx 2a
FG a IJ
∴ cos ψ =
H yK
Thus, from (ii), we have
SuccessClap: Best Coaching for UPSC Mathematics : For Info- 9346856874
Checkout ->22 Weeks Study Plan, Videos, Question Bank Solutions, Test Series

738 Dynamics

mv 2 FG a IJ − m.2gy. a1/2
R = mg cos ψ −
ρ
= mg
H y K 2 y3/2
FG a IJ − mg FG a IJ = 0
= mg
H yK H yK
Hence, the pressure at any point is zero.
To find the time of sliding from y = b to y = c, let us find a relation between y and t.
dy dy ds dy
Now, = . = (2 gy)
dt ds dt ds
= (2 gy) sin ψ = ( 2 gy) (1 − cos 2 ψ )
FG1 − a IJ =
= (2 gy)
H yK {2 g ( y − a)}

dy
(2g) dt =

ap
or ( y − a)
To get the required time, let us integrate between y = b to y = c. This gives

z
T
0 z
(2 g) dt = bc
dy

l
( y − a)
FG 2 IJ [
sC c
or ( 2 g) T = 2 y−a
b
or T=
H gK (c − a) − ( b − c)]

2. A wire, in the form of the parabola y 2 = 4 ax, is fixed with its axis vertical and vertex
es
downwards. If a small smooth bead of mass m can slides on the wire and is released from rest at
one end of the latus rectum, find its acceleration along the tangent when it is at a point x above

FG a IJ 3/2 .
the vertex and show that the pressure on the wire is 2 mg
H a + xK
cc

Solution. The equation of the parabola is


y 2 = 4 ax ...(i)
dy 2 a FG a IJ
Su

∴ tan ψ =
dx
=
y
=
H xK
LM F dy I 2 OP 3/2 x

1+ G J
M
N H dx K PQ
Hence, ρ=
d2y
dx 2 R
(4ay + 4a 2 ) 3/ 2 P
or ρ=
4a 2
Applying the equation of energy, the velocity at P ( x, y)
is given by y
O
1 ψ
mv 2 = mg ( a − x )
2
Whence v 2 = 2 g ( a − x ), since m ≠ 0, we also have mg
SuccessClap: Best Coaching for UPSC Mathematics : For Info- 9346856874
Checkout ->22 Weeks Study Plan, Videos, Question Bank Solutions, Test Series

Motion on Smooth and Rough Plane 739


mv 2
= R − mg sin ψ
ρ
mv 2 R| U| FG IJ 3/ 2
Hence, R= + mg sin ψ = mg S|
a (a − x)
+
a
V|
= 2 mg
a
H K
ρ T
( x + a) 3 / 2
( x + a) 3 / 2
Wa+x
d 2s FG a IJ 1/2
Also,
dt 2
= − g sin ψ = − g
H a + xK
3. A particle slides down the smooth curve
ψ
x
y = a sinh , the axis of x being horizontal and aixs of y
a R
downwards, starting from rest at the point where the tangent A s
is inclined at an angle α to the horizontal. Show that it
will leave the curve when it has fallen through a vertical P ψ
distance a sec α.

ap
Solution. The equation of motion are
mg
dv
mg = mg sin ψ ...(i)
ds

l
mv 2
= mg cos ψ − R
and
ρ
sC ...(ii)

dv dy FG∵ dy = sin ψIJ


From (i), v
ds
= g.
ds H ds K
∴ v dv = g dy
es
Integrating, v 2 = 2 gy + A,
where A is the constant of integration.
Initially, when y = y0 , say v = 0,
cc

∴ A = − 2 gy0
∴ v 2 = 2 g ( y − y0 ) ...(iii)

Putting for v 2 in equation (ii), we get


Su

2 mg ( y − y0 )
= mg cos ψ − R
ρ
when particle leaves the curve, then R = 0.
∴ m.2 g ( y − y0 ) = mg cos ψ ρ ...(iv)
x
Now, y = a sin h
a
dy x
∴ = cos h = tan ψ
dx a
FG1 + sinh x IJ = FG1 + y IJ
2
2
or tan ψ =
H aK H xK 2

Initially, when ψ = α, y = y0
F1 + y I
2
∴ tan α = GH a JK
0
2 ...(v)
SuccessClap: Best Coaching for UPSC Mathematics : For Info- 9346856874
Checkout ->22 Weeks Study Plan, Videos, Question Bank Solutions, Test Series

740 Dynamics
R| F dy I 2 U| 3/ 2

S|1 + GH dx JK V|
ρ cos ψ = T W 1
Now, .
d y 2
FG dy IJ 2

dx 2 1+
H dx K
R| F dy I U| F
V| GH1 + cos h x IJK
2
S|1 + GH dx JK 2

=T W=a a
2
d2y sin h
x
2 a
dx
y2 F I
1 + 1 + sin h
x 2
a = a2
2+ 2
a 2a 2 + y 2
GH JK
or ρ cos ψ = a =
x y y
sin h

ap
a
Hence, from (iv), we get
2a 2 + y 2
2 ( y − y0 ) = or 2 y 2 − 2 yy0 = 2 a 2 + y 2
y

l
or y 2 − 2 yy0 + y02 = 2 a 2 + y02
sC
or ( y − y0 ) 2 = 2 a 2 + a 2 (tan 2 α − 1) = a 2 (1 + tan 2 α ) by (v)
or ( y − y0 ) 2 = a 2 sec 2 α
∴ y − y0 = a sec α
es
Hence, the particle wil leave when it has fallen through a vettical distance a sec α.
4. A small bead, of mass m, moves on a smooth circular wire, being acted upon by a central

attraction to a point within the circle situated at a distance b from the centre. Show
( distance) 2
that in order that the bead may more completely round the circle, its velocity of projection at the
cc

R| 4µb U| .
point of the wire nearest the centre of force must not be less than S|(a − b ) V|
T 2 2
W
Solution. Let O be the centre of attraction. Then OA = a − b , OB = a + b. Let the veloicity at
Su

A be V and that at P be v.

The Central attraction is 2 .
r
The equation of motion will be
dv
=−

cos φ = −
mµ dr
∵ cos φ =
dr LM OP
mv
ds r 2
r 2
.
ds ds N Q
µ
or vdv = − dr
r2 mµ
cos φ
Integrating, we get z z
v dv = −
µ
r2
dr
r2
f
P
r2
/
1 2 µ mµ v
or v = +A ...(i) O
a
2 r A B
when the particle is at A, where r = a − b, the velocity is V. a– b C

1 2 µ
∴ V = +A ...(ii)
2 a−b
Subtracting (i) and (ii), we get
SuccessClap: Best Coaching for UPSC Mathematics : For Info- 9346856874
Checkout ->22 Weeks Study Plan, Videos, Question Bank Solutions, Test Series

Motion on Smooth and Rough Plane 741


1 2 1 2 µ µ
v − V = − ...(iii)
2 2 r a− b
In order that the particle may move completely round the circle its velocity v should not vanish
till the particle has reached the other end of the diameter, i.e., B where r = a + b. Hence, the least
velocity of projection will be obtained by putting v = 0 and r = a + b in (iii).
1 µ µ 2 µb
∴ 0− V2 = − =−
2 a+b a−b a − b2
2

4µb F 4µb I
∴ V2 =
a2 − b2
or
2
V=
2 GH a − b JK
5. A particle is projected from the vertex of a smooth parabolic tube of latus rectum 4a
mgr
along the tube and is acted upon by a repulsive force from the focus. If the velocity of
c
projection is that which would be acquired in moving from focus to the vertex, prove that the time
c FG IJ π+θ FG IJ

ap
of describing angle θ about the focus is 2
g H K
log tan
4
.
H K
Solution. The polar equation of a parabola of latus rectum 4a referred to focus as pole is

l
sC P
φ
/c
gr
m

θ
S a A
es

2a
= 1 + cos θ
cc

r
2a θ
r= = a sec 2
or θ 2
2 cos 2
2
Su

Let V be the velocity from focus to vertex, i.e., from r = 0 to r = a under the given force.
Then
LM v OP
2
V
=
g r2LM OP r

MN 2 PQ 0
c 2 MN PQ 0
g
or V 2 = a2 ...(i)
c
This will be the velocity of projection.
Tangential equation of motion will be
dv r r dr g
mv = mg cos φ = mg or v dv = r dr
ds c c ds c
Integrating, we get
LM v OP
2
v
=
g r2LM OP r

MN 2 PQ V
c 2 MN PQ 0
SuccessClap: Best Coaching for UPSC Mathematics : For Info- 9346856874
Checkout ->22 Weeks Study Plan, Videos, Question Bank Solutions, Test Series

742 Dynamics

v2 V 2 g r 2 a2 F
gr 2 V 2 I
∴ 2

2
=
c 2

2
= GH
2c

2
JK by (i)

v2 =
g 2 ds FG g IJ 2

c
r or v=
dt
=
H cK
ds dθ FG g IJ a sec2 θ
or .
dθ dt
=
H cK 2
...(ii)

θ dr θ θ θ1
Now r = a sec 2 ; ∴ = 2 a sec sec tan
2 dθ 2 2 22
ds
= r2 +
dr FG IJ 2
= a sec 2
θ FG1 + tan θ IJ
2
∴ dθ dθ H K 2 H 2K
θ
= a sec 3

ap
2
Hence, from (ii), we get
θ dθ
a sec 3 .
FG g IJ a sec 2 θ FG c IJ zθ sec θ dθ = z dt r
2 dt
=
H cK 2
or
H gK 0
2 0

FG c IJ LM2 log tan FG π + θ IJ OP F cI

l
θ
π+θ
2 G J log tan
H 4 4K Q
or
H gK N
sC 0
=T or
H gK 4
=T

6. A heavy ring of mass m is free to move on a smooth fixed parabolic wire of latus rectum 4a
whose axis is vertical and vertex upwards and is attached to one end of an elastic string of natural
length a and modulus of elasticity 2 mg whose other end is fixed at the focus. The ring is projected
es
from the vertex of the parabola with velocity 2 ( ga). Prove that that when the focal distances is

RS
r, its velocity is 2 g
r
(3a − r )
UV1/2 and the pressure on the wire is
T a W
cc

1 A N
(3r − 4a) mg
( ar ) S T φφ
Solution. Taking SX as initial line and S on pole, the θ P
Su

equation of the parabola will be


R
2a
= 1 + cos ( π − θ)
r
θ
= 1 − cos θ = 2 sin 2
2
At A where θ = π,
2a π
= 2 sin 2 = 2 X mg
r 2
∴ r=a
In a parabola tangent at any point bisects the angle between the focal radius and perpendicular
from the point on the directrix.
Also, ∠PSX = ∠SPN (alternate angles)
θ
∴ θ = φ + φ = 2φ or φ =
2
θ a FG IJ
or sin φ = sin =
2 r H K
SuccessClap: Best Coaching for UPSC Mathematics : For Info- 9346856874
Checkout ->22 Weeks Study Plan, Videos, Question Bank Solutions, Test Series

Motion on Smooth and Rough Plane 743


where φ is the angle between tangent and radius vector.
Also, pedal equation of parabola is
p 2 = ar i. e., p = a . r
dp a
∴ =
dr 2 r
dr 2 r r FG IJ
∴ ρ=r
dr
=r.
a
= 2r
a H K
Also tension in an elastic string
λ (Extension) ( r − a)
= = 2mg
Natural length a
The equation of motion will be
dv
mv = mg cos φ − T cos φ
ds
RS ( r − a) dr UV

ap
= mg − 2 mg
T a ds W ...(i)

g
or v dv = (3a − 2r ) dr
a

l
2g

sC
v2 =
a
(3ar − r 2 ) + A

At A, r = a, then v 2 = 4 ag, given


2 gt
4 ag = . 2 a 2 + A, ∴ A=0
es
a
2g
Hence, v2 = (3ar − r 2 ) ...(ii)
a
Again the normal equation of motion will be
v2
cc

m = R − mg sin φ + T sin φ
ρ
Putting the values of v 2 ρ, T and sin φ, we get
2 gr 1 FG a IJ − RSmg + 3mg (r − a) UV FG a IJ
Su

R = m.
a
(3a − r ) .
2r H rK T a W H rK
mg FG a IJ [3a − r − a − 2r + 2a] = mg
=
a H rK ar
(4 a − 3r )

EXERCISES
1. A particle slides down a catenary, whose plane is vertical and vertex upwards, the velocity
at any point being due to the fall from the directrix. Prove that the pressure at any point
varies inversely as the distance of that point from the directrix.
2. A small bead is projected with any velocity along a smooth circular wire under the action of
a force varying inversely as the fifth power of the distance from a centre of force situated on
the circumference. Prove that the pressure on the wire is constant.
3. A particle of mass m moves in a smooth circular tube of radius a under the action of a force
equal to mµ × distance to a point inside the tube at a distance from its centre. If the particle
be placed very nearly at its greatest distance from the centre of force, show that it will de-
scribe the quadrant ending at its least distance in time
FG a IJ log (
H µc K 2 + 1)
SuccessClap: Best Coaching for UPSC Mathematics : For Info- 9346856874
Checkout ->22 Weeks Study Plan, Videos, Question Bank Solutions, Test Series

744 Dynamics
4. A particle is projected horizontally from the lowest point of smooth elliptic arc whose major
axis 2a is vertical and moves under gravity along the concave side. Prove that it will leave
the curve at same point if the velocity of projection lies between ( 2 ga) and

[ga (5 − e 2 )] ; and if the velocity have the later value, prove that the particle will continue
to move round the ellipse in time

FG a IJ z R| 1 − e2 cos 2 φ U|1/2 dφ
2
H gK 0
n
S| 3 − e2 + 2 cos φ V|
T W
5. From the lowest point of a smooth hollow cylinder whose crosss-section in one half of the
lemniscate r 2 = a 2 cos 2θ with axis vertical and node downwards, a particle is projected
with velocity V, along the inner surface in the plane of the cross-section. Show that it will
make a complete revolution if 3V 2 > 7 ag.
6. A smooth parabolic tube is placed, vertex downwards, in a vertical plane. A particle slides

ap
down the tube from rest under influence of gravity. Prove that in any position the reaction
h+a
of the tube is 2w . , where w is the weight of the particle, ρ the radius of curvature,
ρ
4a the latus rectum.

l
7. A bead is constrained to move on a smooth wire in the form of an equiangular spiral. It is
sC
attached to pole of the spiral by a force mµ (distance) −2 and starts from rest at a distance b
from the pole. Show that if the equation of the spiral be r = ae θ cos α , the time of arriving at

π F b 2 I sec α.
GH 2µ JK
es
the pole is
2
8. From the lowest point of a smooth hollow cylinder whose cross-section is an ellipse of ma-
jor axis 2a and minor axis 2b and whose minor axis is vertical, a particle is projected from the
lowest point in a vertical plane perpendicular to the axis of cylinder. Show that it will leave
cc

F g . a 2 + 4b 2 I .
the cylinder if the velocity of projection lies between ( 2 gb ) and GH b
JK
Su

µ λ
9. A small bead moves on a thin ellipse wire under a force to the focus equal to 2 + 3 and
r r
is projected from a point on the wire distant R from the focus with the velocity which would
µ
cause it to describe the ellipse freely under a force 2 . Show that the reaction of the wire
r
FG
λ 1 1 1 IJ
is
H −
ρ r 2 ar R 2
+
K
, where ρ is the radius of curvature.

10. An elastic string of modulus λ is attached at one end to a focus of a smooth wire in the
shape of an ellipse of latus rectum 2l and major axis 2a. The other end of the string is
attached to a small ring of unit mass which can slide on the wire which is fixed with its plane
horizontal. If the ring be slightly displaced from its position of unstable equilibrium at the
end of major axis of the ellipse, show that its angular velocity about the focus when the
string becomes slack is
λl 
 3 ( a − l ) .
a 
SuccessClap: Best Coaching for UPSC Mathematics : For Info- 9346856874
Checkout ->22 Weeks Study Plan, Videos, Question Bank Solutions, Test Series

Motion on Smooth and Rough Plane 745


4.3 MOTION ON A ROUGH CURVE UNDER GRAVITY
A particle slides down a rough curve in a vertical plane
under gravity, to discuss the motion. ψ
Let P be the position of the particle at time t. At P, let the µR R
tangent is making an angle ψ with any fixed horizontal line
and the arcural distance of P measured from a fixed point A be s
A
s. Let R be the normal reaction. Since the particle slides
ψ
downwards, hence the force of friction µR acts upwards along
the tangent at P. mg
Resolving forces along the tangent and normal at P, we
get the equations of motions are
FG IJ
dv
mv
H K
ds
= mg sin ψ − µR

1 F dv 2 I
mG J = mg sin ψ − µR

ap
or 2 H ds K
F v2 I
m G J − mg cos ψ − R
and
H ρK

l
Eliminating R from (i) and (ii) by multiplying (ii) by µ and subtracting from (i), we get
sC
1 dv 2 v2
m − µm = mg sin ψ − µ mg cos ψ
2 ds ρ
dv 2
es
or ρ − 2µv 2 = 2 gρ (sin ψ − µ cos ψ )
ds
dv 2 ds
− 2µv 2 = 2 gρ (sin ψ − µ cos ψ )
FG∵ ρ = ds IJ
or .
ds dψ H dψ K
cc

dv2
or − 2µv2 = 2 gρ (sin ψ − µ cos ψ)

This is a linear differential equation in v 2 whose integrating factor is e −2µψ . Hence, its solution
will be
Su

z
v 2 . e −2µψ = 2 g ρ . 2 −2µψ (sin ψ − µ cos ψ ) dψ + c,
where c is a constant of integration.
When the equation of the curve is given, ρ can be determined in terms of ψ. Hence by
substituting the value of ρ in right hand side of (iii) it can be easily integrated. The value of c can
be determine by initial conditions.
Thus, from (iii) we can find the value of v 2 in any position and then by substituting for v 2 in
(ii) the value of R can be determined.
EXAMPLES
1. A particle is projected horizontally with velocity V along the inside of a rough vertical
circle from the lowest point. Prove that if it completes the circle, it will return to the lowest point
with a velocity v given by
2 ga
v 2 = V 2 e −4 πµ + (1 − 2µ 2 ) (1 − e −4 πµ )
1 + 4µ 2
Solution. Equations of motions will be
dv
mv = − µR − mg sin θ ...(i)
ds
SuccessClap: Best Coaching for UPSC Mathematics : For Info- 9346856874
Checkout ->22 Weeks Study Plan, Videos, Question Bank Solutions, Test Series

746 Dynamics
v2
and = R − mg cos θ
m ...(ii)
ρ
For circle, we have ρ = a. Putting the values of R from (ii) into (i), we get
m dv 2 v2 B
. = − µm − µ mg cos θ − mg sin θ
2 ds a
m dv 2 dθ v2
or . = − µm . − mg (µ cos θ + sin θ) dv
2 dθ ds a O mv ds
dθ 1 R
Since s = aθ ∴ = θ
ds a
P
dv 2
∴ + 2µv 2 = − 2 ga (µ cos θ + sin θ) ...(iii) A θ

θ

°–
90
This is a linear equation. Hence, its I.F. = e∫ 2µdθ = e2µθ
Hence, the solution of (iii) will be mg

ap
z
v 2 . e 2µθ = − 2 ga (µe 2µθ cos θ + e 2µθ sin θ) dθ

or v . e2 2 µθ
L
= − 2 ga Mµ .
e µθ 2
( 2µ cos θ + sin θ) +
e 2 µθ
(2µ sin θ − cos θ) + A,
OP
MN 1 + 4µ 2
1 + 4µ 2
PQ

l
where A is the constant of integration.
sC e 2 µθ
or v 2 . e 2 µθ = − 2 ga . [3µ sin θ − (1 − 2µ 2 ) cos θ] + A ...(iv)
(1 + 4µ 2 )
Initially, when θ = 0, v = V (given)
2 ga
V 2 .1 = − [0 − (1 − 2µ 2 )] + A
es
∴ 1 + 4µ 2

2 2 ga
∴ A=V − (1 − 2µ 2 )
2
1 + 4µ
cc

By putting the value of A in (iv), we get


e 2 µθ 2 ga
v 2 e 2 µθ = − 2 ga . [3µ sin θ − (1 − 2µ 2 ) cos θ] + V 2 − (1 − 2µ 2 ) ...(v)
2 2
1 + 4µ (1 + 4µ )
When the particle returns to the lowest point, it would have described an angle θ = 2π and let
Su

the velocity be v1. Then from (v), we get


−2 ga 4 πµ ga
v12 e 4 πµ = e [0 − (1 − 2µ 2 ) . 1] + V 2 − (1 − 2µ 2 )
2 2
1 + 4µ (1 + 4µ )
Dividign by e 4πµ , we get
2 ga
v12 = V 2 e −4 πµ + (1 − 2µ 2 ) (1 − e −4 πµ )
2
1 + 4µ
2. A particle under no forces is projected with velocity V in a rough tube in the form of an
equiangular spiral at a distance a from the pole and towards the pole. Show that it will arrive at
the pole in time
a 1
V cos α − µ sin α
α being the angle of the spiral and (µ < cot α) the coefficient of friction.
Solution. The equation of the equiangular spiral is
r = ae θ cot α ...(i)
SuccessClap: Best Coaching for UPSC Mathematics : For Info- 9346856874
Checkout ->22 Weeks Study Plan, Videos, Question Bank Solutions, Test Series

Motion on Smooth and Rough Plane 747


Also, φ = α and ψ = θ + φ = θ + α and its pedal equation is p = r sin α ;
dp dr
∴ = sin α and ρ = r = r cosec α.
dr dp
The particle is moving under no forces and hence we have the following equations of motion.
dv
mv = µR ...(ii)
ds
v2
and m =R ...(iii)
ρ
dv v2
or mv =µ.m
ds ρ
1 dv 2 dθ µv 2
or . =
2 dθ ds r cosec α
1 dv 2 dθ µv 2

ap
or .r =
2 dθ ds cosec α
dθ dθ
But tan φ = r ; ∴ sin φ = r . = sin α , because φ = α.
dr ds
1 dv 2

l
∴ sin α − µv 2 sin α = 0
2 dθ
dv 2
sC
or − 2 µv 2 = 0

This is a linear differential equation whose integrating factor = e −2µθ .
es
Hence, the solution of the equation will be
v 2 . e −2µθ = c 2 or v = ce µθ ,
where c 2 is the constant of integration.
FG IJ 1/cot α = eθ
cc

r
From (i), we have
a H K
FG r IJ µ tan α = eµθ
or
H aK
Su

F r I µ tan α
v=cG J
∴ H aK
Initially, when r = a, v = − V (towards the pole) ∴ c = − V
FG r IJ µ tan α (−V ) FG r IJ µ tan α
∴ v=
H aK or v=−V
H aK
ds r µ tan α ds dr r µ tan α
or =−V. or . =−V.
dt a µ tan α dr dt a µ tan α
dr dr
But cos φ = or cos α = ,∵ φ = α
ds ds
a µ tan α 0 − µ tan α

1
.
cos α dt
dr
=−
a
V
µ tan α
r µ tan α or −
V cos α a r z
dr = 0t dt z

LM
a µ tan α r − µ tan α + 1 OP 0
=1
or
MN
V cos α − µ tan α + 1 PQ α
SuccessClap: Best Coaching for UPSC Mathematics : For Info- 9346856874
Checkout ->22 Weeks Study Plan, Videos, Question Bank Solutions, Test Series

748 Dynamics
a µ tan α 0 − a − µ tan α + 1 a µ tan α − µ tan α + 1 cos α
∴ t=− . = .
V cos α −µ tan α + 1 V cos α cos α − µ sin α
a 1
= . .
V cos α − µ sin α
3. A particle slides in a vertical plane down a rough cycloidal arc where axis is vertical and
vertex downwards starting from a point where the tangent makes an angle θ with the horizon
µθ
and coming to rest at the vertex; show that µe = sin θ − µ cos θ.
Solution. Intrinsic equation of a cycloid is
B C A
s = 4a sin ψ ...(i)
ds dv
ρ= = 4a cos ψ. Its length = 8a R mv
ds

P
π

ap
Also ψ = for cusp and ψ = 0 at the vertex. s
ψ mg
2
O
Since the particle is sliding down the arc, the
force of friction acts in upwards direction. Hence, equations of motion will be
dv

l
mv = µR − mg sin ψ ...(ii)
ds

and
sC
m
v2
= R − mg cos ψ ...(iii)
ρ
Eliminating R between (ii) and (iii), we get
dv  v2 
es
mv = µm + mg cos ψ  − mg sin ψ
 
ds  ρ 
1 dv 2 v2
or =µ + g (µ cos ψ − sin ψ )
2 ds ρ
cc

Multiplying both sides by 2ρ, we get


dv 2 ds
. − 2µv 2 = 2 g .4a cos ψ (µ cos ψ − sin ψ )
ds d ψ
Su

ds
∵ ρ= = 4a cos ψ

dv 2
or − 2µv 2 = 8ag cos ψ (µ cos ψ − sin ψ ) ...(iv)

−2µψ
This is linear differential equation and its integrating factor is e .
Hence, solution of this equation will be
v 2 .e −2µv = 8ag ∫ e −2µv cos ψ (µ cos ψ − sin ψ ) d ψ + A ...(v)
where A is constant of integration.
Put e −µψ (µ cos ψ − sin ψ ) = z.
∴ [−µe −µψ (µ cos ψ − sin ψ ) + e−µψ (−µ sin ψ − cos ψ )] d ψ = dz
or − e −µψ [1 + µ 2 ] cos ψ d ψ = dz
dz
∴ e −µψ cos ψ d ψ = − ...(vi)
1 + µ2
Hence, from (v) and (vi), we get
SuccessClap: Best Coaching for UPSC Mathematics : For Info- 9346856874
Checkout ->22 Weeks Study Plan, Videos, Question Bank Solutions, Test Series

Motion on Smooth and Rough Plane 749


v 2 e−2µψ = 8ag ∫ e−µψ . cos ψ e −µψ (µ cos ψ − sin ψ ) d ψ + A
− dz
= 8ag ∫ z . +A
1 + µ2
8agz 2
=− +A
2 (1 + µ 2 )
8ag −2µψ
or v 2 e−2µψ = − e (µ cos ψ − sin ψ ) 2 + A ...(vii)
1 + µ2
We are given that v = 0 when ψ = 0.
4ag 4ag
∴ A= e −2µθ (µ cos θ − sin θ) 2 or v 2 = e −2µθ (µ cos θ − sin θ) 2
2 2
(1 + µ ) (1 + µ )
4ag −2µψ
− e (µ cos ψ − sin ψ ) 2 ...(viii)
2
1+ µ

ap
Again, we are given that it comes to rest at the vertex where ψ = 0.
Hence, putting v = 0 and ψ = 0 in (viii), we get
4ag −2µθ 4ag
e (µ cos θ − sin θ) 2 = (µ) 2
2 2
1+ µ 1+ µ

l
or e−2µθ (µ cos θ − sin θ) 2 = µ 2
or
sC
µ = ± e −µθ (µ cos θ − sin θ) or µeµθ = ± (µ cos θ − sin θ)
Taking negative sign, we get
µ eµθ = sin θ − µ cos θ
es
4. A small bead is threaded on a rough rigid wire in the form of an equiangular spiral of
angle α. The bead is projected away from the pole with any velocity. Prove that the intervals of
time between the successive instants at which the bead is moving in the same direction as at first
form a G.P. of common ratio e 2π (µ + cot α ) .
cc

Solution. The equations of motions are


dv
mv = − µR ...(i)
ds
mv 2
Su

and =R ...(ii)
ρ
Eliminating R, we get
dv v2 dv ds
mv = −µ.m or . = − µv
ds ρ ds d ψ
dv dv
or = − µv or = − µ d ψ.
dψ v
Integrating, log v = − µψ + log k ,
where k is a constant.
v ds
or log = − µψ or v = ke −µψ =
k dt
ds d ψ
or . = ke−µψ ...(iii)
d ψ dt
Now, the equation of equiangular spiral of angle α is r = aeθ cot α and its pedal equation is
p = r sin α and φ = α. Hence,
SuccessClap: Best Coaching for UPSC Mathematics : For Info- 9346856874
Checkout ->22 Weeks Study Plan, Videos, Question Bank Solutions, Test Series

750 Dynamics
ds dr r
=ρ=r = = r cosec α. Also ψ = θ + φ = θ + α.
dψ dp sin α
Hence, from (iii), we get
r dψ d ψ k sin αe−µψ k sin α e−µψ
. = ke−µψ or = =
sin α dt dt r aeθ cot α
d ψ k sin α −µψ − (ψ − α ) cot α
or = e e ∵ ψ=θ+α
dt a
d ψ k sin α α cot α − (µ + cot α ) ψ
or = e .e
dt a
k sin α α cot α
or e(µ + cot α ) ψ d ψ = e dt
a
e(µ + cot α ) ψ k sin α α cot α
Integrating, = e t + A,
µ + cot α a

ap
where A is constant of integration. Since α, k , a and µ are constants, above equation can be
put in the form
e(µ + cot α ) ψ = Bt + c ...(iv)
Let initially, when t = 0, ψ = ψ 0

l
∴ c = e(µ + cot α ) ψ 0 ...(v)
sC
Since we are to find the intervals, when the particles is moving in the same direction as at first,
hence we give to ψ the values ψ 0 + 2π, ψ 0 + 4π, ψ 0 + 6π and so on and let the corresponding
values of t be t1 , t2 , t3 and so on. We are to find the values of t2 − t1 , t3 − t2 and so on.
Putting these values in (iv), we get
es
e(µ + cot α ) (ψ 0 + 2π) = Bt1 + c
or a (µ + cot α ) ψ 0 e(µ + cot α ) 2π − c = Bt1
or c {e(µ + cot α ) 2π − 1} = Bt1 by (v)
cc

Similarly, c {e(µ + cot α ) 4π − 1} = Bt2


and c {e(µ + cot α ) 6π − 1} = Bt3 and so on .
c
∴ t2 − t1 = e(µ + cot α ) 2 π [e(µ + cot α ) 2 π − 1]
Su

B
c
t3 − t2 = e(µ + cot α ) 4 π [e(µ + cot α ) 2 π − 1]
B
Obviously, above intervals of time t2 − t1 , t3 − t2 , ... form a G.P. whose common ratio is
e(µ + cot α ) 2 π .
5. A particle starts from rest from the cusp of a rough cycloid whose axis is vertical and
vertex downwards. Show that its velocity at the vertex is to its velocity at the same point when the
cycloid is smooth as
(e −µπ − µ 2 )1/ 2 : (1 + µ 2 )
where µ is the coefficient of friction. Further show that the particle will certainly come to
rest before reaching the vertex if the coefficient of friction be 0.5, having given that log 2 = .69315.
Solution. Proceeding exactly as in Example 3, we arrive at result (vii) as
4ag −2µv
v 2 e −2µψ = − e (µ cos ψ − sin ψ ) 2 + A
2
1+ µ
π
Since the particle starts from rest at the cusp when ψ = , we have
2
SuccessClap: Best Coaching for UPSC Mathematics : For Info- 9346856874
Checkout ->22 Weeks Study Plan, Videos, Question Bank Solutions, Test Series

Motion on Smooth and Rough Plane 751


4ag
0=− e−µπ (0 − 1) 2 + A
1 + µ2
4ag −µπ
∴ A= e
1 + µ2
4ag −µπ 4ag −2µψ
∴ e −2µψ = e − e (µ cos ψ − sin ψ ) 2
2 2
1+ µ 1+ µ
Let v1 be the velocity at the vertex where ψ = 0. Then we have from above by putting
v = v1 and ψ = 0.
4ag −µπ 4ag
∴ v12 .1 = e − 1 . (µ .1 − 0) 2
2 2
1+ µ 1+ µ
2 4ag −µπ
or v1 = (e − µ2 ) ...(i)
2
1+ µ
Let v2 be the velocity at the vertex when the cycloid is smooth, then putting µ = 0 in (i), we

ap
get
4ag
v22 = (1 − 0) = 4ag ...(ii)
1
From (i) and (ii), we have

l
v12 e−µπ − µ2
=
sC v22 1 + µ2
or v1 : v2 = (e −µπ − µ 2 )1/ 2 : (1 + µ 2 )
Now, let the paricle comes to rest at the lowest point, i.e., vertex.
es
Then v1 = 0 and hence from (i),
e−µx − µ 2 = 0 or µ 2 eµπ = 1. Hence, it will coem to rest before reaching the vertex if
µπ
µ 2 eµπ > 1 or µeµπ / 2 > 1 or if log µ + log e > log 1
2
cc

µπ 1 1 22
or log µ + log e > 0 or log + . .1 > 0
2 2 4 7
1 22
since µ = .5 = and π =
Su

2 7
22 22
or if − log 2 > 0 or − .69315 > 0
28 28
22
which is true as = .785.
28
Hence, if µ = .5, the particle will come to rest before reaching the vertex.
6. A rough parabolic wire with latus rectum 4a is placed with its axis vertical and vertex
downwards and a bead is projected along it from the lowest point with velocity u. Show that the
a
bead will come to rest at a distance from the focus where
n2
  
 u2  
µ cos −1 n = log n 1 +  .
 
  2ag  
Solution.Taking S as pole and SX as initial line, the polar equation of the parabola will be
2a θ
= 1 + cos θ or r = a sec 2
r 2
SuccessClap: Best Coaching for UPSC Mathematics : For Info- 9346856874
Checkout ->22 Weeks Study Plan, Videos, Question Bank Solutions, Test Series

752 Dynamics
dr θ θ θ 1
∴ = 2a sec sec tan .
dθ 2 2 2 2
1 dr θ θ
or = tan or cot φ = tan
r dθ 2 2 R
θ P
∴ φ = 90° − S
2 θ
θ
and ψ = θ + φ = 90° + ...(ii) A M
2 µR
 θ θ a
Also, p = r sin φ = r sin  90° +  = r cos = r   = (ar )
 2 2 r
dp a dr
∴ = . Hence, ρ = r
dr 2 r dp
r θ
= 2r   = 2a sec3 ...(iii)

ap
 
a 2
a 2 θ a
Also, when r = , then a sec =
n 2 2 n2
θ

l
or cos = n
2

sC θ
= cos −1 n ...(iv)
2
The equations of motion will be
dv θ
mv = − µR − mg cos φ = − µR − mg sin by (ii) ...(v)
es
ds 2
v2 θ
and m = R − mg sin φ = R − mg cos ...(vi)
ρ 2
Eliminating R between (v) & (vi), we get
cc

dv  mv 2 θ θ
mv =−µ + mg cos  − mg sin
ds  ρ 2  2
 
1 dv 2 µv 2  θ θ
or` + = − g  µ cos + sin 
Su

2 ds ρ  2 2
Multiplying both sides by 2ρ and putting
θ
ρ = 2a sec3 by (iii),
2
dv 2 ds θ θ θ
we get . + 2µv 2 = − 2 g . 2a sec3  µ cos + sin 
ds d ψ 2 2 2
2
dv d θ  θ θ θ
. + 2µv 2 = − 4ag  µ sec 2 + tan sec 2 
dθ dψ  2 2 2
  
2
dv  θ θ θ dθ
or . 2 + 2µv 2 = − 4ag  µ sec 2 + tan sec 2 θ  ∵ ψ = 90° + , ∴ = 2
dθ  2 2   2 d ψ 
dv 2 2  2 θ θ 2 θ
or + µv = − 2ag  µ sec + tan sec 
dθ  2 2 2
This is linear equation in v 2 and its I.F. = eµθ . Hence, the solution of this equation will be
 θ θ θ θ
v 2 .eµθ = − 2ag ∫ eµθ  tan sec sec + µ sec2  d θ + A
 2 2 2 2
SuccessClap: Best Coaching for UPSC Mathematics : For Info- 9346856874
Checkout ->22 Weeks Study Plan, Videos, Question Bank Solutions, Test Series

Motion on Smooth and Rough Plane 753


 θ θ θ 
= − 2ag eµθ sec2 − ∫ eµθ . µ sec2 d θ + ∫ eµθ µ sec2 d θ + A
 2 2 2 
θ
or v 2 eµθ = − 2ag eµθ sec2 + A
2
Initially, for the vertex θ = 0 and v = u
∴ u 2 = − 2ag + A or A = u 2 + 2ag
θ
∴ v 2 eµθ = − 2ageµθ sec2 + u 2 + 2ag
2
The bead will come to rest when v = 0
θ
∴ 2ag eµθ sec2 = u 2 + 2ag
2
θ µ2
or eµθ sec2 = 1 +
2 2ag

ap
µθ / 2 θ  u2 
e = cos  1 + 
or 2  2ag 
 u2 
= n 1 +  , by (iv)

l
 2ag 

sC
Taking log of both sides, we get
  
µθ u2  
= log  n 1 + 
2   2ag  
 
es
  2 
u
µ cos −1 n = log  n 1 + 
  2ag   by (iv).
 
cc

7. A particle P of unit mass describes on ellipse under an attraction f to focus S and attraction
f ′ to the other focus H. If SP = r and HP = r ′, prove that
1 d 2 1 d
(r f ) = (r ′2 f ′)
2 dr
r ′2 dr ′
Su

r
Hence, show that if one force obeys the Newtonian law, so also must obey the other. If the
forces are equal, then each varies inversely as the product of focal distance of P.
Solution. There is a force f at P towards focus S, where
SP = r and a force f ′ at P towards focus H where HP = r ′. Z
Y P
We know that tangent and normal at any point on an ellipse φ
φ r'
are equally inclined to the focal radii SP and HP. r F'
F
∴ ∠SPY = ∠HPZ = φ
S G M
SY SY
sin φ = =
SP r
HZ HZ
and sin φ = =
HP r′
SY . HZ b2
sin 2 φ = = ∵ SY . HZ = b 2
rr ′ rr ′
b
Hence, sin φ =
( rr ′)
SuccessClap: Best Coaching for UPSC Mathematics : For Info- 9346856874
Checkout ->22 Weeks Study Plan, Videos, Question Bank Solutions, Test Series

754 Dynamics
Again we know that in an ellipse,
x = a cot t , y = b sin t
3/ 2
 dx 2  dy 2 
  −   
 dt   dt   (a 2 sin 2 t − b 2 cos 2 t )3 / 2
ρ=  =
∴  dx d 2 y dy d 2 x  ab
 . 2 − . 2
 dt dt dt dt 
[a 2 (1 − cos 2 t ) + b 2 cos 2 t ]3 / 2 [a 2 − (a 2 − b 2 ) cos 2 t ]3 / 2
= =
ab ab
(a 2 − a 2 e2 cos 2 t )3 / 2 (a 2 − e2 x 2 )3 / 2 [(a − ex) (a + ex)]3 / 2 ( SP.HP )3 / 2
= = = =
ab ab ab ab
(rr ′)3 / 2
= ...(ii)
ab

ap
Also, SP + HP = 2a or r + r ′ = 2a
Resolving the forces along the tangent, we have
dv dr dr ′ dr
v = f cos φ − f ′ cos φ = f − f′ , ∵ cos φ = .
ds ds ds ds

l
1 2
∴ dv = − ( f dr + f ′dr ′) ...(iii)
2
Resolving along the normal, we get
sC
v2
= f sin φ + f ′ sin φ [∵ m = 1]
ρ
v 2 = ( f + f ′) ρ sin φ
es
or ...(iv)
Putting for v 2 from (iv) in (iii), we get
d [( f + f ′) ρ sin φ] = − 2 f dr 2 f ′ dr ′
 (rr ′)3 / 2 b 
cc

or d  ( f + f ′) .  = − 2 f dr − 2 f ′ dr ′ by (i) and (ii)


 ab (rr ′) 
1
or d [( f + f ′) rr ′] = − 2 f dr − 2 f ′ dr ′
a
Su

or d ( frr ′) + d ( f ′ rr ′) = − 2a ( fdr + f ′ dr ′)
d d
or ( frr ′) dr + ( f ′ rr ′) dr ′ = − 2a ( f dr + f ′ dr ′)
dr dr ′
Now, we know that in an ellipse sum of the focal distances is constant and equal to 2a.
∴ r + r ′ = 2a, ∴ dr = − dr ′
d d
∴ ( frr ′) dr − ( f ′ rr ′) dr = − 2a ( f dr − f ′ dr )
dr dr ′
d d
or ( frr ′) − ( f ′rr ′) = − (r + r ′) ( f − f ′)
dr dr ′
 df dr ′   df ′ dr 
 rr ′ + fr ′ + fr  −  rr ′ ′ + f r′ + f r′ ′ ′  = − f (r + r ′) + f ′ (r + r ′)
 dr dr   dr dr 
dr dr ′
Putting = = − 1, we get
dr ′ dr
 ′ dr   df ′ 
 rr + fr ′ − fr  −  rr ′ + f r′ + f r′ ′  = − f (r + r ′) + f ′ (r + r ′)
 dr   dr ′ 
SuccessClap: Best Coaching for UPSC Mathematics : For Info- 9346856874
Checkout ->22 Weeks Study Plan, Videos, Question Bank Solutions, Test Series

Motion on Smooth and Rough Plane 755


df df ′
or rr ′ + 2 fr ′ = rr ′ + 2 f ′r.
dr dr ′
Dividing throughout by rr ′, we get
df 2 f df ′ 2 f ′
+ = +
dr r dr ′ r′
1  2 df  1  ′2 df ′ 
r + 2r . f  = r + 2r ′r ′ 
or 2  dr  r  2 dr ′ 
r
1 d 2 1 d 2
or (r f ) = (r ′ f ′) ...(ii)
r 2 dr r ′2 dr ′
µ
Now if f obeys Newtonian Law, then f = 2 or r 2 f = µ, i.e., constant.
r
Hence, from (iv), we get
1 d 1 d 1 d
(µ) = (r ′2 f ′) or 0 = (r ′2 f ′)
2
r dr r ′ dr ′ r ′2 dr ′

ap
µ′
∴ r ′2 f ′ = cons tant = µ′ ∴ f ′ =
r ′2
Hence, clearly f ′ also obeys Newtonian Law.
From above

l
µ µ′ µµ′
sC f =
r 2
and also f ′ =
r′ 2
∴ ff ′ =
r 2 r ′2
But if he forces be equal, i.e., f ′ = f .
µµ′ (µµ′) k
then f2 = or f = =
es
2 ′2 rr ′ rr ′
r r
i.e., force varies inversely as the product of the focal distances of the point.
EXERCISES
1. A particle falls from a position of limiting equilibrium near the top of a nearly smooth glass
cc

sphere. Show that it will leave he sphere at the point whose radius is inclined to the vertical
 4α  2
at an angle α + µ  2 −  where cos α = and µ is the small coefficient of friction.
 3 sin α  3
2. A particle is projected along the inner surface of a rough sphere and is acted on by no
Su

a
forces. Show that it will return to the point of projection at the end of time (e2µπ − 1),
µV
where a is the radius of the sphere, V is the velocity of projection and µ is the coefficient of
friction.
3. A rough cycloid has its plane vertical and the line joining its cusp horizontal. A heavy particle
slides down the curve from rest at a cusp and comes to rest again at the point on the other
side of the vertex where the tangent is inclined at 45° to the vertical. Show that the coefficient
of friction satisfies the equation
3µπ + 4 log (1 + µ) = 2 log 2
4. A bead moves along a rough curve wire which is such that it changes its direction of
motion with constant angular velocity. Show that a possible form of wire is an
equiangular spiral.
5. A smooth wire in the form of a parabola of latus rectum 4a is fixed in a horizontal place. A
small ring of mass m can slide on it and is attached to the focus by a light elastic string of
natural length a and modulus λ. If the ring is held at the end of the latus rectum and released,
show that its distance from the axis of the parabola at time t is
  λ 
2a cos t  
  4am   ❐❐❐
SuccessClap: Best Coaching for UPSC Mathematics : For Info- 9346856874
Checkout ->22 Weeks Study Plan, Videos, Question Bank Solutions, Test Series

Motion in a Resisting Medium


5.1 INTRODUCTION
We are familiar with several motions in vacuum, for example the motion of a projectile in vacuum.
In vacuum the moving body does not experience any resistance to its motion but when a body
moves in a medium like air, it feels a resistance to its motion and hence it is called the motion in a
resisting medium. It has been observed that resistance increases as the velocity of the body increases
and hence the resistance may be assumed to be some function of the velocity. The resisting force
always acts in the opposite direction of motion and is non-conservative and hence the principle

ap
of conservation of energy can not be applied.
All the laws of resistance, in fact, are more or less empiric.

5.2 TERMINAL VELOCITY

l
Let a particle of mass m be falling under gravity in air and let the resistance be proportional to
sC
n
v , where v is the velocity at a distance x. Then equation of motion will be
d2x dv
= = g − kv n ..(i)
2 dt
dt
es
where g is acceleration due to gravity. The negative sign to kv n shows that there is retardation
d2x
due to air resistance. Equation (i) shows that with the increase in velocity the acceleration
dt 2
cc

goes on decreasing till the particle acquires a velocity which makes the right hand side of (i) vanish.
Let this velocity be V, then
1/ n
g
0 = g − kV n or V =   ...(ii)
k
Su

dv
After v has reached the value V, vanishes. Hence, it ceases to increase any more and thus
dt
v can never exceed the value V.
If we consider the case in which the particle is projected downwards with a velocity greater
dv
than V, then the right hand side of (i) will be negative and hence will be negative. It means that
dt
from the beginning of motion there is retardation. Due to this retardation, the velocity of particle
dv
will go on decreasing till it reaches the velocity V. When this velocity is reached, = 0 and then
dt
velocity ceases to decrease.
Obviously, in either case the ultimate velocity is V. This velocity V is called the terminal velocity.
For example, a rain drop falling on the earth can not give an idea of the height from which it is
coming as it will attain terminal velocity much earlier than it has fallen on the ground, while for a
feather falling from the top of a tower, it is easy to see that the feather will have practically the
same constant velocity in the last four or five metres.
SuccessClap: Best Coaching for UPSC Mathematics : For Info- 9346856874
Checkout ->22 Weeks Study Plan, Videos, Question Bank Solutions, Test Series

Motion in a Resisting Medium 757


5.3 MOTION IN A VERTICAL LINE DOWNWARDS
A particle falls under gravity, supposed constant, in a resisting medium whose resistance
varies as the square of the velocity; to find the motion of the particle if it starts from rest.
Let a particle falls, from rest, vertically downwards from A. Let P be its position when it has
described a distance x in time t. The forces acting at P will be : A
(i) The weight mg of the particle acting vertically downwards, and
(ii) The force mkv 2 due to resistance acting vertically upwards.
Hence, the equation of motion will be λ mkv2
d2x dv
m = mv = mg − mkv 2
dt 2 dx
dv  k 
or v = g 1 − v 2  P
...(i)
mg
dx  g 
Let V be the terminal velocity, i.e., the velocity when the downward acceleration is zero. Then
from (i)

ap
 k  k
0 = g 1 − V 2  or 1 − V 2 = 0
 g  g
2 g
or V = ...(ii)
k

l
From (ii) and (i), we get
sC
v
dv
dx

= g 1 −
 V2 
v2 
 ...(iii)
 
v dv g
or = dx
2 2
V −v V2
es
2 gx
Integrating, we get log (V 2 − v 2 ) = − + C,
V2
where C is constant of integration.
cc

Initially at A, x = 0, v = 0. ∴ C = log V 2
2 gx
Hence, log (V 2 − v 2 ) = − + log V 2
V2
 V 2 − v2  2 gx V 2 − v2 2
Su

log   = − or = e( −2 gx / V )
or  V 2  2 2
  V V
2
v 2 = V 2 (1 − e( −2 gx / V ) ) ...(iv)
Equation (iv) gives the velocity in any position.
Now from equation (iii), we get
dv  v2  (V 2 − v 2 ) dv g
= g 1 −  =g or = dt
dt  2  2 (V − v ) V 2
2 2
 V  V
Integrating, we get
1  v  gt
tanh −1   = + A,
V V  V
where A is constant of integration.
Initially, at A, v = 0 when t = 0, ∴ A = 0.
1 v gt
Hence, tanh −1 =
V V V2
 gt 
or v = V tanh   ...(v)
V 
SuccessClap: Best Coaching for UPSC Mathematics : For Info- 9346856874
Checkout ->22 Weeks Study Plan, Videos, Question Bank Solutions, Test Series

758 Dynamics
Equation (v) gives velocity at any time.
Equating the values of v from (iv) and (v), we get
 gt  2
V 2 tanh 2   = V 2 (1 − e−2 gx / v )
V 
−2 gx / V 2  gt   gt 
or e = 1 − tanh 2   = sech 2  
V  V 
2 gx / V 2 2  gt 
or e = cosh  
V 
2 gx  gt 
or = 2 log cosh  
2 V 
V
V 2   gt 
x=  log cosh  
or  g  V 
...(vi)
 
This equation gives relation between x and t.

ap
5.3.1 Motion in a Vertical Line in Upward Direction
A particle is projected upwards under gravity, supposed constant, in a resisting medium whose
resistance varies as the square of the velocity; to discuss the motion.
Let a particle of mass m be projected from A with initial velocity u, vertically under gravity. Let

l
P be its position when it has described a distance x in time t. The forces acting on the particle at P
sC
will be P
(i) Weight of the particle mg, acting vertically downwards, and
(ii) The force mkv 2 due to resistance acting vertically downwards. mg mkv2
Hence, the equation of motion will be x
es
d2x dv
m = mv = − mg − mkv 2
2 dx
dt
dv  k 
or v = − g 1 + v 2  A ...(i)
cc

dx  g 
Let V be the terminal velocity, i.e.,the velocity when the downwards acceleration is zero, then
from (i) of 5.3.
 k  g
0 = g  1 − V 2  or V 2 = ...(ii)
Su

 g  k
Substituting from (ii) in (i), we get
dv  v2  v dv g
v = − g 1 +  or =− dx
dx  V2  2
V +v 2
V2
 
Integrating, we get
2 gx
log (V 2 + v 2 ) = − + C,
V2
where C is constant of integration.
Initially, at A, v = u, x = 0;
∴ C = log (V 2 + u 2 )
2 gx
∴ log (V 2 + v 2 ) = − + log (V 2 + u 2 )
2
V
 V 2 + u 2  2 gx
log  =
or  V 2 + v2  V 2 ...(iv)
 
This gives velocity in any position, Equation (ii) may be written as
SuccessClap: Best Coaching for UPSC Mathematics : For Info- 9346856874
Checkout ->22 Weeks Study Plan, Videos, Question Bank Solutions, Test Series

Motion in a Resisting Medium 759


dv g  dv dv 
=− (V 2 + v 2 ) ∵ v dx = dt 
dt V2  
dv g
or =− dt
2 2
V +v V2
Integrating, we get
1 v gt
tan −1 = − + A,
V V V2
where A is constant of integration.
1 u
At A, v = u where t = 0, ∴ A = tan −1
V V
1 −1 u gt 1 u
∴ tan =− + tan −1
V V 2 V V
V
V  −1  u   v 
or t =  tan   − tan −1    ...(v)
g   
V  V 

ap
This is the relation between velocity and time.
5.4 RESISTANCE PROPORTIONAL TO THE VELOCITY
(a) A particle falls under gravity (supposed constant) from rest in a medium whose resistance

l
varies as the velocity; to discuss the motion.
sC
Let at time t, the particle has fallen through a distance x when v is its velocity. The forces
acting at the particle at P will be
(i) The weight mg of the particle acting vertically downwards, and
O
(ii) The force mkv due to resistance acting vertically upwards.
Then the equation of motion will be
es
d2x dv
m = mv = mg − mkv

mkv
2 dx
dt
dv  k 
v =  g − v υ
cc

or ...(i)
dx  g  P m
Let V be the terminal velocity, then mg
k g2
0 = g − V or V = . O'
g k
Su

dv g
∴ v =g− v
dx V
dv g
+ v=g  dv dv 
or ...(ii) ∵ v =
dt V  dx dt 
This is a linear differential equation whose
I.F. = e ∫ ( g / V ) dt = e( g / V ) t
Hence solution of (ii) will be
ve( g / v ) t = g ∫ e( g / V ) t dt + A or ve( g / V ) t = Ve( g / V ) t + A,
where A is constant of integration.
Initially, t = 0, v = 0, ∴A= −V
∴ ve( g / V ) t = Ve( g / V ) t − V
or v = V − Ve −( g / V ) t ...(iii)
dx
or = V − Ve − ( g / V ) t
dt
SuccessClap: Best Coaching for UPSC Mathematics : For Info- 9346856874
Checkout ->22 Weeks Study Plan, Videos, Question Bank Solutions, Test Series

760 Dynamics
g
V2 −V t
Inetgrating, we get x = vt + e + B,
g
where B is constant of integration.
V2
Initially, t = 0, x = 0, ∴ B=−
g
g
V2 V2 −V t
Hence, x = Vt − + e ...(iv)
g g
This is the relation between x and t.
(b) A particle is projected upwards with a velocity u in a medium whose resistance varies as
the velocity to discuss the motion.
Let the particle be projected from A with initial velocity u, vertically up-wards, under gravity.
Let P be its position when it has described a distance x time t. The force acting on the particle at P
will be
(i) The weight mg of the particle acting vertically downwards, and

ap
A'
(ii) The force mkv due to resistance acting vertically downwards.
Hence, the equation of motion will be
d2x dv υ
m = mv = − mg − mkv P
2 dx

l
dt
mkv
 k 
or
sC v
dv
dx
= − g 

1 + v
g 
 ...(i) mg

g u
Let V be the terminal velocity, then V = .
k A
es
dv g
∴ v =−g− v
dx V
dv g
or v =− (V + v ) ...(ii)
dx V
cc

v dv g
or = dx
V +v V
 V  g
or 1 −  dv = dx
 V + v  V
Su

Integrating,
g
v − V log (V + v ) = − x + A,
V
where A is constant of integration.
Initially, x = 0, v = u, ∴ A = u − V log (V + u )
V +v g
∴ v − u − V log =− x
u +V V
V2 V +u V
or x= log + (u − v) ...(iii)
g V +u g
Let H be the maximum height then at x = H , v = 0. Hence,
V2 V Vu
H= log +
g V +v g
Equation (iii) gives relation between x and v.
Again equation (ii) may be written as
dv g  dv dv 
+ =−g ∵v =
dt V  dx dt 
SuccessClap: Best Coaching for UPSC Mathematics : For Info- 9346856874
Checkout ->22 Weeks Study Plan, Videos, Question Bank Solutions, Test Series

Motion in a Resisting Medium 761


This is a linear equation whose I.F. = e( g / V ) t . Hence, its solution will be
ve( g / V ) t = − g ∫ e( g / V ) t dt + B
= − Ve( g / V ) t + B,
where B is constant of integration. Initially, t = 0, v = u,
∴ B = u +V
∴ v = (u + V ) e − ( g / V ) t − V ...(v)
This is the relation between v and t. Again, (v) can be written as
dx
= (u + V ) e −( g / V ) t − V or dx = [(u + V ) e − ( g / V ) t − V ] dt
dt
Integrating, we get
V
x = − (u + V ) e − ( g / V ) t − Vt + C ,
g
where C is constant of integration.

ap
V
Initially, t = 0, x = 0 ∴ C = (u + V ) .
g
V
∴ x = (u + V ) [1 − e− ( g / V ) t ] − Vt ...(vi)
g

l
This is the relation between x and t.
sC
From relation (v) it is clear that v decreases as t increases till at the highest point, where v = 0,
we have
V
e− ( g / V ) t =
u +V
u +V 
es
V
or t = log   ...(vii)
g  V 
From (vi) and (vii) the greatest height H will be
V2 V2 u +V V
H=− − log + (u + V )
cc

g g V g
uV V 2 u +V 
= − log   ...(viii)
g g  V 
Su

5.5 RESISTANCE VARYING AS nTH POWER OF THE VELOCITY


A particle falls under gravity (supposed constant), in a resisting medium whose resistance
varies as nth power of the velocity; to find the motion of the particle if it starts from rest.
In this case, the equation of motion will be
d2x dv
m = mv = mg − mkv n
2 dx
dt
dv  k 
or v = g 1 − v n  ...(i)
dx  g 
Let V be the terminal velocity, i.e., the velocity when the downwards acceleration is zero, then
from (i)
 k  g
0 = g  1 − V n  or V n = ...(ii)
 g  k
Hence, from (i); we have
dv  vn 
v = g 1 − 
dx  Vn 
 
SuccessClap: Best Coaching for UPSC Mathematics : For Info- 9346856874
Checkout ->22 Weeks Study Plan, Videos, Question Bank Solutions, Test Series

762 Dynamics
v dv
= g dx
 vn 
or  1 − n 
 V 
 v n
v 2n v3n 
or 1 + n + 2n + 3n + ... v dv = g dx
 V V V 
Integrating,
 v2 vn + 2 v 2n + 2 
 + n
+
2 n
+ ... = gx + A
 2 (n + 2) V (2n + 2) V 
where A is constant of integration.
Initially, v = 0, x = 0; ∴ A = 0.
Hence,
v2  1 
n 2n 3n
1 v 1 v 1 v
x=  +   +   +   + ... ...(iv)
g  2 ( n + 2)  V  (2n + 2)  V  (3n + 2)  V  

ap
This is the relation between v and x.
Again, equation (iii) can be written as
dv  vn   dv dv 
= g 1 −  ∵v =
dt 
 V 
n   dx dt 

l
dv


sC
gdt =
 vn 
1 − n 
 V 
v
Since < 1, hence expanding by Binomial theorem, we get
es
V
 vn v 2n v3n 
gdt = 1 + + + + ...  dv
 V n V 2 n V 3n 
 
Integrating, we get
cc

 vn + 1 v 2n + 1 v3n + 1 
gt = v + + + + ... + B,
 (n + 1) V n (2n + 1) V 2n (3n + 1) V 3n 
where B is constant of integration.
Su

Initially, v = 0 and t = 0, ∴ B = 0 . Hence,


v  
n 2n 3n
1 v 1 v 1 v
t = 1 +   +   +   + ... ...(v)
g  ( n + 1)  V  (2n + 1)  V  (3n + 1)  V  

This is the relation between v and t.
EXAMPLES
1. Show that a particle projected upwards with a velocity U in a medium whose resistance
varies as the square of the velocity will return to the point of projection wtih velocity
UV V  U v 
v1 = after a time  tan −1 + tanh −1 1  where V is the terminal velocity.
U +V22
g  V V 
Solution. We know that the equation of motion when the particle is moving upwards is given
by
dv
= − mg − mkv 2
mv ...(i)
dx
Now, the particle moving upwards reaches the highest point B and then return back moving in
the downwards path. We know that the equation of motion when the particle is descending is given
by
SuccessClap: Best Coaching for UPSC Mathematics : For Info- 9346856874
Checkout ->22 Weeks Study Plan, Videos, Question Bank Solutions, Test Series

Motion in a Resisting Medium 763


B
dv
mv = mg − mkv 2 ...(ii)
dy
Q
dv
If V be the terminal velocity, then putting v = V and v = 0 in (ii), we get
dy
g P
0 = mg − mkV 2 or V2 = ...(iii)
k
Hence, from (i)
dv  v2  A
v = − g 1 + 
dx  V2  [using (iii)] ...(iv)
 
2v dv 2g
or =− dx
2 2
V +v V2
Integrating, we get
2g
log (V 2 + v 2 ) = −

ap
+A
V2
where A is constant of integration.
Initially, when x = 0, V = U .
∴ A = log (V 2 + U 2 )

l
2 gx

sC
log (V 2 + v 2 ) = −
V2
+ log (V 2 + U 2 )

V2 V 2 + U2 
x = log  2 
or 2g  V + v2  ...(v)
 
es
Let h be the greatest height attained by the particle, the putting x = h, v = 0 in (v), we get
V2 V 2 + U2 
h= log  
2g  V2  ...(vi)
 
cc

Again, we may write equation (iv) as


dv g  dv dv 
=− (V 2 + v 2 ) ∵ =v 
dt 2  dt dx 
V
dv g
= dt
Su

or ...(vii)
V 2 + v2 V 2
Let t1 be the time taken by the particle in reaching the highest point where is velocity become
zero. Then by integrating (vii) between the limits t = 0 to t = t1 and v = U to v = 0, we get
0
1 −1 v  g t1
V tan V  = − 2 [t ]0
 U V
1 U gt V U
or − tan −1 = − 1 or t1 = tan −1 ...(viii)
V V 2 g V
V
Now, after reaching the highest point, the particle stars moving downwards from rest. Therefore,
its equation of motion will be (ii), and it may written as
dv  k  dv  v2 
v = g 1 − v 2  or v = g 1 − 
dy  g  dy  V2 
 
dv g
or v = (V 2 − v 2 ) ...(ix)
dy V 2
v dv g
or =− dy
2 2
(V − v ) V2
SuccessClap: Best Coaching for UPSC Mathematics : For Info- 9346856874
Checkout ->22 Weeks Study Plan, Videos, Question Bank Solutions, Test Series

764 Dynamics
Integrating, we get
2 gy
log (V 2 − v 2 ) = − + B,
V2
where B is constant of integration.
Initially, at highest point y = 0, v = 0
∴ B = log V 2
2 gy
Hence, log (V 2 − v 2 ) = − + log V 2
V2
2 gy V2
or = log
V2 V 2 − v2
V2  V2 
y= log  
or 2g  V 2 − v2

 
Let v1 be the velocity of particle when it return back the point of projection, then y = h, v = v1 ,

ap
hence from (x).
V2  V2 
h= log  
2g  V 2 − v2 
 1 
V2 V 2 + u2 V 2 V2

l
or log = log [from (vi)]
2g
sC V2
V2 +U2
2g V 2 − v12
V2
or log = log
V2 V 2 − v12
2 2 2
V +U V
es
or =
2 2
V V − v12
or V 4 = V 4 − V 2 v12 + U 2V 2 − U 2 v12
U 2V 2 UV
cc

v12 = or v1 =
or 2 2 ...(xi)
U +V (U 2 + V 2 )
Equation (ix) may be written as
dv g
= (V 2 − v 2 )
Su

dt V 2
V2 dv
or dt = ...(xii)
g V 2 − v2
Let t2 be the time taken by the particle in reaching the point of projection, then integrating (xii)
between the limits t = 0 to t = t1 and v = 0 to v = v1 , we get
V2 dv
[t ]t02 = ∫gv1 2 2
g V −v
v
V2 1  v 1 V  v 
or t2 = .  tanh −1  =  tanh −1 1 − tanh −1 0 
g V  V 0 g  V 
V v
or t2 = tanh −1 1 ...(xiii)
g V
Adding (viii) and (xiii), the required time will be
V  −1 u v 
t1 + t2 =  tan + tanh −1 1  ...(xiv)
g  V V
SuccessClap: Best Coaching for UPSC Mathematics : For Info- 9346856874
Checkout ->22 Weeks Study Plan, Videos, Question Bank Solutions, Test Series

Motion in a Resisting Medium 765


2. A heavy particle is projected upwards with velocity u in a medium the resistance of which
is gu −2 tan 2 α times the square of the velocity, α being a constant. Show that the particle will
return to the point of projection with velocity u cos α after a time
 cos α 
ug −1 cot α  α + log 
 1 − sin α 
Solution. Let the particle be projected upwards from a point O with velocity u. Let v be the
velocity at a point P distance x from O. Then A
Resistance = gu −2 tan 2 α . v 2
Now, equation of motion when the particle is rising will be
Q
dv
mv = − mg − mgu −2 tan 2 α . v 2
dx
dv  g 
or v = −   (u 2 + v 2 tan 2 α ) ...(i) P
dx  u2 

ap
2
2v tan α dv g
or = − 2 tan 2 α . dx O
2 2 2
u + v tan α u2
Integrating, we get
gx

l
log (u 2 + v 2 tan 2 α) = − tan 2 α .
sC +A
u2
where A is constant of integration.
Initially, x = 0, v = u
∴ A = log (u 2 + u 2 tan 2 α) = log u 2 sec2 α
es
 2 gx 
log (u 2 + v 2 tan 2 α) = −  tan 2 α + log u 2 sec 2 α
Hence, 2 
 u 
 2 gx  2 2 2 2 2 2
or  2  tan α = log (u sec α ) − log (u + v tan α ) ...(ii)
cc

 u 
Let h be the maximum height attained by the particle. Then v = 0, x = h.
Hence, from (ii), we get
 2 gh  2 2 2 2
 2  tan α = log (u sec α ) − log u
Su

 u 
 u2 
h =   cot 2 α . log (sec2 α)
or  2g  ...(iii)
 
Equation (i) may be written as
dv g 2
=− (u + v 2 tan 2 α)
dt 2
u
u2 dv
or dt = − . ...(iv)
g (u + v 2 tan 2 α)
2

Let t1 be the time taken by the particle to reach the highest point B.
Integrating (iv) between the limits t = 0 to t = t1 and v = u to v = 0, we get
u
u2
 1 v tan α 
[t ]t01 = −  . tan −1 
 u tan α
g u 0
u uα
or t1 = tan −1 (tan α) = ...(v)
g tan α g tan α
SuccessClap: Best Coaching for UPSC Mathematics : For Info- 9346856874
Checkout ->22 Weeks Study Plan, Videos, Question Bank Solutions, Test Series

766 Dynamics
From the highest point, particle starts moving downwards from rest. Let v be the velocity at a
point Q distant y from B. Equation of motion in downwards direction will be
dv
mv = mg − mgu −2 tan 2 α v 2
dy
dv g 2
or v = (u − v 2 tan 2 α) ...(vi)
dy u 2
2v dv 2g
or − = dy
u − v tan α u 2
2 2 2

2 gy
Integrating, cot 2 α log (u 2 − v 2 tan 2 α) = − + B,
u2
where B is constant of integration.
Initially, when y = 0, v = 0.
∴ B = cot 2 α log u 2

ap
2 gy
∴ cot 2 α log (u 2 − v 2 tan 2 α) = − + cot 2 α . log u 2
2
u
2 gy 2 u2
or = cot α log ...(vii)
u2 u 2 − v 2 tan 2 α

l
Let v1 be the velocity of return to the point of projection A, then putting y = h, v = v1 in (vii),
we get
2hg
sC u2
= cot 2 α log
u2 u 2 − v12 tan 2 α
u2
es
or log sec 2 α = log [putting for h from (iii)]
u − v12 tan 2 α
2

u2
or sec2 α = or u 2 sec2 α − v12 tan 2 α sec 2 α = u 2
u 2 − v12 tan 2 α
cc

or u 2 − v12 tan 2 α = cos 2 α . u 2 or v12 tan 2 α = u 2 sin 2 α or v12 = u 2 cos 2 α


or v1 = u cos α ...(viii)
This is the velocity when particle comes back to the point of projection.
Su

Equation (vi) may be written as


dv g 2
= (u − v 2 tan 2 α)
dt u 2
u2 dv
or dt = . ...(ix)
g u 2 − v 2 tan 2 α
Let t2 be the time taken to reach the point A. Then integrating (ix) between the limits t = 0 to
t = t2 and v = 0 to v = u cos α, we get
t u 2 u cos α dv
[t ]02 = ∫
g 0 u − u 2 tan 2 α
2

u cos α
u2 1  u + v tan α 
or t2 = .  log 
g 2u tan α  u − v tan α  0
u u + u sin α u  1 + sin α 
= log = log  
2 g tan α u − u sin α 2 g tan α  1 − sin α 
SuccessClap: Best Coaching for UPSC Mathematics : For Info- 9346856874
Checkout ->22 Weeks Study Plan, Videos, Question Bank Solutions, Test Series

Motion in a Resisting Medium 767


u 1 − sin 2 α u cos 2 α
= log = log
2 g tan α (1 − sin α)2 2 g tan α (1 − sin α)2
u  cos α 
= log   ...(x)
g tan α  1 − sin α 
∴ Required time = t1 + t2
u  cos α 
=  α + log  [from (v) and (x)]
g tan α  1 − sin α 
3. Two particles move in a medium whose resistance varies as square of the velocity. One is
let fall from a height h and the other projected upwards at the same instant with initial velocity
sufficient to carry it to a height h. Show that the particles meet at a depth y below the highest
point given by cosh β cos (α − β) = 1, where gy = V 2 log cosh β and gh = V 2 sec α, V being the
terminal velocity.
O
Solution.Let the particles meet at P after time t at a depth y below O.

ap
y
Then from (vi) of 5.3, we get
2
V g P
y= log cosh t
g V
gy = V 2 log cosh β

l
∴ ...(i)
h–y
where β = t.
V
g
sC
Again, if a particle be projected upwards with velocity u, A
then from (iv) of 5.3.1, we have
V 2 + u2
es
2g
x = log ...(ii)
V2 V 2 + v2
If h be the greatest height, then v = 0 and x = h.
2g  V 2 + u2   u2 
cc

h =   = log  1 + 

V2  V2   V2  ...(iii)
   
u
Let = tan α, then
V
Su

2g
h = log (1 + tan 2 α) = log sec2 α = 2 log sec α
V2
∴ gh = V 2 log sec α ...(iv)
Let t be the time when the velocity is v at P where x = OP = h − y, then, since this t is same as
in (i) as the particle is projected at the same time when the other falls from (v) of 5.3.1, we have
V  u v
t =  tan −1 − tan −1  ...(v)
g V V
2g V 2 + u2
and (h − y ) = log by (ii)
V2 V 2 + v2
2 gh 2 g  u2   v2 
− y = log 1 +  − log 1 + 
∴ V V2  V2   V2 
   
2g  v2 
y = log 1 + ,
or
V2  V2  by (iii)
 
SuccessClap: Best Coaching for UPSC Mathematics : For Info- 9346856874
Checkout ->22 Weeks Study Plan, Videos, Question Bank Solutions, Test Series

768 Dynamics
 v2 
2 log cosh β = log 1 + ,
or  V2  by (i)
 
v2 v2
or cosh 2 β = 1 + or cosh 2 β − 1 =
V2 V2
v
or sinh β = ...(vi)
V
Now from (v), putting the values of (i) and (vi), we get
β = tan −1 tan α − tan −1 sinh β
or tan −1 (sinh β) = (α − β)
∴ sinh β = tan (α − β)
1
cos (α − β) =
∴ [1 + tan 2 (α − β)]

ap
1 1
= =
2
(1 + sinh β) cosh β
∴ cosh β cos (α − β) = 1 ...(vii)
Results (i) and (vii) prove the results.

l
4. A particle is projected wtih velocity V along a smooth horizontal plane in a medium whose
sC
resistance per unit mass is µ times the cube of the velocity. Show that the distance it has described
V
1
in time t is [ (1 + 2µtV 2 ) − 1] and that its velocity is .
µV (1 + 2µtV 2 )
Solution. In this case, since the particle is moving in a horizontal plane, hence its weight mg
es
will not act. Hence, the only force acting on the particle will be that due to resistance and equal to
− mµv 2 .
The equation of motion of the particle will be
cc

dv  dv 
m = − mµv 2 or −   = µ dt
dt  v2 
Integrating, we get
1
= µt + A, where A is constant of integration.
Su

2v 2
1
Initially, when t = 0, v = V , ∴ A =
2V 2
1 1 1 2µtV 2 + 1
∴ = µt + or =
2v 2 2V 2 v2 V2
V
v=
or ...(i)
(1 + 2µtV 2 )
Let x be the distance traversed by the particle in time t. Then from equation (i), we have
dx V Vdt
= or dx =
dt 2
(1 + 2µtV ) (1 + 2µtV 2 )
integrating :
V (1 + 2µtV 2 )1/ 2
x= + B,
1
2µV 2 ×
2
where B is constant of integration
SuccessClap: Best Coaching for UPSC Mathematics : For Info- 9346856874
Checkout ->22 Weeks Study Plan, Videos, Question Bank Solutions, Test Series

Motion in a Resisting Medium 769


1
or x= (1 + 2µtV 2 ) + B ...(ii)
µV
1
Initially, when t = 0, x = 0 ∴ B = −
µV
Hence, equation (ii) becomes
1 1
x= (1 + 2µtV 2 ) −
µV µV
1 2
or x= [ (1 + 2µtV ) − 1] ...(iii)
µV
5. A particle of mass m is projected vertically under gravity; the resistance of the air being
mk times the velocity. Show that the greatest height attained by the particle is
V 2 
  [λ − log (1 − λ)]
 g 
 

ap
where V is the terminal velocity of the particle and λV is the initial velocity.
Solution. In this case the particle is projected under gravity and hence the equation of motion
will be
dv
mv = − mg − mkv
dx

l
dy  k 
= − 1 + v 
or
sC v
dx  g 
To find the terminal velocity V, the equation of motion in the downwards direction will be
...(i)

 dv 
mv   = mg − mkv ...(ii)
 dx 
es
We know that the terminal velocity of the particle is that velocity for which its downwards
acceleration is zero. Hence, if V be the terminal velocity, then from (ii), we have
g
mg − mkV = 0 or V = ...(iii)
k
cc

Hence, (i) becomes


dv  v g
v = − g 1 +  = − (V + v)
dx  V V
vdV g (v + V ) − V g
= − dx or dv = − dx
Su

or V +v V v +V V
 V  g
or 1 −  dv = − dx
 v +V  V
Integrating, we get
gx
v − V log (V + v ) = − +A ...(iv)
V
Initially, when x = 0, v = λ V
∴ A = λV − V log (V + λV )
Substituting this value of A in (iv), we get
gx
v − V log (V + v ) = − + λV − V log (V + λV )
V
gx
or = λV − V log {V (1 + λ )} − v + V log (V + v)
V
= λV − V log V − V log (1 + λ) − v + V log (V + v) ...(v)
Let h be the greatest height attained by the particle. Then at x = h, v = 0.
SuccessClap: Best Coaching for UPSC Mathematics : For Info- 9346856874
Checkout ->22 Weeks Study Plan, Videos, Question Bank Solutions, Test Series

770 Dynamics
Hence, from equation (v), we get
gh
= λV − V log V − V log (1 + λ ) − 0 + V log V
V
gh
or = λV − V log (1 + λ )
V
V 2 
h=  [λ − log (1 + λ)].
or  g 
 
EXERCISES
1. A particle is projected vertically upwards with velocity V and the resistance of the air pro-
duces a retardation kv 2 , where v is the velocity. Show that the velocity V ′ with which the
particle returns to the point of projection is given by
1 1 k
= +
2 2 g

ap
V′ V
2. A particle falls from rest in a medium in which the resistance is kv 2 per unit mass. Prove
that the distance fallen in time t is
1

l
  log cosh {t ( gk )}
k
sC
If the particle were ascending, show that at any instant the distance from the highest point
of its path is
1
  log sec {t ( gk )},
k
es
where t now denotes the time it will take to reach its highest point.
3. A heavy particle is projected upwards in a medium the resistance of which varies as the
square of the velocity. It has a kinetic energy K in its upwards path at a given point, when it
K2
cc

passes through the same point on the way down, show that its loss of energy is ,
K + K′
where K ′ is the limit to which energy approaches in its downwards course.
4. A particle moves from rest at a distance a from a point O under the action of a force to O
equal to µ times the distance per unit of mass; if the resisatnce of the medium in which it
Su

moves be k times the square of the velocity per unit mass, show that the square of the
velocity, when it is at a distance x from O, is
µx µa 2k ( x − a ) µ
− e + [1 − e2k ( x − a ) ]
k k 2
2k
Show also that when it first comes to rest it will be at a distance b given by
(1 − 2bk ) e2bk = (1 + 2ak ) e−2ak
5. An attracting force varying as the distance acts on a particle initially at rest at a distance a.
Show that if V be the velocity when the particle is at a distance x and v ′ the velocity when
 k (2a + x ) ( a − x) 
the resistance of air is taken into account, then v ′ = V 1 −  nearly, the
 3 (a + x) 
resistance being k times the square of the velocity, k being very small.
6. A particle moving in a straight line is subjected to a resistance kv3 , where v is the velocity..
Show that if v is the velocity at time t, when the distance is s,
u s 1
v= ; t =   + ks 2
1 + uks u 2
SuccessClap: Best Coaching for UPSC Mathematics : For Info- 9346856874
Checkout ->22 Weeks Study Plan, Videos, Question Bank Solutions, Test Series

Motion in a Resisting Medium 771


7. A particle of mass m is falling under the influence of gravity through a medium whose resis-
tance equals µ times the velocity. If the particle were released from rest, show that the dis-
tance fallen through in time t is
gm 2  − (µ / m) t µt 
e + − 1
2  m 
µ
8. If the resistance vary as the fourth power of the velocity, the energy of m lbs. at a depth x
 (mgx) 
below the highest point when moving in a vertical line under gravity will be E tan  
 E 
 mgx 
when rising and E tan h   when falling, where E is the terminal energy in the medium.
 E 
9. A heavy particle is projected vertically upwards with a velocity U in a medium, the resis-
tance of which varies as the cube of the particle’s velocity. Determine the height to which
the particle will ascend.
 V2 U 2 + V 2 − UV πV 2 3 V 2 3  3U − V  

ap
 Ans. h = log + + tan −1  
 6g (U + V )2 18 g 3g  V 3  
10. A heavy particle is projected in a resisting medium, the resistance varying as velocity. If v1
and v2 are its velocities at any point in its upward and downwards path and t the interval

l
between its passage through this point; prove that
sC
v1 + v2 = gt , V − v2 = (V + v1 ) . e gt / V
where V is the terminal velocity.
11. A particle falls from rest under gravity through a distance x in a medium whose resistance
varies as square of the velocity. If v be the velocity actually acquired by it. v0 the velocity it
es
would have acquired, had there been no resisting medium and V the terminal velocity, show
that
v2 1 v02 1 v04 1 v06
=1− + − + ...
v02 2 V 2 2.3 V 4 2.3.4 V 6
cc

12. A particle of mass m falls from rest at a distance a from the centre of the earth, the motion
meeting with a small resistance proportional to the square of the velocity v and the retarda-
tion being µ for unit velocity, show that the kinetic energy at a distance x from the centre is
1 1  x  a 
Su

mgr 2  − + 2µ  1 −  − 2µ log    the square of µ being neglected and r the ra-


x a  a  x 
dius of the earth.
13. A particle is projected in a resisting medium whose resistance varies as (velocity)n and it
comes to rest after describing a distance h in time t, find the values of h and t1 and show
that h is finite to n < 2, and infinite if n = or > 2, whilst t is finite if n < 1, but infinite if
n = or > 1.
5.6 MOTION OF PROJECTILES IN A RESISTING MEDIUM
A particle is projected under gravity and a resistance equal to Y
mk (velocity) with a velocity u at an angle α to the horizon; to
)

discuss the motion.


1
y
1,

ψ
(x

Let the particle be projected from O with velocity u at an angle α


kv
m

to the horizon. Let O be the origin. Let P ( x, y ) be its position after a


time t. The forces acting on the particle at P will be : u
mg
(i) weight of the particle mg, acting vertically downwards, and
X
(ii) the resistance force mkv along the tangent in the direction PT,
SuccessClap: Best Coaching for UPSC Mathematics : For Info- 9346856874
Checkout ->22 Weeks Study Plan, Videos, Question Bank Solutions, Test Series

772 Dynamics
d2x d2y
We know that acceleration along x and y axes are and respectively. Hence, equations
dt 2 dt 2
of motions along coordinate axes will be
d2x
m = − mkv cos ψ
dt 2
d2x ds dx  dx 
or =−k . ∵ cos ψ = 
dt 2 dt ds  ds 
d2x dx
or =−k
dt 2 dt
d 2 x / dt 2
or =−k ...(i)
dx / dt
d2y
and m = mkv sin ψ − mg
dt 2

ap
d2y ds dy
or =−k. . − g, [∵ sin ψ = dy / ds]
2 dt ds
dt
d2y dy
or =−k −g ...(ii)
2

l
dt dt
Integrating (i), we get
sC  dx 
log   = − kt + A,
 dt 
where A is constant of integration.
Initially, when t = 0; dx = initial horizontal component of velocity = u cos α
es
dt
∴ A = log (u cos α)
 dx 
∴ log   = − kt + log (u cos α)
 dt 
cc

 dx / dt 
or log   = − kt
 u cos α 
dx
= u cos α . e − kt
Su

or ...(iii)
dt
we can write equation (ii) as
 k d 2 y / dt 2 
 =−k
 k dy / dt + g 
Integrating, we get
 dv 
log  k + g  = − kt + B,
 dt 
where B is constant of integration.
dy
Initialy, when t = 0, = u sin α
dt
∴ B = log (ku sin α + g )
 kdy / dt + g 
∴ log   = − kt
 ku sin α + g 
dy
or k + g = (ku sin α + g ) e − kt ...(iv)
dt
SuccessClap: Best Coaching for UPSC Mathematics : For Info- 9346856874
Checkout ->22 Weeks Study Plan, Videos, Question Bank Solutions, Test Series

Motion in a Resisting Medium 773


Equations (iii) and (iv) give the horizontal and vertical components of velocity of the particle
at any time t.
Now, we can write equation (iii) as
dx = u cos α . e− kt dt
u
Integrating, x = − cos α e − kt + C ,
k
where C is constant of integration.
Initially, at O, x = 0, t = 0,
u
∴ C = cos α
k
u
∴ x = cos α (1 − e − kt ) ...(v)
k
Equation (iv) may be written as
kdy + gdt = (ku sin α + g ) e− kt dt
Integrating, we get

ap
1
ky + gt = − (ku sin α + g ) e − kt + D,
k
where D is constant of integration,
Initially, at 0, y = 0, t = 0

l
1

sC
D = (ku sin α + g )
k
g + ku sin α
∴ ky + gt = (1 − e − kt ) ...(vi)
k
Equations (v) and (vi) represent the horizontal and vertical distances travelled by the particle
es
in time t. These equations are called the parametric equations of the trajectory.
To obtain Cartesian equation, we will eliminate t between these equations. From (v)
kx kx
1 − e − kt = , ∴ e − kt = 1 −
u cos α u cos α
cc

1  kx 
or t = − log  1 − 
k  u cos α 
Substituting these values of t and e − kt in (vi), we get
Su

g  kx  g + ku sin α kx
ky − log  1 − = .
k  u cos α  k u cos α
g  kx  x
or y= log  1 − + ( g + ku sin α ) ...(vii)
2 α α
k  u cos  ku cos
This is the Cartesian equation of path of the particle.
Deductions
(a) Range on the horizontal Plane
Let the required range be R. The coordinates of the point where the particle strikes the horizontal
plane are (R, 0). This point (R, 0) must satisfy the equation (vii) of trajectory. Thus,
g  kR  R
0= log  1 − + ( g + ku sin α ) ...(viii)
2 α α
k  u cos  ku cos
Approximate value of R, if k is small.
From equation (viii), we get
g  kR k 2 R2 k 3 R3  R
0 = 2 − − 2 − − ... + ( g + ku sin α)
k  u cos α 2u cos α 3u cos α
2 3 3
 ku cos α
SuccessClap: Best Coaching for UPSC Mathematics : For Info- 9346856874
Checkout ->22 Weeks Study Plan, Videos, Question Bank Solutions, Test Series

774 Dynamics

Neglecting K 3 , ... etc., we have


kRg k 2 R2 g
−g− − + g + ku sin α = 0
2u cos α 3u 2 cos 2 α
kRg k 2 R2 g
or − − + ku sin α = 0
2u cos 2 α 3u 2 cos 2 α
2u 2 sin α cos α 2kR 2
or R= − ...(ix)
g 3u cos α
2u 2 sin α cos α
∴ R= for first approximation
g
Substituting this value of R in right hand side of (ix), we get
2
2u 2 sin α cos α  2u sin α cos α 
2
2k
R= −  
g 3u cos α  g 

ap
2 3 2
2u sin α cos α 8 ku cos α sin α
= − ...(x)
g 3 g2
This is value of R up to second apporoximation.

l
(b) Time of Fight sC
Let T be the time of flight, i.e., the time required to strike the horizontal plane through the point
of projection. Thus, during the time T the particle will describe zero vertical distance. Hence, putting
y = 0, t = T in equation (vi), we get y = 0, t = T
g + ku sin α
gT = (1 − e − kT )
es
k
1   k 2T 2 k 3T 3  
or gT = ( g + ku sin α )  1 −  1 − kT + − + ... 
k   2! 3!  
cc

 kT 2 2
k T 3 
or gT = ( g + ku sin α) T − + + ...
 2 6 
1 1 1
or gT = gT + kTu sin α − gkT 2 − k 2T 2 u sin α + gk 2T 3 + ...
Su

2 2 6
1 1 2
or 0 = kt [2u sin α − gT − kT u sin α + gkT + ...]
2 3
2u sin α k gT  2 
T= +  − Tu sin α 
or g g  3 

...(xi)

[neglecting k 2 , k 3 etc.]
2u sin α
∴ T= , upto first approximation.
g
Substituting this value of T in right hand side of equation (xi), we have the value of T up to
second approximation as
2u sin α k  g 4u 2 sin 2 α 2u sin α 
T= +  . − . u sin α 
g g  3 2 g
g 
2u sin α 2 k 2 sin 2 α
= − . . ...(xii)
g 3 g2
SuccessClap: Best Coaching for UPSC Mathematics : For Info- 9346856874
Checkout ->22 Weeks Study Plan, Videos, Question Bank Solutions, Test Series

Motion in a Resisting Medium 775


(c) Greatest height and time to reach this height
Let h be greatest height attained by the particle and t1 the time for the same. At the highest
dy
point velocity will be horizontal and hence the vertical component of velocity, i.e., will be zero.
dt
dy
Hence, by putting = 0 and t = t1 in equation (iv), we get
dt
g = (ku sin α + g ) e − kt1
1
or ekt1 = (ku sin α + g )
g
1 k 
∴ t1 = log  u sin α  ...(xiii)
k g 
Now, let us put y = h, t = t1 in (vi), we get
g + ku sin α
kh + gt1 = (1 − e − kt )

ap
k
g  k  g + ku sin α 1
or kh + log  1 + u sin α  = − g [by (xiii)]
k  g  k k
1 g  k 

l
or h= u sin α − log  k + u sin α  ...(xiv)
2
k  g 
sC k
(d) Time to greatest height is less than half the time of flight.
1
Now, we want to prove that t1 < T , i.e., 2t1 − T < 0.
2
   2u sin α 2 ku 2 sin 2 α 
es
2 k
Now, 2t1 − T = log  1 + u sin α  −  − 
k  g   g 3 g2 

[from (xii) and (xiii)]
2 k 1 h2 2   2u sin α 2 ku 2
sin 2 α 
=  u sin α − u sin 2 α  −  − 
cc

k  g 2 g2  g 3 g2 
  
ku 2 sin 2 α 2ku 2 sin 2 α
=− − < 0.
g2 3g 2
Su

5.6 MOTION OF A PROJECTILE IN A RESISTING MEDIUM, RESISTANCE


VARYING AS SQUARE OF VELOCITY
Let P be the position of particle at time t and u b the horizontal component of velocity, i.e.,
u = v cos ψ
Equations of motion in normal and horizontal directions are Y υ
2
v
= g cos ψ ...(i)
ρ
du 2 u
= − kv 2 cos ψ ...(ii) kυ
dt
ds u υ2/ρ
mg
But ρ = − ,
dψ ψ
X
negative sign is because ψ is decreasing when s increasing T O

∴ g cos ψ = − v 2
ds
ds d ψ dψ
=−v. . =−v ...(iii)
dt ds dt
SuccessClap: Best Coaching for UPSC Mathematics : For Info- 9346856874
Checkout ->22 Weeks Study Plan, Videos, Question Bank Solutions, Test Series

776 Dynamics
du d ψ
From (ii), . = − kv 2 cos ψ
d ψ dt
du k
or = sec3 ψ d ψ [∵ u = v cos ψ]
u3 g
Integrating, we get
1 k
− = [sec ψ tan ψ + log (sec ψ + tan ψ )] + A ...(iv)
2 2g
2u
Initially, u = u0 and ψ = α
1 k
∴ A=− =− [sec α tan α + log (sec α + tan α)]
2u02 2g
1 k 1 k  sec α + tan α 
= +
[sec α tan α − sec ψ tan ψ] + log   ...(v)
∴ 2
u02
u 2g 2g  sec ψ + tan ψ 
du ds
Also, = − kv 2 cos ψ = − kvu = − ku

ap
dt dt
du
∴ = − k ds
u
Integrating,

l
u = u0 e− ks [∵ u = u0 , s = 0] ...(vi)
Putting this value in (v), we have
u2k
sC u2k  sec α + tan α 
e2ks = 1 + 0 [sec α tan α − sec ψ tan ψ] + 0 log   ...(vii)
2g 2g  sec ψ + tan ψ 
This is kinetic equation of path of projectile.
es
Remarks : (i) From (v) it is clear that as s → ∞, u → 0 i.e., the particle moves vertically
ultimately.
The terminal velocity V = ( g / k )
(ii) From equation (iv) we have
cc

sec3 ψ 1
= − k [tan ψ sec ψ + log (sec ψ + tan ψ )] + ...(viii)
ρ ρ0
 v2 u 2 sec2 ψ 
∵ ρ = sec ψ = 
Su

 g g 
If ρ = ρ0 , ψ = 0 at the highest point.
5.7 TRAJECTORY IN A RESISTING MEDIUM WHEN RESISTANCE VARIES
AS (VELOCITY) n
If a particle describes a trajectory under gravity in a resisting medium whose resistance is
equal to mk (velcoity) n , to find the motion.
Let the particle be projected from O. Let P be the position of particle at any time t. Let at P, v be
its velocity and u the horizontal component of velocity, so that
u = v cos ψ ...(i)
Equations of motion along the normal and parallel to x-axis are
mv 2
= mg cos ψ
ρ

v2
or = g cos ψ ...(ii)
ρ
SuccessClap: Best Coaching for UPSC Mathematics : For Info- 9346856874
Checkout ->22 Weeks Study Plan, Videos, Question Bank Solutions, Test Series

Motion in a Resisting Medium 777


du Y
and m = − mkv n cos ψ
dt
du ψ
= − kv n cos ψ

2
or ...(iii) u


dt

m
ψ
ds
We have ρ = − because ψ decreases as s increases. mg
n

Putting this value of ρ in (ii), we get T O
X

2  dψ  ds d ψ
v −  = g cos ψ or − v . = g cos ψ
 ds  dt ds

or v = − g cos ψ ...(iv)
dt
du d ψ
From (iii), . = − kv n cos ψ
d ψ dt
du  g cos ψ 

ap
n
or .−  = − kv cos ψ [from (vi)]
dψ  v 
du  g cos ψ  k n +1
or −  = (u sec ψ ) [from (i)]
dψ  v  g

l
du nk
−n = secn + 1 ψ d ψ
or
1
u
sC
n −1

nk
g
n +1
Integrating we get
n
=− ∫ sec ψ dψ + A
u g
where A is the constant of integration
es
1 nk n +1
or =− ∫ sec ψ dψ + A
(v cos ψ ) g
This gives velocity v at any position, constant A being determined by initial conditions.
cc

5.8. MOTION ON A SMOOTH CURVE


A bead moves on a smooth wire in a vertical plane under a
R

resistance equal to k (velocity) 2 ; to find the motion.


N

Let P the position of the bead at any time t. Since bead is


Su

P
moving on the wire, hence normal reaction will also act. The forces
2

acting on the bead at P will be



m

(i) weight mg of the bead acting vertically downwards. mg


ψ
(ii) The resistance force mkv 2 along the tangent at P, and X
O
(iii) Normal reaction R of the wrie along PN.
The equations of motion along the tangent and normal at P are
dv dv
mv = mg sin ψ − mkv 2 or v + kv 2 = g sin ψ
ds ds
1 dv 2 dv 2 d ψ
or + dv 2 = g sin ψ or + 2kv 2 = 2 g sin ψ
2 ds d ψ ds
dv 2  ds 
or + 2k ρv 2 = 2 g ρ sin ψ ∵ ρ = d ψ  ...(1)
dψ  
And other equation (along normal) is :
v2
= g cos ψ − R ...(2)
ρ
SuccessClap: Best Coaching for UPSC Mathematics : For Info- 9346856874
Checkout ->22 Weeks Study Plan, Videos, Question Bank Solutions, Test Series

778 Dynamics
If the equation to the curve is given, ρ can be determined. The equation can be integrated
after substituting for ρ.
Particle Case : If the curve is a circle of radius a, then ρ = a. Equation (i) becomes
dv 2
+ 2kav 2 = 2 ga sin ψ ...(3)

This is a linear differential equation. Its integrating factor
= e ∫ 2ak d ψ = e2ak ψ
∴ Solution of (3) is
e2ak ψ
v 2 e2ak ψ = 2 ga ∫ e2ak ψ sin ψ d ψ + C = 2ag [2ak sin ψ − cos ψ ] + C
1 + 4a 2 k 2
2ag
or v2 = [2ak sin ψ − cos ψ ] + Ce ak ψ ...(4)

ap
2 2
1 + 4a k
Equation (4) gives velocity of the bead at any point ψ .

5.9. ORBITS IN A RESISTING MEDIUM (CENTRAL FORCE)

l
(a) Resistance varying as velocity.
sC
Let P be the central force and kv the resistance, then equations of motion along normal and
transverse directions will be
v2 υ
= P sin ψ ...(i)
ρ
es
1 d  2 dθ 
and r  = − kv sin φ ...(ii) υ2/p
r dt  dt  Q
φ
Putting vp = h = r 2 d θ , which is not constant here, we get r
cc

dt
dh O A
= − kvr sin φ = − kvp = − kh.
dt
Integrating, we get
Su

h = h0 e − kt , where h = h0 , when t = 0

v2 v2 h 2 dp  d 2u 
Hence, P= = = = h2u 2 u + 2 
ρ sin φ r dr p 3
p dr  d θ 
. 
dp r
 d 2u 
= h02u 2 e− kt  u + 
 d θ2 
...(iii)

(b) Resistance varying as square of velocity
The equations of motion will be
v2
= P sin φ ’
ρ
1 d  2 dθ  2
and r  = − kv sin φ
r dt  dt 

Putting up = h = r 2 , we get
dt
SuccessClap: Best Coaching for UPSC Mathematics : For Info- 9346856874
Checkout ->22 Weeks Study Plan, Videos, Question Bank Solutions, Test Series

Motion in a Resisting Medium 779


dh ds
= − kv 2 r sin φ = − kv 2 p = − kvh = − kh
dt dt
dh
∴ = − k ds
h
Integrating, h = h0 e − ks , where h = h0 where s = 0.
 d 2u 
Hence, P = h2u 2  u + 
 d θ2 

 d 2u 
= h02u 2 e−2ks  u + 
 d θ2 
...(iv)

(c) If the resistance be R.
The equations of motion wil be
v2 1 d  2 dθ 
= P sin φ, and r  = − R sin φ

ap
ρ r dt  dt 
dr p dr
∴ v 2 = P ρ sin φ = P . r . =Pρ .
dp r dp
dr

l
∴ h 2 = v 2 p 2 = Pp3
sC ...(v)
dp
1 dh 1 dh
Hence, R= =−
r sin φ dt p dt
1 dh ds v dh h dh
es
=− . . =− =
p ds dt p ds p 2 ds
1 dh 2 1 d  3 dr 
=− =
2 ds  
. Pp ...(vi)
2 ds
2p 2p  dp 
cc

EXAMPLES
1. If the resistance vary as the velocity and the range on the horizontal plane through the
point of projection is maximum, show that the angle α which the direction of projection makes
Su

with the vertical is given by


µ (1 + µ cos α)
= log (1 + µ sec α),
µ + cos α
where µ is the ratio of the velocity of projection to the terminal velocity.
Solution.In the present case resistance = kv.
Let the particle be falling under gravity. Hence, downward acceleration = g − kv. When this
acceleration = g − kv. When this acceleration will be zero, the velocity will be the terminal velocity, i.e.,
g
g − kV = 0 or V = ...(i)
k
Equations of motion in horizontal and vertical directions will be
d2x ds dx  dx 
= − kv cos ψ = − k . ∵ cos ψ = 
dt 2 dt ds  ds 
dx
=−k ..(ii)
dt
d2y ds dy
and = − kv sin ψ − g = − k . . − g
2 dt ds
dt
SuccessClap: Best Coaching for UPSC Mathematics : For Info- 9346856874
Checkout ->22 Weeks Study Plan, Videos, Question Bank Solutions, Test Series

780 Dynamics
 dy   dy 
= − k + g ∵ sin ψ = ds  ...(iii)
 dt   
π
In this case the direction of projection makes an angle α with the vertical, i.e., − α with the
2
horizon.
Then as in 5.7, we get
π 
u cos  − α 
x= 2  (1 − e− kt )
k
u sin α
= (1 − e − kt ) ...(iv)
k
π 
g + ku sin  − α 
and ky + gt = 2  (1 − e− kt )
k

ap
g + ku cos α
= (1 − e − kt ) ...(v)
k
Let R be the range on the horizontal plane. The value of x will be the range of R if y = 0.
Hence, from (iv) and (v), R will be

l
u sin α
R= (1 − e − kt ) ...(vi)

where t is given by
sCgt =
k
ku cos α + g
(1 − e − kt ) ...(vii)
k
dR
For R to the maximum = 0. Hence, from (vi), we get
es

dR u cos α u sin α − kt dt
= (1 − e − kt ) + e .k .
dα k k dα
− kt − kt dt
or 0 = cos α (1 − e ) + k sin α e . ...(viii)
cc


Differentiating (vii) w.r.t. α, we get
dt dt
g = (ku cos α + g ) e − kt − u sin α (1 − e − kt )
dα dα
Su

dt
or {(ku cos α + g ) e − kt − g} = u sin (1 − e − kt ) ...(ix)

From equation (viii), we get
dt
k sin α e − kt . = − cos α (1 − e − kt ) ...(x)
dx
Dividing (ix) by (x), we get
(ku cos α + g ) e− kt − g u sin α
=−
− kt cos α
k sin α . e
or (ku cos 2 α + g cos α) e − kt − g cos α = − ku sin 2 α e− kt
g cos α
or e− kt =
ku + g cos α
 ku + g cos α   ku 
∴ e kt =  = + 1
 g cos α   g cos α 
1  ku 
∴ t = log  1 +  ...(xii)
k  g cos α 
SuccessClap: Best Coaching for UPSC Mathematics : For Info- 9346856874
Checkout ->22 Weeks Study Plan, Videos, Question Bank Solutions, Test Series

Motion in a Resisting Medium 781


Substituting the values of t and e − kt in (vii), we get
g  ku   g g cos α 
log 1 +  =  u cos α +  1 − 
k  g cos α   k   ku + g cos α 
g  ku   g ku 
or log  1 +  =  u cos α +   
k  g cos α   k  g cos α + ku 
 
 u   u 
or V log 1 +  = (u cos α + V )  g  [from (i)]
 V cos α   cos α + u 
k 
 
u  1   u 
or V log (1 + µ sec α) = V  cos α + 1   ∵ V = µ 
V   cos α u 
V  

ap
u 
 
 1  µ (1 + µ cos α)
or log (1 + µ sec α) = (µ cos α + 1)  =
 1 cos α + 1  µ + cos α

l
 µ 
sC
2. If a particle is projected at an angle α with a velocity u in a medium whose resistance
varies as square of velocity, then prove that
ρρ′ cos3 ψ cos 2 ψ ′ = ρ02
es
where ρ and ρ′ are the radii of curvature at two points at equal arcural distances from the
vertex and ψ, ψ′ the incinations to the horizon of the tangents at the these points and ρ0 is the
radius of curvature at the vertex.
Solution. From 5.8, equation (v), we have
cc

v2
u = u0 e − ks and = g cos ψ (v = u sec ψ)
ρ
v2 u2 2 u02 −2ks
∴ ρ = sec ψ = sec ψ = e sec3 ψ
Su

g g g
u02 −2ks
∴ ρ cos3 ψ = e
g
u02 −2k ( − s ) u02 2ks
and ρ′ cos3 ψ ′ = e = e
g g
u04 1  u02 
∴ ρρ′ cos3 ψ cos3 ψ ′ = =  Since ρ 0 = , if s = 0 
2 2
g ρ0  g 
3. If the resistance of the air to a particle’s motion is n times the weight and the particle be projected
horizontaly with velocity V, show that its velocity, when moving at an angle ψ to the horizontal is
V (1 − sin ψ )( n − 1) / 2 (1 + sin ψ )− ( n + 1) / 2
Solution. Equations of motion are
du
= − ng cos φ ...(i)
dt
v2
and = g cos ψ ...(ii)
ρ
SuccessClap: Best Coaching for UPSC Mathematics : For Info- 9346856874
Checkout ->22 Weeks Study Plan, Videos, Question Bank Solutions, Test Series

782 Dynamics
ds
and ρ= ...(iii)

as s increases when ψ increase.
Also, u = v cos ψ
du d ψ v2 ds d ψ O A
∴ . = − n = − nv . . ψ
d ψ dt ρ dt ds
S
du −v
or = = − nu sec ψ P
d ψ cos ψ u
Integrating,
log u = − n log (sec ψ + tan ψ) + A,
v 2/ρ v
where A is constant of integration. g
Initially, u = V , ψ = 0, ∴ C = log V
∴ log u = − n log (sec ψ + tan ψ) + log V

ap
V
∴ u=
(sec ψ + tan ψ ) n
V cos −1 ψ V (1 − sin 2 ψ )n − 1/ 2
∴ v = u sec ψ = =
(1 + sin ψ )n (1 + sin ψ )

l
= u (1 − sin ψ ) n − 1/ 2 (1 + sin ψ ) − ( n + 1) / 2
sC
4. A shot is fired in atmosphere in which the resistance varies as the cube of the velocity. if f
be the retardation when the shot is ascending at an inclination α to the horizon, f0 when it is
moving horizontally and f ′ when it is descending at an inclination α to the horizon, prove that
es
1 1 2 cos 2 α 1 1 2 sin α
+ = and − = (3 − 2 sin 2 α)
f f′ f0 f′ f g
Solution. Let u be the horizontal component of velocity v at the point where tangent to this
point makes an angle ψ with the horizontal. Thus,
cc

u = v cos ψ ...(i)
Now, when the shot is ascending at an inclination α, then resistance is
kv3 = f
Su

Hence, from (i), we get


f 
u 2 =   cos3 ψ ...(ii)
k
Equations of motion along the normal and parallel to x-axis will be
v2
= g cos ψ ...(iii)
ρ
du
and = − kv3 cos ψ ...(iv)
dt
ds
Also, ρ=− ,

since in this case ψ decreases as s increases.
From (v) and (iii), we get
or  dψ  ds d ψ
v2  −  = − g cos ψ or v . . = − g cos ψ
 ds  dt ds

or v = − g cos ψ ...(vi)
dt
From equation (iv), we get
SuccessClap: Best Coaching for UPSC Mathematics : For Info- 9346856874
Checkout ->22 Weeks Study Plan, Videos, Question Bank Solutions, Test Series

Motion in a Resisting Medium 783


du d ψ du k 4
. = − kv3 cos ψ or = v , from (vi)
d ψ dt dψ g
du k 4
or = u sec4 ψ
dψ g
−3du 3k
or =− sec 4 ψ
4 g
u
Integrating, we get
1 3k
= ∫− sec2 ψ (1 + tan 2 ψ ) d ψ + C
3 g
u
3k  1 3 
=−  tan ψ + tan ψ  + C ,
g  3 
where C is constant of integration.
when ψ = 0, i.e., particle is moving horizontally, let u = u0
1

ap
C=
∴ u03
1 1 3k  1 3 
− =−  tan ψ + tan ψ  ...(vii)
∴ u 3
u0 3 g  3 

l
Also, when particle is moving horizontally,
sC
i.e., ψ = 0, f = f0 , u = u0
Hence from (ii), we get
f
u03 = 0 ...(viii)
k
es
3 3
Substituting values of u and u0 from (ii) and (viii) in (vii), we get
k k 3k  1 3 
− =−  tan ψ + tan ψ 
3 g  
f cos ψ f 0 3
k1 1 sec ψ (3 − 2 sin 2 ψ)
cc

or − .= ...(ix)
f cos3 ψ f 0 g cos3 ψ
Now, when ψ = α, f = f (given), and when ψ = − α, f = f ′ (given). Thus, equation (ix)
provides
Su

1 1 1 sin α (3 − 2 sin 2 α)
= − . ...(x)
f cos3 α f0 g cos3 α
1 1 1 sin α (3 − 2 sin 2 α)
and = + ...(xi)
f ′ cos 2 α f′ g cos3 α
Adding (x) and (xi), we get
1 1 2 cos3 α
+ =
f f′ f0
Subtracting (x) from (xi), we get
1 1 2 sin α (3 − 2 sin 2 α)
− =
f′ f g
5. If a particle of mass m be acted upon by equal constant forces mf tangentially and normally
v2
to the path and if the resistance be mf 2 , prove that intrinsic equation of the path is
k
2 2 fs / k 2 2 2ψ
k (e − 1) = u (e − 1), where u is the velocity of projection.
SuccessClap: Best Coaching for UPSC Mathematics : For Info- 9346856874
Checkout ->22 Weeks Study Plan, Videos, Question Bank Solutions, Test Series

784 Dynamics
Solution. Since the particle is not projected in a vertical plane T
O ψ
hence weight of the particle will not be considered. u
mf (υ2/k2)
The equations of motion of the particle along the tangent and
normal are
dv v2
mv = mf − mf
ds k2 mf
2
mv
and = mf
ρ
1 dv 2 v2
or + f = f ...(i)
2 ds k2
v2 ds
and =ρ= ...(ii)
f dψ
As ψ increases when s increases. Hence ρ = + ds .

ap

2
From (i), dv 2f 2
+ v = 2f.
ds k2
2
This is linear equation whose integrating factor = e2 fs / k . Hence, its solution will be

l
2 2
sC
v 2 .e2 fs / k = ∫ 2 f .e2 fs / k .ds + A.
2 2
+ A,
or v 2 .e2 fs / k = k 2 .e2 fs / k
where A is the constant of integration.
Initially, s = 0, v = u , ∴ A = u 2 − k 2 .
es
2
∴ v 2 = k − (u 2 − k 2 ) e−2 fs / k .
Hence, by putting the value of v 2 in equation (ii), we get
k 2 u 2 − k 2 −2 fs / k 2 ds
cc

+ e =
f f dψ
2 ds k 2 2 fs / k 2 u 2 − k 2
or e2 fs / k . − e =
dψ f f
Su

k 2 2 fs / k 2 2 ds dz
Put e = z, ∴ e2 fs / k . =
2f dψ dψ
dz u2 − k 2
∴ − 2z =
dψ f
This is a linear equation whose I.F. = e −2ψ . Hence, its solution will be
u 2 − k 2 −2ψ
z.e−2ψ = ∫ e dψ + B
f
k2 u 2 − k 2 −2ψ
or . 2 fs / k 2 . e− 2ψ = e + B,
2f −2 f
where B is constant of integration.
Initially, when s = 0, ψ = 0.
k2 u2 k 2 u2
∴ =− + + B, ∴ B =
2f 2f 2f 2f
k 2 2 fs / k 2 −2ψ u 2 − k 2 −2ψ u 2
∴ .e .e = e +
2f −2 f 2f
SuccessClap: Best Coaching for UPSC Mathematics : For Info- 9346856874
Checkout ->22 Weeks Study Plan, Videos, Question Bank Solutions, Test Series

Motion in a Resisting Medium 785


2
or k 2 e2 fs / k = − u 2 + k 2 + u 2 e2ψ
2
or k 2 (e2 fs / k − 1) = u 2 (e2ψ − 1)
This is the required intrinsic equation.
EXERCISES
1. A particle is projected with a velocity whose horizontal and vertical components are U and
V from a point in a medium whose resistance per unit of mass is k times the speed. Obtain
the equation of the depth, and prove that if k is small, the horizontal range is approximately
2UY 8UV 2 k
− .
g 3g 2
2. A particle acted on by gravity is projected in a medium, the resistance of which varies as the
velocity. Show that its acceleration retains a fixed direction and diminishes without limit to
zero.
3. In the case of a flat trajectory with initial velocity u and resistance equal to µ (velocity) 2 ,
show that the path of projectile approximately is

ap
gx 2 µg
y = x tan α − − x3 − ...
2 3
2u 3u
4. A heavy particle is projected in a resisting medium. If v be the velocity at any time, φ be

l
inclination to the vertical of the direction of motion and f the retardation prove that
sC
1 dv f
= cos t φ +
v dφ g sin φ
2
5. If the resistance per unit mass is g  v  prove that
 
V 
es
du  g  2 dψ  g 
=− u , =   cos3 ψ
2
ds V  ds  u2 
where u is the horizontal component of velocity.
cc

6. A heavy bead of mass m slides on a smooth wire in the shape of a cycloid, whose axis is
vertical and vertex upwards in a medium whose resistance is mv 2 / 2c and the distances of
starting point from the vertex is c, show that the time of descent to the cusp is
 8a (4a − c) 
Su

  , where 2a is the length of the axis of cycloid.


 gc 
7. A particle moving in resisting medium acted upon by a central force (µ / r n ), if the path be
an equiangular spiral of angle α, whose pole is at the centre of force, show that the resis-
tance is
n − 3 µ cos α
.
3 rn
8. A bead moves on a smooth wire in the form of a circle in a vertical plane under a resistance
{= k (velocity)2 }. Find the velocity of the bead at any point.
9. Prove that by a proper choice of axes the equation of the path of the projectile in a resisting
medium can be put in the form y + ax = b log x.
10. A particle of unit mass is projected with velocity u at an inclination α above the horizon in
a medium whose resistance is k times the velocity. Show that its direction will again make an

1   2ku  
angle α with the horizon after a time  k  log 1 +  g  sin α  .
     
SuccessClap: Best Coaching for UPSC Mathematics : For Info- 9346856874
Checkout ->22 Weeks Study Plan, Videos, Question Bank Solutions, Test Series

786 Dynamics
11. A heavy particle describes a path given by
cos ψ = f (ρ cos ψ) ;
show that the law of resistance is given by
df d
Rv = − g (1 − f 2 ) (vf ),
dv dv
 v2 
when f = f   .
 g 
 
d2y g
12. Prove that in the motion of a projectile in a resisting medium the equation =− is
2
dx u2
satisfied whatever be the law of resistance, u being the horizontal component of velocity
the axes of x and y being horizontal and vertically upwards. If the resistance is constant and
equal to kg, show that the velocity at any point is given by
v (1 − sin ψ ) k = u0 (cos ψ )k −1 ,

ap
ψ being the slope and u0 the velocity at the highest point.

❑❑❑

l
sC
es
cc
Su
SuccessClap: Best Coaching for UPSC Mathematics : For Info- 9346856874
Checkout ->22 Weeks Study Plan, Videos, Question Bank Solutions, Test Series

Motion of Particles of Varying Mass


6.1 MOTION WHEN MASS VARIES
We come across a number of problems in mechanics in which moving mass changes with time.
Obviously, the motion of a body will be affected by the material added or removed. For example, a
rocket ejecting burned fuel in the form of hot gases looses its mass while a rain drop acquires
additional mass by condensations it falls through a cloud. Such type of motion will be discussed ni
this section.
We know that the equation P = mf holds only if the moving mass m remains constant

ap
throughout the motion.
Let the moving mass does not remain constant. Let m be its mass and v the velocity at time t.
Then by Newton’s second law
FG d IJ (mv)
P=
H dt K ...(i)

l
sC
Let there be an increment δm of mass in time δt and let this increment δm be moving with
velocity V.
The increment in the momentum of the particle in time δt
= m ( v + δv − v) + δm ( v + δv − V ) ...(ii)
es
= m δv + δm ( v + δv − V )
The impulse of force in time δt = P . δt ...(iii)
cc

Hence equating (ii) and (iii) and taking limits at δt → 0, we get


FG dv IJ FG IJ
dm
P=m
H dt K H K
+v
dt
− V ( dmdt ) [neglecting δm δv ]
FG IJ
dv FG IJ
dm FG IJ
dm
Su

or m
H K
dt
+v
H K
dt
=P+V
H K dt
FG IJ
d FG IJ
dm
or H K
dt
( vm ) = P + V
H K
dt
...(iv)

If V = 0, the equation (iv) reduces to (i).


Equation (iv) is the equation of motion when mass varies.
EXAMPLES
1. A spherical raindrop, falling freely, receives in each instant an increase of volume equal
to λ times its surface at that instant; find the velocity at the end of time t, and the distance fallen
through in that time.
Solution. Let m be the mass and r the radius of the drop when it has fallen through a distance
x in time t. Also, let v be the velocity at that instant, so that
dx
=v ...(i)
dt
Since drop is falling freely under gravity, hence
P = mg
SuccessClap: Best Coaching for UPSC Mathematics : For Info- 9346856874
Checkout ->22 Weeks Study Plan, Videos, Question Bank Solutions, Test Series

788 Dynamics

FG 4 IJ πr 3ρ
Now, m = mass of drop at time t =
H 3K
dm
= 4 πr 2 ρ
dr FG IJ
∴ dt dt H K
But, given rate of increase of mass
dm
= λ ( 4 πr 2 ) ρ
dt
dm
∴ Equating two values of , we get
dt
4 πr 2 ρ
dr FG IJ
= λρ ( 4 πr 2 ) or
dr FG IJ
dt H K dt

H K
Integrating, r = λt + A
where A is constant of integration.

ap
Initially, at t = 0, r = a (say) ∴ A = a ∴ r = λt + a
4
∴ m = πρ ( a + λt ) 3 [from (iii) and (iv)]
3
Since the mass is picked up from rest, hence V = 0.

l
Hence, from the equation of variable mass
FG IJ FG IJ
H
d
dt
sC
K
( mv) = P + V
dm
dtH K, we get

FG
d IJ 4
[v . 4 / 3 πρ ( a + λt ) 3 ] = πρ ( a + λt ) 3 g
H
dt K 3
[from (ii) and (iv)]
es
FG
d IJ
[ v ( a + λt ) 3 ] = g ( a + λt ) 3
H
dt K
Integrating, we get
v ( a + λt ) 3 = g ( a + λt ) 4 / 4 + B,
cc

where B is constant of integration.


F I
ga 4
Initially, v = 0, t = 0 ; ∴ B=− GH JK

Su

v ( a + λt ) 3
F gI
= G J [( a + λt ) 4
− a4 ]
∴ H 4λ K
FG g IJ ( a + λt ) − a 4 / ( a + λt ) 3
or v=
H 4λ K ...(vi)
This is expression for velocity at any time t.
Now, from (i) and (vi), we get
dx
=
g LM
( a + λt ) −
a4 OP
dt 4λ MN ( a + λt ) 3 PQ
g
Integrating, x= ( a + λt ) 2 / 2λ + a 4 / 2λ ( a + λt ) 2 + C

where C is constant of integrating. Initially, x = 0 at t = 0
g FG IJ
(a 2 + a 2 )
∴ C=−
8x 2 H K
SuccessClap: Best Coaching for UPSC Mathematics : For Info- 9346856874
Checkout ->22 Weeks Study Plan, Videos, Question Bank Solutions, Test Series

Motion of Particle of Varying Mass 789


FG g IJ [(a + λt)2 + a4 / (a + λt)2 − 2a 2 ]
Hence, x =
H 8λ2 K
=G
F g IJ [{(a + λt) − a } / (a + λt)]
2 2
H 8λ K
2

=G
F g IJ [{(a + λt) − a } / (a + λt)]
2 2 2
H 8λ K
2

= [{gt 2 (2 a + λt 2 )} / {8 ( a + λt ) 2}] ...(vii)


This is the required expression for distance.
2. A spherical drop of liquid falling freely in a vapour acquires mass by condensation at a
constant rate c. Show that the velocity after falling from rst in time t is
FG 1 IJ gt LM1 + RS M UVOP
H 2 K MN T ( M + ct) WPQ

ap
where M is the initial mass of the drop.
Solution. Since the drop is falling freely under gravity, hence
P = mg
where m is initial mass of the drop at any time t.

l
Let M be the initial mass of the drop, then
sCm = M + ct ...(i)
Also v = 0, since the mass is picked up from rest.
Hence, equation of motion when mass varies will be
FG d IJ (vm) = mg + V FG dm IJ
es
H dt K H dt K [∵V = 0]

FG d IJ (vm) = mg
In this case, it will becomes
H dt K
FG d IJ {v ( M + ct)} = ( M + ct ) g,
cc

or H dt K [from (i)]

Integrating, v ( M + ct ) = [{g ( M + ct ) 2} / 2c] + A,


Su

where A is constant of integration.


F gM I
2
Initially at t = 0, v = 0; ∴ A=− GH 2c JK
F gI
v ( M + ct ) = G J ( M + ct ) − G
F gM I 2

H 2c JK
2
Hence,
H 2c K
F gI
v = G J ( M + ct ) − MG
LF gM 2 I ( M + ct)OP
or H 2c K MNH 2c JK PQ
F g I L{( M + ct) − M } OP
=G J M
2 2

H 2c K MN ( M + ct ) PQ
F g I L (2mct + c t ) OP = FG gt IJ LM (2 M + ct ) OP
=G J M
2 2

H 2c K MN ( M + ct) PQ H 2 K N ( M + ct) Q
F gt I L{( M + ct) + M} OP = FG gt IJ LM1 + RS M UVOP
=G J M
H 2 K N ( M + ct ) Q H 2 K MN T ( M + ct ) WPQ
SuccessClap: Best Coaching for UPSC Mathematics : For Info- 9346856874
Checkout ->22 Weeks Study Plan, Videos, Question Bank Solutions, Test Series

790 Dynamics
3. Snow slides off a roof clearing away a part of uniform breadth; show that if it all slide at
(2 / 3) !
once, the time in which the roof will be cleared is {6πa / g sin α}
but that, if the top
(1 / 6) !
move first and gradually set the rest in motion, the acceleration is (1/3) g sin α and the time will
be ( 6a / g sin α ) , where α is the inclination of the roof and a the original length of the snow.
Solution. Let y be the length of the snow on the roof at time t and b the constant breadth
dy
∴ m = ybρ ; v = − , f = g sin α
dt
FG d IJ (mv) = ρ, becomes
Hence, equation of motion
H dt K
FG d IJ [− ybρ (dy / dt)] = ybρg sin α
H dt K [∵ρ = mb]

FG d IJ LM y FG dy IJ OP = − gy sin α

ap
or H dt K N H dt K Q
FG d IJ LMy FG dy IJ OP FG dy IJ = − gy sin α
or H dy K N H dt K Q H dt K

l
LMy FG dy IJ OP FG d IJ LMy FG dy IJ OP = − gy 2 sin α
sC
or
N H dt K Q H dy K N H dt K Q
FG 1 IJ LM y FG dy IJ 2 OP = A − 1 gy 3 sin α ,
Integrating, H 2 K MN H dt K PQ 3 ...(i)
es
where A is constant of integration.
dy 1
Initially, at y = a, = 0, ∴ A = ga 3 sin α
dt 3
cc

Putting the value of A in (i), we get


FG IJ FG IJ
1 2 dy
2
1
= ( a 3 − y 3 ) g sin α
HK H K
2
y
dt 3
dy F2 IR
= − G g sin αJ S a 3 − y 3 / y V
U
Su

or
dt H3 KT W
FG 3 g sin αIJ R|S y U|V dy
or dt = −
H2 K |T (a − y ) |W 3 3 ...(ii)

Integrating (ii) from y = a to y = b, the time t in which the roof will be cleared off is given by
F3
t = − G g sin αJ z S
I R| y U|V dy
0
H2 K |T (a − y ) |W
a
3 3

3 1/ 2
Putting y 3/ 2 = a 3/ 2 sin θ, ∴ y dy = a 3/ 2 cos θ dθ
2
FG 3 g sin αIJ z π/2
( a1/ 2 sin 1/ 3 θ .
2 3/ 2
cos θ dθ) / ( a 3/ 2 cos θ)
or t=
H2 K 0
3
a
SuccessClap: Best Coaching for UPSC Mathematics : For Info- 9346856874
Checkout ->22 Weeks Study Plan, Videos, Question Bank Solutions, Test Series

Motion of Particle of Varying Mass 791


FG 6a IJ z π/2 1 sin1/3 θ dθ
=
H g sin α K 0 3
= G
F 6a IJ LM 1 . FG Γ 2 . Γ 1IJ / FG 2 . Γ 7 IJ OP
H g sin α K N 3 H 3 3K H 6 K Q
= G
F 6πa IJ FG Γ 2 / Γ 1 !IJ
H g sin α K H 3 6 K
Let z be the portion of the roof cleared off in time t, then equation of motion will be
FG d IJ FG z dz IJ = gz sin α.
H dt K H dt K
F dz I F d I F dz I
z G J . G J G z J = gz sin α 2
or H dt K H dz K H dt K
1 F dz I
2
G z J = gz sin α + c,
1 3

ap
Integrating,
2 H dt K 3
...(iii)
where c is constant of integration.
dz
Initially, z = 0, = 0, ∴ c = 0
dt

l
sC FG dz IJ 2
2
Hence, from (iii), we get
H dt K =
3
gz sin α ...(iv)
Differentiating (iv) with respect to t, we get
FG IJ FG IJ
dz d 2 z 2 FG IJ
dz d2z 1
2
H
dt KH K 2
= g
3 H K
dt
sin α or
2
=
3
g sin α
es
dt dt
1
Hence, the acceleration is g sin α.
3
FG 3 g sin αIJ dz .
H2 K z
cc

Again, from (iv), we get dt =

Integrating from z = 0 to z = a, the required time t is given by


F3
t = G g sin αJ G
I F a IJ = FG 6a IJ
1/ 2

H2 K H 1 / 2 K H g sin α K
Su

4. A uniform chain of length 1 and mass ml is coiled on the floor and a mass mc is attached to
one end and projected vertically upwards wth velocity ( 2gh). Show that according as the chain
does or does not completely leave the floor, the velocity of the mass finally reaching the floor
1
again, is the velocity due to a fall through a height [2l − c + a 2 / (l + c) 2 ] or a − c, where
3
a 3 = c 2 (c + 3h).
Solution. Let the length of the portion of the chain uncoiled in time t be x. Then the equation
of motion will be
FG d IJ RS (mx + mc) UV = − g (mx + mc)
H dt K T (dx / dt ) W
SuccessClap: Best Coaching for UPSC Mathematics : For Info- 9346856874
Checkout ->22 Weeks Study Plan, Videos, Question Bank Solutions, Test Series

792 Dynamics
FG dx IJ FG d IJ RS( x + c) FG dx IJ UV = − g ( x + c)2
or ( x + c)
H dt K H dx K T H dt K W
1R F dx I U 1
2
S ( x + c) G J V = − g ( x + c) + A , 3
Integrating, we get
2T H dt K W 3 ...(i)

where A is constant of integration.


1 2
Initially, x = 0, dx / dt = ( 2 gh), ∴ A = gc (3h + c)
3
Hence, (i) becomes

( x + c) 2
FG dx IJ 2
2
g [c 2 (3h + c) − ( x + c) 3 ]
H dt K =
3
2
= g {a 3 − ( x + c) 3} ...(ii)
3
where a 3 = c 2 (c + 3h).

ap
Now, we have
Case I. Let the coil does not completely leave the floor when the mass comes to rest at a height
dx
x above the floor. Putting = 0 in (ii), we get
dt

l
0 = a 3 − ( x + c) 3 or x = a − c
sC
Hence, the velocity of the mass on reaching the floor again is that due to fall through a height
a − c.
Case II. Let the initial velocity be sufficient enough so that the coil leaves the floor and mass
does not come to rest. Let V be the velocity of the mass just at the time coil leaves the floor, then
es
dx 2 2 2 3 3
putting x = l, = V in (ii), we get (l + c) V = g {g − (l + c) }.
dt 3
Hence, total height attained by the mass
cc

1 3
= l + V 2 / 2g = l + {a / (l + c) 2 − (l + c)}
3
1
= [2l − c + a 3 / (l + c) 2 ] ...(iii)
Su

3
Hence, the velocity of the mass on reaching the floor again is that due to a fall through a
height given by (iii).
5. A rocket whose total initial mass (fuel + shell) is m0 , ejects fuel at a constant rate cm0
and at a velocity V relative to the case. Show that lowest rate of fuel consumption that will permit
the rocket to rise at once is c = g / V . Assuming this design condition is wet, find the greatest
speed and height reached by the rocket.
Solution. We know that the equation of motion of variable mass is
d FG IJ dm FG IJ
dt H K
( vm ) = P + V1
dt H K ...(i)
where m is mass at time t.
Here P = − mg (since mass is moving in upward direction)
dm
= − cm0 as mass is ejected. ...(ii)
dt
V1 = velocity of ejecting mass = v − V
SuccessClap: Best Coaching for UPSC Mathematics : For Info- 9346856874
Checkout ->22 Weeks Study Plan, Videos, Question Bank Solutions, Test Series

Motion of Particle of Varying Mass 793


Integrating (ii), we have m = − cm0t + A
Initially, m = m0 , t = 0 ∴ A = m0
m = m0 (1 − ct ) ...(iii)
Substituting the values of P and V1 in equation (i), we get
FG IJ
dv FG IJ
dm FG IJ
dm
H K
dt
m+v
H K
dt
= − mg + ( v − V )
H Kdt
FG IJ
dv FG IJ
dm dv
or m
H K
dt
= − mg − V
H K
dt
and
dt
= − g + (Vcm0 ) / m, from (ii) ...(iv)

dv
or = − [− g + {Vc / (1 − ct )}], from (iii) ...(v)
dt
This equation holds for all values of t.
 dv 
Initially, at t = 0, we have   = − g + Vc
dt t = 0

ap
In order, the rocket rise at once dv / dt must be positive.
g
i.e., Vc − g > 0 or c >
V
g
∴ minimum value of c = .

l
V
sC
Now, let us assume that masses are so arranged that the rocket lifts at time zero (c > g / V ),
integrating (v), we get
z
v = 0t [− g + Vc / (1 − ct )] dt = − gt − V log (1 − ct ) ...(vi)
dx
es
∴ = − gt − V log (1 − ct )
dt
Integrating, we get z
x = 0t [− gt − V log (1 − ct ) dt ]
F 1I
x = − G J gt − v [t log (1 − ct ) + c z t {dt / (1 − ct )}]
2 t t
H 2K
cc

or 0 0

F 1I
= − G J gt − Vt log (1 − ct ) + V z [(1 − ct − 1) / (1 − ct )] dt
2 t
H 2K 0

F 1I
Su

= − G J gt − Vt log (1 − ct ) + V z [1 − 1 / (1 − ct )] dt
2 t
H 2K 0

F 1I
= − G J gt − Vt log (1 − ct ) + V [t + (1 / c) log (1 − c)]
2
H 2K
F 1I
= − G J gt + (V / c) (1 − ct ) [log (1 − ct ) − 1] + (V / c)
2
H 2K ...(vii)
Now, let M be the mass of the shell (or case). The rocket will be lightest when all fuel is burnt,
i.e., when m = − M. Putting this value of m in (iii), we get
M = m0 (1 − ct ) or ct = 1 − M / m0 ...(viii)
The speed v will be maximum when all fuel is burnt, i.e., when (viii) holds.
Using this value of t in (vi), we get
FGg IJ LM FG IJ OP
M FG IJ
M
Vmax = −
H
ct K MN H K PQ
1−
m0
− V log
H K
m0 ...(ix)
And at this instant, the height x will be
FG IJ
g
[1 − ( M / m0 )]2 + (V / c) [( M / m0 ) log ( M / m0 ) + 1 (1 − M / m0 )]
x=−
H K
2c 2 ...(x)
SuccessClap: Best Coaching for UPSC Mathematics : For Info- 9346856874
Checkout ->22 Weeks Study Plan, Videos, Question Bank Solutions, Test Series

794 Dynamics
When the shell (rocket) reaches this height, it moves freely as projectile under gravity with
initial vertical velocity vmaximum .
Hence, additional height reached = ( vmax ) 2 / (2 g)
FG IJ
1
g [g 2 c 2 (1 − M / m0 ) 2 + 1 / 2 (log M / m0 ) 2
=
2H K
+ (2 gV / c) (1 − M / m0 ) 2 × log M / m0 ] ...(xi)
Adding (ix) and (x), we get
FG V IJ FG1 − M + log M IJ + FG V 2 IJ FG log M IJ 2
x max =
H c K H m 0 m0 K H 2 g K H m 0 K
EXERCISES
1. A spherical raindrop of radius a cms falls from rest through a vertical height h, receiving

ap
throughout the motion an accumulation of condensed vapour at the rate of k grammes per
square cm. per second, no vertical force but gravity acting. Show that when it reaches the
ground its radius will be

( 2 hg) [1 + ( ga 2 / 2 hk 2 ) ]

l
sC
2. A mass in the form of a solid cylinder, the area of whose cross-section is A, moves parallel
to its axis, being acted on by a constant force F, through a uniform cloud of fine dust of
volume density ρ which is moving in a direction opposite to that of cylinder with constant
velocity V. If all the dust that meets the cylinder clings to it, the cylinder starts from rest and
es
its initial mass was m;show that the velocity after time t is ( mV + Ft / k ) − V and the dis-
tance described in that time is
k m
− Vt − where k = ( m 2 + 2 mAρvt + AFρt 2 ).
cc

Aρ Aρ
3. A particle of mass M is at rest and beings to move under the action of a constant force F in
a fixed direction. It encounters the resistance of a stream of fine dust moving in the opposite
direction with velocity V, which deposits matter on it at a constant rate c. Show that its mass
Su

will be m when it has travelled a distance


FG k IJ [m − M (1 + log m / M )],
Hc K 2 where k = F − cV .

4. A chain of length l is coiled at the edge of a table. One end is fastened to a particle whose
mass is equal to that of the whole chain and the other end is put over the edge. Show that
1
immediately after leaving the table, the particle is moving with velocity (5 / 6) gl .
2
❐❐❐
SuccessClap: Best Coaching for UPSC Mathematics : For Info- 9346856874
Checkout ->22 Weeks Study Plan, Videos, Question Bank Solutions, Test Series

Central Orbits
7.1 DEFINITIONS
(i) Central force : A force which is directed towards a fixed point O is called central force. The
fixed point O is called the centre of force.
(ii) Central Orbit : The path described by the particle moving along a plane curve under a
central force is called the central orbit.
7.2 DIFFERENTIAL EQUATION OF CENTRAL ORBIT (POLAR FORM)

ap
A particle is moving in a plane with an acceleration F which is
always directed towards a fixed point O in the plane, to find the
differential equation of the path. P (r, θ)
Let a particle moves in a plane curve under an acceleration F which
is always directed towards a fixed point O in the plane. Let OX be a

l
fixed line in the plane of orbit. Let O be the pole and OX as the initial
sC
line. Let the position of the particle at time t be P (r, θ).
Since the acceleration F is always directed towards O, hence the
particle has only radial acceleration towards O of magnitude F and no O
transverse acceleration. P N
es
Hence, the equation of motion in the radial and transverse
directions will be
d 2r FG dθ IJ = − F 2

2
−r
H dt K ...(i)
cc

dt
1 d F dθ I
and G r dt JK = 0
r dt H
2
...(ii)
Integrating (ii), we get
Su


r2 = constant = h (say) ...(iii)
dt
To integrate these equations, let
1
r=
u
dr 1 du 1 du dθ
∴ =− . =− . .
2
dt u dt u 2 dθ dt
dr
=−
1 du h
,
RS UV ∴ from (iii)
dθ h
= .
or
dt 2
u dθ r 2
T W dt r 2
dr du
or =− h ...(iv)
dt dθ
d 2r d
=
RS
−h
du
=−h
UV
d du dθ
.
RS UV
and
dt 2 dt T dθ W
dθ dθ dt T W
d 2u h
=−h . , from (iii)
2
dθ r2
SuccessClap: Best Coaching for UPSC Mathematics : For Info- 9346856874
Checkout ->22 Weeks Study Plan, Videos, Question Bank Solutions, Test Series

796 Dynamics
d 2r d 2u
or = − h2u2
dt 2 dθ 2
2
dθ d r
Substituting the values of and from (iii) and (v) in (i), we get
dt dt 2

− h2u2
d 2u
−r
h
2
RS UV
=−F
dθ 2 r2 T W
d 2u
or h2u2 + h2u3 = F ...(v)
dθ 2
d 2u F
or +u= ...(vi)
2 2 2
dθ h u
This is the required differential equation of the central orbit in polar form as it is satisfied by
the coordinates (u, θ) of a point on the orbit.

ap
7.2.1 Differential equation of a central orbit (pedal form)
To obtain differential equation to the path in `p’ and `r’ for a particle moving in a plane with
acceleration F, directed towards a fixed point O.

l
We know that if p be the length of the perpendicular drawn from the pole to the tangent to a
sC
curve at any point p (r, θ) , then
1 RS dr UV2
=
1
+
1
p 2 r 2 r 4 T dθ W
1 R dr U
1 du
S V
es
Putting u = , we have =−
r dθ r T dθ W 2
∴ Above equation reduces to
1 R du U2
= u2 + S V
T dθ W ...(i)
cc

p2
Differentiating (i) with respect to θ, we get
2 dp du du d 2 u
− . = 2u +2 .
p3 dθ dθ d θ d θ2
Su

F d 2u I

1 dp
p 3 dθ GH
=
dθ 2
+u
du
d
JK
θ
F F IJ . FG − 1 . dr IJ , from (vi) of 7.2 above
=G
H h u K H r dθ K
2 2 2

du F 1 I dr or 1 dp = F dr
=−G J
and dθ H r 2 K dθ p 3 dθ h 2 dθ
h2dp
or =F
. ...(ii)
3
p dr
This is the required differential equation of the central orbit in the pedal form.
EXAMPLES
1. Find the law of force to the pole if the path of the particle is cardioid r = a (1 + cos θ).
Solution. It is given that
1 1
u= =
r a (1 + cos θ)
SuccessClap: Best Coaching for UPSC Mathematics : For Info- 9346856874
Checkout ->22 Weeks Study Plan, Videos, Question Bank Solutions, Test Series

Central Orbits 797


or log u = − log a − log (1 + cos θ)
Differentiating with respect to θ,
1 du sin θ θ
. = = tan
u dθ 1 + cos θ 2
du θ
∴ = u tan
dθ 2
2
d u du θ u θ θ u θ
or = tan + sec 2 = u tan 2 + sec 2
2 dθ 2 2 2 2 2 2

d 2u θ u θ 3u θ 3u
∴ u+ = u + u tan 2 + sec 2 = sec 2 =
2 2 2 2 2 2 (1 + cos θ)

= 3au 2
d 2u F
But u+ =
2 2 2

ap
dθ h u
2 4 3ah 2
∴ F = 3 ah u =
r4
1
i.e., F∝ .

l
sC r4
2. A particle describes a circle, pole on its circumference, under a force P to the pole. Find
the law of force.
Solution. The equation of the circle with pole on its circumference is
r = a cos θ
1
es
or = a cos θ ...(i)
u
or 1 = au cos θ
Differentiating both sides with respect to θ, we get
cc

du
0=a cos θ + au ( − sin θ)

du
or = u tan θ

Su

d 2u du
or = u sec 2 θ + tan θ
2 dθ

1 1
= sec 2 θ + sec θ tan 2 θ
a a
1
= sec 3 θ (sec 2 θ + tan 2 θ)
a
1
= sec θ (2 sec 2 θ − 1) ...(iii)
a
Now, the differential equation of the path will be
d 2u P
+u= ,
2 2 2
dθ h u
1 P
or sec θ (2 sec 2 θ − 1) + u = , from (iii)
a 2 2
h u
1 1 P
or sec θ (2 sec 2 θ − 1) + sec θ = , from (i)
a a h u2
2
2 P 2
or sec 2 θ = or P = ( au) 3 . h 2 u 2 , from (i)
a 2 2 a
h u
SuccessClap: Best Coaching for UPSC Mathematics : For Info- 9346856874
Checkout ->22 Weeks Study Plan, Videos, Question Bank Solutions, Test Series

798 Dynamics
2a 2 h 2 1
or P = 2a 2 h 2 u5 = or P∝
5
r r5
3. A particle describes the curve r n = a n cos nθ under a force F to the pole. Find the law of
force.
Solution. The equation of given curve is
r n = a n cos nθ
1
or 1 = a n u n cos nθ, ∵ u=
r
Differentiating both sides w.r.t. θ, we get
LM
0 = a n u n ( − n sin nθ) + nu n − 1
du
cos nθ
OP
N dθ Q
du
or = u tan nθ

Again differentiating w.r.t. θ, we get

ap
d 2u du
= un sec 2 nθ + tan nθ
2 dθ

= un sec 2 nθ + u tan 2 nθ

l
The differential equation of the path is
sCd 2u F
+u=
2 2 2
dθ h u
F
or (un sec θ + u tan 2 nθ) + u =
2
2 2
h u
es
F
or u ( n + 1) sec 2 nθ =
h 2u 2
or F = h 2 u 3 ( n + 1) sec 2 nθ
Fa I
cc

2
h2 n
rn
= ( n + 1) G J , ∵ cos nθ =
r 3
Hr K
n
an
h 2 ( n + 1) a 2 n 1
Su

or F= or F∝
2n + 3 2n + 3
r r
4. A particle describes the curve au = tan h (θ / 2 ) under a force F to the pole. Find the law
of force.
Solution. The equation of given curve is
θ FG IJ
au = tan h
2 H K ...(i)
Differentiating with respect to θ, we get
du 1
sec h 2
θ FG IJ
a

=
2 2 H K
d 2u
= − sec h 2
FG θ IJ tan h FG θ IJ
or a
dθ 2 H 2K H 2K ...(ii)
The differentiating equation of the path is
d 2u F
+u=
2 2 2
dθ h u
SuccessClap: Best Coaching for UPSC Mathematics : For Info- 9346856874
Checkout ->22 Weeks Study Plan, Videos, Question Bank Solutions, Test Series

Central Orbits 799


1 θ θ 1 θ F
or − sec h 2 tan h + tan h = , from (i), (ii)
a 2 2 a 2 h u2
2

1
tan h
θ LM
1 − sec h 2
θ
=
F OP
or a 2 N 2 2 2
h u Q
LM
u tan h 2
FG θ IJ OP = F ,
or
MN H 2 K PQ h2u2 from (i)

h2a2 1
or F = h 2 a 2 u5 = or
. F∝
5
r r5
5. A particle describes the curve p 2 = ar under a force F to the pole, find the law of force.
Solution. The equation of given curve is
p 2 = ar ...(i)
Differentiating with respect to r,

ap
dp
2p . =a
dt
dp a a
or = = from (i)
dr 2 p 2 ( ar )
FG IJ

l
1 a
sC 2
We have the pedal equation of central orbit as
=
r H K ....(ii)

h 2 dp h2 1 a h2 1 FG IJ
F= . =
p 3 dr ( ar ) 3/ 2 2
.
r
= .
2a r 2 H K
es
1
∴ F∝ .
r2
EXERCISES
cc

1. A particle describes the equiangular spiral r = ae θ cot α under a force F to the pole. Fnid
LMAns. F ∝ 1 OP
the law of force.
N r Q 3
Su

2. Find the law of force, if the path of the particle is r n = A cos nθ + B sin nθ.
LMAns. F ∝ 1 OP
N r Q2n + 3

3. Find the law of force, if the path of the particle is r 2 = a 2 cos 2θ.
LMAns. F ∝ 1 OP
4. A particle is describing an ellipse under a force to a pole, find the law of force.
N r Q 7

LMAns. F ∝ 1 OP
N r Q 2

5. Find the force to the pole when an particle describes the curve r = a sin θ.
LMAns. F ∝ R| 2n a
2 2
( n 2 − 1) U|OP
MN S| r − V|P
T 5
r3 WQ
SuccessClap: Best Coaching for UPSC Mathematics : For Info- 9346856874
Checkout ->22 Weeks Study Plan, Videos, Question Bank Solutions, Test Series

800 Dynamics

cos hθ − 2
6. A particle describes the curve au = under a force F to the pole. Find the law of
cos hθ + 1
LMAns. F ∝ 1 OP
force.
N r4 Q
7. A particle describes the curve r n cos nθ = a n under a force F to the pole. Find the law of
force. [Ans. F ∝ r 2 n − 3 ]
8. A particle describes the curve r cos hnθ = a under a force F to the pole. Find the law of
LMAns. F ∝ 1 OP
force.
N r Q2

9. A particle describes a parabola 2 p 2 = lr under a force to its pole, find the law of force.
LMAns. F ∝ 1 OP
N r Q

ap
2

b2 2a
10. A particle describes the curve = + 1 under a force F to the pole, find the law of
2 r
p
LMAns. F ∝ 1 OP

l
force.
sC N r2 Q
7.3 AREAL VELOCITY
Let A and B be the two neighbouring positions of the particle at
B
es
times t and t + δt respectively, moving along the curve. With O as (r + δr, θ + dθ)
A (r, θ)
pole, let coordinates of A and B be (r, θ) and ( r + δr, θ + δθ)
respectively. δθ

Then in time δt, the sectorial area OAB swept out by OA


cc

N
1 θ
= OA . OB sin δθ
2 O X
1
= . r ( r + δr ) sin δθ
Su

2
1
= r 2 dθ, to the first approximation.
2
Hence rate of description of the sectorial area OAB as radius vector passes through OA
LM 1 r δθ OP
2

= lim
δt → 0
MM 2 δt PP = 21 r 2 dθ
dt
MN PQ
1
= h, from equation (iii), of 7.2
2
Hence, the rate of description of the sectorial area is constant or in other words the sectorial
area traced out by the radius vector to the centre of force increases uniformly per unit of time.
This rate of description of sectorial area is defined as the areal velcoity of particle at A about
the fixed point O.
Again, sectorial area OAB
1
= (base AB) . perpendicular from O on AB.
2
SuccessClap: Best Coaching for UPSC Mathematics : For Info- 9346856874
Checkout ->22 Weeks Study Plan, Videos, Question Bank Solutions, Test Series

Central Orbits 801


∴ Rate of description of sectorial area OAB
1LM. AB . perpendicular form O on AB
OP
= lim
δt → 0
2
MM δt
PP
MN PQ
=
1
lim
LM
AB δs
. . (perp. form O on AB)
OP
N
2 δt → 0 δ s δ t Q
Now, as δt → 0, B → A and secant AB → tangent at A to the curve. Hence, perpendicular
AB
distance from O on AB → p and → 1.
δs
Hence, from (ii), we get
rate of description of sectorial area OAB
=
1 LM
1.
ds 1
. p = vp
OP
2 N dt 2 Q ...(iii)

ap
Hence, from (i) and (iii), we get
1 1
h = vp
2 2
h
or h = vp or v = ...(iv)

l
p
Hence, the linear velocity varies inversely as the perpendicular from the centre upon the
tangent to the path.
Again from (iv)
sC
h h2 v2 1
v= or v 2 = or =
2 2
p p h p2
es
We have from differential calculus
1
=
1
+
1 dr RS UV
2
= u2 +
du
2
RS UV
, 1
p 2 r 2 r 4 dθ T W dθ T W where u =
r
v2 R du U
+S V
2
cc

= u2

h2 T dθ W
LM R du U OP 2
v2 = h2
MNu + ST dθ VW PQ
2
or ...(v)
Su

7.4 AREAS AND APSIDAL DISTANCE


An apse is a point on a central orbit at which the radius vector drawn from the centre of the
force is a maximum or minimum. The length of the radius vector at such a point is known as
apsidal distance and the angle between two apsidal distances is called the apsidal angle.
1
As u = , hence r will be maximum or minimum according as u will be minimum or maximum
r
du
and from differential calculus, we know that = 0.

Also, we know that
dθ dθ du
tan φ = r =r .
dr du dr
tan φ = r


1RS =−
UV
1 dθ
=−u

or
du r 2
T r duW du
SuccessClap: Best Coaching for UPSC Mathematics : For Info- 9346856874
Checkout ->22 Weeks Study Plan, Videos, Question Bank Solutions, Test Series

802 Dynamics
du
∴ = − u cot φ = 0.

π
Hence, cot φ = 0 or
. φ=
2
But φ is the angle between the tangent and radius vector. Hence, at an apse tangent is
perpendicular to radius vector.
7.4.1 Properties of The apse Line
If the central force is a single valved function of the distance, every apse line divides the
orbit into two equal and similar portions and thus there can only be two apse distances.
Let the central force be F, which is single valued depending upon the distances only say F( r ).
∴ F = F (r ) ...(i)
Equation of motion gives
d 2u F

ap
u+ =
2 2 2
dθ h u
du
Multiplying by 2 and integrating with respect to θ, we get

FG du IJ FG du IJ z F(r) . r . d FGH 1r IJK

l
2 2
u2 +
H dθ K z2 F
u2 +
H dθ K
2 2
=
h u2
sC du
2 or =
h 2

=−
2
h2
z F(r) dr
1 RS du UV2
es
= u2 +
But
p2 T dθ W

p h2
1
2
=−
2
f ( r ) dr.z
cc

It is clear that p is a function of r and hence φ is also a function of r only.


Above relation also gives,
h2
p2
= − 2 F( r ) dr z
Su

∴ v 2 = − 2 F( r ) dr z ...(ii)
This gives that velocity and therefore, acceleration depends upon radius vector only, i.e., for
the same r, velocity remains the same.
Equation (ii) shows that F is a single valued function of the distance r, so the velocity is same
at the same distance r and does not depend on the direction of the motion. Also, acceleration is
same at the same distance because P is single valued function of r. Thus both velocity and
acceleration are the same distance r from the centre.
Hence if at an apse, direction of velocity is reversed it will describe a symmetrical curve on
both sides of the apsidal distance. When the particle arrives at the second apse, the path is again
symmetrical about this second apse. But this is possible only if the next, i.e., third apsidal distance
is equal to the one before it, i.e., first. Hence, there are only two different apsidal distances.

7.5 VELOCITY IN A CIRCLE


If v be the velocity with which a particle is projected at r = a from the centre of force at right
angles to the radius vector, then the particle describes a circle of radius a and the velocity v is
called velocity in a circle.
SuccessClap: Best Coaching for UPSC Mathematics : For Info- 9346856874
Checkout ->22 Weeks Study Plan, Videos, Question Bank Solutions, Test Series

Central Orbits 803


Thus, along the radius vector,
v2
= F = [ F( r )]at r = a
a
or v 2 = aF( a)
Velocity from infinity to r = a.
It is the velocity acquired by the particle in falling from infinity to a point r = a under the
attraction F = R(r ) towards the centre of force.
v2
Hence,
2
=− z∞a F(r) dr
Velocity of fall to the point of projection :
If a particle falls from the centre of repulsion under a force F to a point at a distance r. Then
1 2
2
v = 0r F dr.z

ap
7.6 Time in an Orbit
The time of passing through one point A to another point P of a central orbit is given from the
equation
dθ dy dx P
h = r2 =x −y

l
sC dt dt dt
where x, y are the coordinates of P with centre of force as origin.
Also, time for any are AP is given by
area AOP
t= .
1
h O A
2
es
EXAMPLES
R| a4 U|
1. A particle moves with a central acceleration µ r + S| V| being projected from an apse at
T r3 W
cc

a distance a with a velocity 2 µa. Prove that it describes the curve r 2 (2 + cos 3θ) = 3a 2 .
Solution. We have the differential equation of path as
d 2u F
+u= ...(i)
Su

2 2 2
dθ h u
Fa4 I FG IJ
Given that F = µ r + GH
r 3

1
u
JK
+ a4u3
H K
Hence, from (i), we get
 d 2u  1 
h 2  2 + u  = µ  3 + a 4u 
 d θ  u 
du
Multiplying both sides by 2 and integrating, we get

LMF I 2 OP FG IJ
MNGH JK
du 1 1
v2 = h2 + u 2 = 2 µ − u −2 + a 4 u 2 + C ,
dθ PQ 2 2H K ...(ii)
where C is the constant of integration.
1
Initially, r = a, i. e., u = ,
a
du
v = 2 µ a and = 0 (at an apse).

SuccessClap: Best Coaching for UPSC Mathematics : For Info- 9346856874
Checkout ->22 Weeks Study Plan, Videos, Question Bank Solutions, Test Series

804 Dynamics
Hence, from (ii) we get
4 µa 2 = h 2 .
1
= 2µ −
LM 1 2 1 2
a + a +C
OP
a 2
N 2 2 Q
2 2
∴ h = 4µa and C = 4µa 2
Hence, from (ii), we get
 du 2   1 
4µa2   + u 2  = µ − + a4u 2  + 4µa2
2
 d θ    u 
2
 du  1 4a2u 2 − 3a4u 4 − 1
or 4a2   = − 2 + a4u 2 + 4a2 − 4a2u 2 =
 dθ u u2
LM
− a2u2 3−
2 OP 2
+
4
−1
1 RS
− a2u2 3−
2 UV 2

= N 3Q 3
=
3 T 3W
u2 u2
LM 1 F OP

ap
IJ 2

MN 3 − GH a u
du 1 2
2a 2 2 2
or dθ
=−
u
3−
3K PQ
− 2 3 a 2 u du

l
= 3dθ
or
LM 1 F 2 2 2 I OP
sC 2

MN 3 − GH a u 3− J
3K P
Q
2
Let a2u2 3− =t
es
3
∴ 2 3a 2 u du = dt
− dt
= 3dθ
FG IJ
cc

or 1 2
3H −t
K
Integrating, we get
cos −1 (t 3 ) = 3θ + A ...(iii)
Su

where A constant of integration.


1 FG 2 1IJ
a H
Initially, u = , i. e., t = 3 −
3
=
3Kand let θ = 0, then A = 0.
Hence, from (iii), we get
cos −1 (t 3 ) = θ 3 or t 3 = cos (θ 3 )
FG
3 a2u2 3 −
2 IJ
or
H 3 K
= cos (θ 3 ) or 3a 2 u 2 − 2 = cos (θ 3 )

2 2
3a 2
or 3a u = 2 + cos ( 3θ) or = 2 cos ( 3θ)
r2
or r 2 (2 + cos 3θ) = 3a 2
This is the required equation of the path.
2. A particle moves under a force mµ {3au 4 − 2 ( a 2 − b 2 ) u 2} a > b and is projected from an
µ
apse at a distance (a + b) with velocity . Show that the equation of its path is
a+b
r = a + b cos θ.
SuccessClap: Best Coaching for UPSC Mathematics : For Info- 9346856874
Checkout ->22 Weeks Study Plan, Videos, Question Bank Solutions, Test Series

Central Orbits 805


Solution.We know that the differential equation of the path is
d 2u F
+u=
dθ 2 2
h u
We have F = µ {3au 4 − 2 ( a 2 − b 2 ) u 5}
Hence, the path becomes
d 2u µ
+u= {3au 2 − 2 ( a 2 − b 2 ) u 3}
2 2
dθ h

h2
d 2
u LM OP
+ u = µ {3au 2 − 2 ( a 2 − b 2 ) u 3}
or
dθ 2 MN PQ
du
multiplying both sides by 2 and integrating, we get

2LMF I OP
MNGH JK
du
v2 = h2 + u 2 = µ {2 au 3 − ( a 2 − b 2 ) u 4} + c
PQ ...(i)

ap

w h e r e c is constant of integration.
1 µ du
Initially, r = a + b, i. e., u = ,v= , =0
( a + b) ( a + b) dθ

l
∴ From (i),
LM OP = µ LM 2a − (a − b ) OP + c
µ
sC
= h2 0 +
1 2 2

( a + b) 2
MN
( a + b) PQ MN (a + b) (a + b) PQ
2 3 4

µ
=
h 2
=µM
L 2a − (a − b) OP + c
or
MN (a + b)3 PQ
es
2 2
( a + b) ( a + b)
µ h µ
or = = +c
2 2
( a + b) ( a + b) ( a + b) 2
cc

∴ h = µ, c = 0
Then, from (i)
LMF du I 2 OP
µ
MNGH dθ JK + u 2 = µ [2 au 3 − ( a 2 − b 2 ) u 4 ]
PQ
Su

FG du IJ 2
= 2 au 3 − ( a 2 − b 2 ) u 4 − u 2
or
H dθ K
1 du 1 dr
Putting u = ⇒ =− 2 we get
r dθ r dθ
1 dr
2
FG IJ
2a (a 2 − b 2 ) 1
r 4 dθ
=
rH K
3

r 4

r2
FG dr IJ = 2ar − (a
2
2
− b 2 ) − r 2 = b 2 − ( r − a) 2
or
H dθ K
dr
or = ± {b 2 − ( r − a) 2}

dr
− = dθ
or
{b 2 − ( r − a) 2}
SuccessClap: Best Coaching for UPSC Mathematics : For Info- 9346856874
Checkout ->22 Weeks Study Plan, Videos, Question Bank Solutions, Test Series

806 Dynamics

Integrating, cos −1
RS (r − a) UV = θ + c
T b W 1

Initially, r = a + b, θ = 0, so we get c1 = 0
r − a
∴ cos−1   = θ ⇒ r − a = b cos θ
 b 
or r = a + b cos θ.
This is the required equation of the path.
3. A particle moves with a central acceleration which varies inversely as the cube of the
distance. If it be projected from an apse at a distance a from the origin with a velocity which is
2 times the velocity for a circle of radius a. Show that the equation of its path is
r cos (θ / 2 ) = a.
Solution. Let v1 be the velocity for a circle of radius a with a central acceleration
∝ (distance) −3 . Then

ap
v12 µ µ µ
= or v12 = or v1 =
3
a a a2 a
Hence, if v2 be the velocity of projection, then

l
µ (2µ)
sC v2 = 2. v1 = 2 .
Now, the differential equation of the path is
a
=
a
...(i)

d 2u F
+u= ,
2 2 2
dθ h u
FG IJ
es
µ 1
it is given that F = 3 = µu 3 , where u = .
r H K r

or
d 2u
+u=
µu 3
or h2
LM d u + uOP = µu
2

MN dθ PQ
cc

dθ 2 h2u2 2

du
Multiplying both sides by 2 and integrating, we get

LMF I OP F I 2
u2
Su

MNGH JK PQ GH JK
du
v2 = h2 + u 2 = 2µ +C ...(ii)
dθ 2
1 F du I
Initially, r = a, i. e., u = , G J = 0 and v = (2µ ) / a.
a H dθ K
Hence, from (ii), we have
2µ h2 µ µ
= = + C or h 2 = 2µ and C =
2 2 2
a a a a2
Hence, from (ii) we get
LMF du I OP F 1 I
2

MNGH dθ JK + u PQ = µ GH u + a JK
2 2

2

FG du IJ 2 + u 2 1F 1I F du I 1 F 1 IJ
2
1 − a2u 2
= Gu + 2
J or G J = G − u2 =
or
H dθ K 2H a K H dθ K 2 H a
2 2 K 2a 2
a du 1
= dθ
or
[(1 − a 2 u 2 )] 2
SuccessClap: Best Coaching for UPSC Mathematics : For Info- 9346856874
Checkout ->22 Weeks Study Plan, Videos, Question Bank Solutions, Test Series

Central Orbits 807

sin −1 ( au) =
FG θ IJ + A,
Integrating, we have
H 2K ...(iii)
where A is the constant of integration.
1 π
Initially, u = and θ = 0, ∴ A=
a 2
Hence, from (iii) we get
sin −1 ( au) =
1 FG θ IJ
2
π+
H 2K
FG π + θ IJ = cos FG θ IJ
or au = sin
H2 2K H 2K
a = r cos G
F θ IJ .
or
H 2K
This is the equation of the path.

ap
4. A particle moves with a central acceleration [µ / (distance) 5 ] and projected from the apse
at a distance a with a velocty equal to n times that which would be acquired in falling from
infinity, show that the other apsidal distance is a / ( n 2 − 1).

l
If n = 1 and particle be projected in any direction, show that the path is a circle passing
sC
through the centre of force.
dv µ
Solution. If v be the velocity at a distance x from the centre, then v =− , negative sign
dx x5
indicates that the particle is moving towards the centre.
es
Hence, if V be the velocity from infinity to a distance a from the centre then
a
v 1  1  µ
∫0 v du = − µ ∫∞a dx or V 2 = 2µ  4  = 4
x5  4 x  ∞ 2a
FG µ IJ
cc

or V=
H 2a 4 K
Hence, the velocity of projection of the particle
FG IJ
Su

µ
= nV = n
2a 4 H K ...(i)
We know that the differential equation of the path is
d 2u F
+u= ...(ii)
2 2 2
dθ h u

or
2
d u
+u=
µu 5
or h
2
2 d u
LM
+ u = µu 3
OP
dθ 2 h 2u 2 dθ 2 MN PQ
du
Multiplying both sides by 2 and integrating, we get

LMF I
2 OP
u4
MNGH JK
du
v2 = h2 + u2 = µ + C,
dθ PQ
2 ...(iii)
where C is constant of infegration.
FG IJ
µ du 1
Initially, v = n
H K
2a 4
,

= 0 and u =
a
SuccessClap: Best Coaching for UPSC Mathematics : For Info- 9346856874
Checkout ->22 Weeks Study Plan, Videos, Question Bank Solutions, Test Series

808 Dynamics
∴ From (iii), we get
n2µ
= h2
1 LM OP
=
µ
+ C or h 2 =
n2µ
and C =
µ ( n 2 − 1)
2a 4
Hence, from (iii), we get
a2 N Q
2a 4 (2a 2 ) 2a 4

n 2 µ LF du I O µu 4 µ (n2 − 1)
2
MG J + u2 P =
2 a 2 MNH dθ K PQ 2 + 2a 4
FG du IJ + u = 1 [a u + (n − 1)]
2
2 4 4 2
or
H dθ K a n 2 2

FG IJ = M OP [a u − a n u + (n
du
2
L 1 4 4 2 2 2 2
or
H dθ K N a n Q 2 2
− 1)] ...(iv)
du
Now, at any apse = 0 , so form (iv) the apsidal distances are given by

ap
a 4 u 4 − a 2 n 2 u 2 + ( n 2 − 1) = 0 or ( n 2 − 1) r 4 − a 2 n 2 r 2 + a 4 = 0
Let r12 and r22 be the roots of this quadratic equation in r 2 . Then
a4 a2
r12 . r22 = or r1r2 =

l
sC ( n 2 − 1) ( n 2 − 1)
But one of the apsidal distance is a. Let r1 = a.
a2 a
ar2 = or r2 =

[( n 2 − 1)] [( n 2 − 1)]
es
If n = 1, from (iv), we have
LM du OP 2
=
1
( a 4 u 4 − a 2 u 2 ) = u 2 ( a 2 u 2 − 1)
N dθ Q a 2
cc

du
= dθ
or
u [( a 2 u 2 − 1)]
Integrating, we get
sec −1 ( au) = θ + A
Su

1
Initially, u = and θ = 0, ∴ A = 0. Hence,
a
sec −1 ( au) = θ
a
or au = sec θ or = sec θ or r = a cos θ
r
This is the polar equation of a circle passing through the pole, i.e., centre of force.
5. A particle subject to a force producing an acceleration µ ( r + 2 a) / r 5 towards the origin
is projected from the point (a, 0) with a velocity equal to the velocity from infinity at an angle
cot −1 2 with the initial line. Show that the equation to the path is
r = a (1 + 2 sin θ)
Solution. The differential equation of the path is
d 2u F
+u=
2 2 2
dθ h u
1 FG IJ
Here, F=
µ ( r + 2 a )
=
µ
u
+ 2a
H K
= µ (u 4 + 2 au5 )
5 5
r 1/ u
SuccessClap: Best Coaching for UPSC Mathematics : For Info- 9346856874
Checkout ->22 Weeks Study Plan, Videos, Question Bank Solutions, Test Series

Central Orbits 809


d 2u µ (u 4 + 2 au 5 )
∴ +u=
dθ 2 h 2u 2
F d u + uI = µ ( u
2
or h2 GH dθ JK 2
2
+ 2 au 3 )

du
Multiplying both sides by 2 and integrating, we have

2R|F I U|
v2 = h2
du
S|GH JK V|
+ u2

T W
R 1 1 U
= 2µ S u + au V + c 3 4
T 3 2 W ...(i)
Now, the velocity of projection of the particle is equal to the velocity acquired by a particle in
falling from infinity to the point of projection under given acceleration. Let v1 be the velocity thus

ap
acquired. We have,
v
dv
=−µ
x + 2a RS UV
dx x5 T W
RS UV

l
1 2a
v dv = − µ + dx
or
T W
sC x 4
x5
FG IJ

v1
z a
v = 0 v dθ = − µ x = 0
x
1
4
+
2a
x5
dx z H K
v12 1 2a LM OP a
es
=−µ − −
or
2 3x 3
4x4 N Q

1
+
1 LM OP
3a 3
2a 3 N Q
cc


or v12 =
( 3a 3 )
Also, from p = r sin φ, we have initially, p0 = a sin φ 0 .
φ 0 = cot −1 2
Su

Here, or cot φ 0 = 2
1
or sin φ 0 =
5
a
∴ p0 = a sin φ 0 =
5
F IJ 2
FG du IJ 2
4 GH
1 1 1 dr
= u2 +
Also, we have
p2
=
r2
+
r K dθ H dθ K
FG du IJ 2 + u 2 = 1 = 5
∴ Initially,
H dθ K p02 a 2
∴ From (i), initially, we have

= h2
5
= 2µ
LM 1 +
1 OP + c
3a 3
2
a 2
N 3a 3
2a 3
Q
5µ 5h 5µ
or = = +c
3 2
3a a 3a 3
SuccessClap: Best Coaching for UPSC Mathematics : For Info- 9346856874
Checkout ->22 Weeks Study Plan, Videos, Question Bank Solutions, Test Series

810 Dynamics
µ
∴ h2 = and c = 0 .
3a
∴ From (i), we get
LMF du I OP L u au
2 3 4 OP
MNGH dθ JK + u PQ = 2µ MMN 3 + 2
µ 2
3a PQ
FG du IJ = 2au + 3a u − u
2
3 2 4 2
or
H dθ K
1 du 1 dr
Putting u= , =− , we get
r dθ r 2 dθ
1 dr FG IJ
2
2 a 3a 2 1
r 4 dθ H K
=
r 3
+
r 4

r2
FG dr IJ 2
= 2 ar + 3a 2 − r 2 = 4 a 2 − ( r − a) 2
or
H dθ K

ap
dr
= dθ
{(2a) − ( r − a) 2}
2

Integrating, we get

l
sC sin −1
RS (r − a) UV = θ + c
T 2a W 1

Initially, θ = 0, r = a ∴ c1 = 0

sin −1
RS r − a UV = θ
Hence,
T 2a W
es
or r − a = 2a sin θ
or r = a (1 + 2 sin θ)
This is the required equation of the path.
F 5 + 8c I . and is
cc

2
6. A particle of mass m moves under a central attractive force mµ GH r r JK
3 5

3 µ 2
projected from an apse at a distance c with velocity , prove that the orbit is r = c cos θ,
Su

c 3
πc 2
and that it will arrive at the origin after a time .
8 µ
Solution. Here, the central acceleration is
5 8c 2 F I
F=µ
r3
+ GH
r5
JK
= µ (5u 3 + 8c 2 u 5 )

The differential equation of the central orbit is


d 2u P
u+ =
2 2 2
dθ h u

h2 u +
LMd 2
u
=
µ OP
(5u 3 + 8c 2 u 5 ) = µ (5u + 8c 2 u 3 )
or
MN
dθ 2
u2 PQ
du
Multiplying both sides by 2 and integrating, we get

SuccessClap: Best Coaching for UPSC Mathematics : For Info- 9346856874
Checkout ->22 Weeks Study Plan, Videos, Question Bank Solutions, Test Series

Central Orbits 811


LM FG du IJ 2 OP
v2 = h2 u2 + 2
+ 4c 2 u 4 ) + c
MN H dθ K PQ = µ (5u ...(i)

1 du
Initially, at an apse r = c, u = , = 0, v = 3 µ / c,
c dθ
∴ From (i), we get
9µ h 2 5 4 FG IJ
c 2
=
c 2

c 2
+
c2 H+C
K
9 µ h 2 9µ
or = = +C
c2 c2 c2
∴ h 2 = 9µ , c = 0
Substituting these values in (i), we get
LM du
2
FG IJ OP
9µ u 2 + = µ [5u 2 + 4c 2 u 4 ]
MN H K PQ

ap

FG du IJ 2
= 4 c 2 u 4 − 4u 2
or 9
H dθ K

l
1 du 1 dr
Putting u = , so that =− , we get
r dθ
sC r 2 dθ
FG IJ FG
2
c2 I
or 9.
r
1 dr
4 dθ H K H
=4
r 4

1
r2
JK
F dr I 2
9 G J = 4 (c 2 − r 2 )
es
H dθ K
dr 2
or =− (c 2 − r 2 )
dθ 3
cc

2 − dr
dθ =
or 3 (c 2 − r 2 )
FG IJ
Integrating, 2 θ + A = cos −1 r , where A is constant of integration.
HK
Su

3 c
Initially, when r = c, let θ = 0. Then, A = cos −1 1 = 0
2
θ = cos −1
r FG IJ
∴ 3 c HK
r 2
or = cos θ
c 3
2
or r = c cos θ
3
This is the required equation of the path.
Again, we have
h = r2
dθ FG IJ
dt H K
2
But h = 3 µ and r = c cos θ
3
2
c 2 cos 2 θ
r2 3 dθ
∴ dt = dθ = ...(ii)
h 3 µ
SuccessClap: Best Coaching for UPSC Mathematics : For Info- 9346856874
Checkout ->22 Weeks Study Plan, Videos, Question Bank Solutions, Test Series

812 Dynamics
At the point of projection, we have taken θ = 0. Also, at the point O, r = 0.
Putting r = 0 in the equation of path, we get
2
0 = c cos θ
3
2 π 3
i.e., θ= or θ = π
3 2 4
3
So, at O, θ = π. Let t1 be the time from the point of projection to the point.
4
Then, integration (ii), we get

z
t1 = 03π / 4
c2
cos 2
2 FG IJ
3 µ 3
θ dθ
H K
2 2 3 π
Put θ = φ, so that dθ = dφ. When θ = 0, φ = 0 and when θ = π, φ = .
3 3 4 2
2

ap
∴ z
t1 = 0π / 2
c
3 µ
2
(cos 2 φ) dφ
3
c2
=
2 µ
z0
π/ 2
cos 2 φ dφ

l
=
sC c2
1 1
. π=
2 µ 2 2
πc 2
8 µ
. .

7. A particle is moving with central acceleration µ ( r 5 − c 4 r ) being projected from an apse


2µ 3
es
at a distance c with velocity c , show that its path is the curve x 4 + y 4 = c 4 .
3
Solution. Here P = µ ( r 5 − c 4 r )
F 1 − c4 I LM∵ r = 1 OP
GH u5 u JK
cc


N uQ
∴ Differential equation of central orbit
d 2u P
+u= becomes
Su

2 2 2
dθ h u
1 c4

LM OP
2
d u
+u=µ u 5
MM
u PP
MM PP
2 2 2
dθ h u
N Q
d 2u µ 1 c  4
or + u =  − 
d θ2 h2  u 7 u3 
du
Multiplying both sides by 2 and integrating above, we get

2LMF I OP
MNGH JK
du
v2 = h2 + u2
dθ PQ
= 2µ M −
L 1 +
c4 OP + C
MN 6u 6
2u 2
PQ 1 ..(1)
SuccessClap: Best Coaching for UPSC Mathematics : For Info- 9346856874
Checkout ->22 Weeks Study Plan, Videos, Question Bank Solutions, Test Series

Central Orbits 813


From initial conditions, i.e., at an apse;
1 du
u= , =0
c dθ
2µ 3
and v= c .
3
2µ 6 h 2 c6 c6 F I
∴ 3
c =
c2
= 2 µ −
6
+
2
+ C1 GH JK
2µ 8
∴ h2 = c and C1 = 0
3
Substituting values of h 2 and C1 in (1), we get
LMF du I 2 OP LM c4 OP
MNGH dθ JK
2µ 8 1
+ u 2 = 2µ − +
3
c
PQ MN 6u 6
2u 2
PQ
  du  2   1 c4 

ap
c8
or    + u 2  = − 6 + 2 
 
3  d θ   6u 2u 

c8
FG du IJ 2 = 3 LM− 1 + c 4 OP − c8u 2
or H dθ K MN 6u 6 2u 2 PQ

l
sC =
1
6
[−1 + 3c 4 u 4 − 2c 8u8 ]
2u
=
1 LM− 1 + 3 c 4u4 − c8u8 OP
u 6
N 2 2 Q
es
LM 1 F 3I 9O
2
− − Gc u − J + P
1 4 4
=
u6 MN 2 H 4K 16 P
Q
du 1
cc

or c4 =± 1 − (4c 4 u 4 − 3) 2
dθ 3
4u
4.4c 4 u 3du
− = 4 dθ

1 − (4c 4 u 3 − 3) 2
Su

Putting 4c 4 u 4 − 3 = V , so that 4.4c 4 u 3 du = dV we get


dV
− = 4dθ
1− V2
Now integrating above, we get
cos −1 (V ) = 4θ + C2
or cos −1 (4c 4 u 4 − 3) = 4θ + C2 ..(2)
1
Intially when u = , then θ = 0; C2 = 0
c
∴ (2) becomes
cos −1 (4c 4 u 4 − 3) = 4θ
or 4c 4 u 4 = 3 + cos 4θ
or 4c 4 = r 4 [3 + cos 4θ]
= r 4 [2 + (2 cos 2 θ − 1) 2 ]
= r 4 [4 + 8 cos 4 θ − 8 cos 2 θ]
SuccessClap: Best Coaching for UPSC Mathematics : For Info- 9346856874
Checkout ->22 Weeks Study Plan, Videos, Question Bank Solutions, Test Series

814 Dynamics

= 4r 4 [1 + 2 cos 4 θ − 2 cos 2 θ]
= 4r 4 [cos 4 θ + (1 − cos 2 θ)]2
or c 4 = r 4 [cos 4 θ + sin 4 θ]
or c4 = x 4 + y4 [∵ x = r cos θ, y = r sin θ]
EXERCISES
1. A particle describes an orbit with a central acceleration µu 3 − λu 5 , being projected from an
apse at distance a with a velocity equal to that from infinity, show that the path is
θ 2µa 2
r = a cos h , where n 2 + 1 = .
n λ
2. A particle moves is a curve under a centarl acceleration so that its velocity at any point is
equal to that in a circle at the same distance and under the same attraction. Show that law of
force is inverse cube and path is an equiangular spiral.

ap
3. A particle moving with a central acceleration µ /(distance)3 is projected from an apse at a
 (µ − a 2V 2 ) 
distance a with velocity V, show that the path is r cosh  θ  = a, or
 aV 
 

l
 (a 2V 2 − µ) 
r cos 
 aV
sC
θ  = a according as V >< the velocity from infinity..

 
4. In a central orbit the force is µu 3 (3 + 2a 2u 2 ); if the particle be projected at a distance a
es
−1  1 
with a velocity (5µ / a 2 ) in a direction making an angle tan   with the radius, show
2
that the equation of the path is r = a tan θ.
5. A particle is acted on by a central repulsive force which varies as the nth power of the
cc

distance. If the velocity at any point be equal to that which would be acquired in falling
from the centre to the point, show that the equation of the path is of the form
1
r ( n + 3) / 2 cos (n + 3) θ = constant .
2
Su

6. A particle of mass m moves under a central force mµ /(distance)3 and is projected at a dis-
tance a from the centre of force with the velocity which at angle α to the radius would be
acquired by a fall from rest at infinity to the point of projection, prove that the orbit is an
equiangular spiral.
7. A particle is projected from an apse at a distance a with the velocity from infinity, the accel-
eration being µu 7 , show that the equation to its path is r 2 = a 2 cos 2θ.
8. A particle moves with central acceleration (µu 2 + λu 3 ) and the velocity of projection at
disatnce R is V. Show that the particle will ultimately go off to infinity if
2µ λ
V2 > + .
R R2
9. A particle of mass m is attached to a fixed point by an elastic string of natural length a, the
coefficient of elasticity nmg. It is projected from an apse at a distance a with velocity
(2 pgh); show that the other apsidal distance is given by the equation
nr 2 (r − a ) − 2 pha (r + a ) = 0
SuccessClap: Best Coaching for UPSC Mathematics : For Info- 9346856874
Checkout ->22 Weeks Study Plan, Videos, Question Bank Solutions, Test Series

Central Orbits 815

 2µ  µ
10. A particle is projected with velocity  3  from a point P in a field of attractive force 4
 3c  r
to a point O distant C from P, where r denotes the distance from O.
If the direction of projection makes an angle 45° with PO, prove that the orbit is cardioid
  3c 
5
 3π
and the particle will arrive at O after a time  − 2  .
 4   4 
11. A particle subject to a central force per unit of mass equal to µ {2 (a 2 + b 2 ) u 5 − 3a 2b 2u 7 }
(µ)
is projected at the distance a with velocity in a direction at right angles to the initial
a
distance. Show that the path in the curve r 2 = a 2 cos 2 θ + b 2 sin 2 θ.
 mµ 
12. A particle moves under a repulsive force =   and is projected from an apse at a
2
 (distance) 

ap
distance a with velocity V. Show that the equation to the path is r cos pθ = a, and the angle
1 −1  pVt  2 (µ + a 2V 2 )
θ , described in time t is p tan  a  , where p = .
  a 2V 2

l
µ 
13. A particle subject to a central attractive acceleration  3  + f is projected from an apse
sC r 
µ 1
at a distance a with a velocity . Prove that the at any subsequent time t, r = a − ft 2 .
a 2
14. A particle moves with a central acceleration λ 2 (8au 2 + a 4u 5 ), it is projected with velocity
es
λ from an apse at a distance a/3 from the origin, show that equation to its path is
1  au + 5  θ
  = cot
3  au − 3  6
cc

15. A particle moves under a central force mλ [3a3u 4 + 8au 2 ], it is projected from an apse at a
distance a from the centre of force with velocity (10λ). Show that the second apsidal
Su

 θ 
distance is half the first, and that the equation to path is 2r = a 1 + sec h  .
 5
 10 5 
16. If the law of force is µ  u 4 − au  and the particle be projected from an apse at a dis-
 9 
5
tance 5a with a velocity equal to   of that in a circle at the same distance, show that
7
the orbit is the limacon r = a (3 + 2 cos θ).
17. Show the only law for a central attraction for which velocity in a circle at any distance is
equal to the velocity acquired in falling from infinity to the distance is that of inverse cube.
µr
18. A particle is acted on by a repulsive central force 2 is projected from an apse at
( r − 9c 2 ) 2
 µ 
a distance c with velocity  2  . Find the equation of its path and show that the time to
 8c 
4 2 2
the cusp is πc  . [ Ans. 8 p 2 = 9c 2 − r 2 ]
3 µ
❐❐❐
SuccessClap: Best Coaching for UPSC Mathematics : For Info- 9346856874
Checkout ->22 Weeks Study Plan, Videos, Question Bank Solutions, Test Series

Planetary Motion
8.1 NEWTONIAN LAW OF ATTRACTION
According to this law the mutual attraction between two particles of masses m1 and m2 and
mm
placed at a distance r apart is γ . 1 2 2 , where γ is universal constant. This law is found to hold
r
good in the case of the motion of all planets in the solar system. Therefore motion of earth about

ap
the Sun, that of planets about the earth or of moon about the Sun is governed by this law. Here in
this chapter we will discuss the case of central orbits when the force is an attraction varying inversely
as the square of the distance from the centre of force.
8.2

l
A particle moves in a path so that its acceleration is always directed to a fixed point and is

equal to
µ
sC
; Show that the path is a conic section, and distinguish between the three
( dis tan ce )2
cases that arise.
es
This is a case of central orbit as force is always directed towards a fixed point.
µ
Here P= 2.
r
Also pedal form of differential equation is
cc

h 2 dp µ h2 µ
= P = or 3
dp = 2 dr.
3 dr 2 p r
p r
Integrating above, we get
h2 2 µ
Su

h µ = + B.
− = − + A or
2 p2 r p2 pr
Also we know that h = p v; hence the equation reduces to

2 2µ 2 h2 2 µ
v = + B. ∴ v = = + B.
r p2 r
Also we know that pedal equation of ellipse, parabola and hyperbola (that branch which is
nearer to centre of force) all referred to focus as pole are
b 2 2a 2 b2 2a
= − 1, p = ar and 2
= + 1,
2 r p r
p
respectively, where 2a and 2b are the lengths of major and minor or transverse and conjugate axes.
Now comparing equation (1) with these pedal equations of ellipse, parabola and hyperbola, we
get
Case 1. With ellipse
h2 µ B µb 2 µ
= = ; ∴ h2 = and B = − .
2 a − 1 a a
b
SuccessClap: Best Coaching for UPSC Mathematics : For Info- 9346856874
Checkout ->22 Weeks Study Plan, Videos, Question Bank Solutions, Test Series

Planetary Motion 817


Therefore (1) reduces to
h 2 2µ µ 2 1
v2 = 2 = − = µ  − .
p r a r a
 2 1 2µ
From v 2 = µ  −  , we see that v 2 < .
 r a r
Case 2. With parabola.
On comparing (1) with pedal equation of parabola, we get B = 0 and h 2 = 2aµ .
2 2µ
∴ (1) reduces to v = .
r
Case 3. With hyperbola.
h2 µ B 2 b2 µ
2
= = ; h = µ and B = .
b a 1 a a
2 2µ µ  2 1

ap
∴ (1) reduces to v = + = µ  + .
r a  r a
2  2 1 2 2µ
From v = µ  +  , we see that v >
 r a r

l

Thus from above three cases, we conclude that v 2 = + B always represents a conic section
sC r
whose focus is at the centre of force and it represents an ellipse, parabola or hyperbola according
as B is – ve, 0 or + ve.
Also we have found that
 2 1
es
if v 2 = µ  −  , it is an ellipse;
 r a
2 2µ
if v = , it is a parabola;
r
cc

2  2 1
if v = µ  +  , it is a hyperbola;
 r a
or from above relations, we conclude that

if v 2 <
Su

, it is an ellipse;
r

if v 2 = , it is a parabola;
r

if v 2 > , it is a hyperbola;
r
Also we have found that
b2
h2 = µ = µl in case of ellipse;
a
h 2 = 2aµ = µl in case of parabola;
b2
h2 = µ = µl in case of hyperbola.
a
Therefore in all the three cases.
h 2 = µl
or h = { (µl )} [where l is semi-latus rectum].
We observe that velocity at any point in independent of the direction of the velocity.
SuccessClap: Best Coaching for UPSC Mathematics : For Info- 9346856874
Checkout ->22 Weeks Study Plan, Videos, Question Bank Solutions, Test Series

818 Dynamics
Cor 1. If a particle be projected from a point at a distance R from the focus with velocity V in
any direction, the path will be an ellipse, parabola or hyperbola according as

V2 < = >
R

Here we observe that is the square of the velocity the paraticle will acquire in falling from
R
infinity to the distance R from the centre as
dv µ V1 R 1
v
dr
=− 2;
r
∫0
v dv = − µ ∫
∞ r2
dr
R
V12  µ  µ 2µ
or =  = ; V12 =
2  r ∞ R R
Therefore the path will be an ellipse, parabola or hyperbola according as the velocity at any
points is < = > the velocity the particle will acquire in falling from infinity to that point.
Cor 2. The velocity V2 for the description of a circle of R is given by

ap
V22 µ
= (normal acceleration)
R R
µ 1 2µ 1  2µ 
∴ V22 = = . or V 2 =  

l
R 2 R sC 2 R
 2µ 
As we have already proved above that   is velocity the particle would acquire in falling
R
from infinity to the point distant R from the centre, hence
1
es
V2 = (velocity from infinity)
2
1
or velocity for the description of a circle of radius R = (velocity from infinity) to the distance
2
cc

R from the centre.


Note. In the above article, when the central acceleration was towards the focus, the branch
of hyperbola described in the one nearest to the focus.
Now if we consider the case when the central acceleration is away from the focus, then the
further branch of hyperbola is described.
Su

Now in this case equation of motion (orbit) is


h 2 dp µ h2 µ
= − or 3
dp = − 2 dr.
3 dr 2 p r
p r
Integrating above, we get
h2 2µ
v2 = 2 = − + A. ...(1)
p r
Also the pedal equation of further branch of hyperbola is
b2 2a
2
=1− ...(2)
p r
Now comparing (1) and (2), we get
h2 µ A 2 b2 µ
= = . ∴ h = µ and A = .
b 2 a 1 a a
Therefore (1) reduces to
1 2
v2 = µ  −  .
a r 
SuccessClap: Best Coaching for UPSC Mathematics : For Info- 9346856874
Checkout ->22 Weeks Study Plan, Videos, Question Bank Solutions, Test Series

Planetary Motion 819


8.3 KEPLER’S LAWS
There are three laws discovered by the astronomer Kepler connecting the motion of various
planets about the sun :
(i) Each planet describes an ellipse with the Sun as one of its foci.
(ii) The areas described by the radii drawn from the planet to the Sun are, in the same orbit,
proportioanl to the times of describing them.
(iii) The squares of the periodic times of the planets are proportional to the cubes of the major
axes of their orbits.
Deductions
(1) We have proved in chapter of central forces that if the orbit is an ellipse under an acceleration
towards one of its foci, then the law of the acceleration is that of the inverse square of the distance
from the centre (focus). Therefore we conclude that the acceleration of each planet towards the
Sun varies inversely as the square of its distance from the Sun.
(2) From the second law we find that the rate of description of sectorial area is constant and

ap
this is true only when the acceleration of the planet and therefore force on it is directed towards
the sun.
(3) We know that periodic time of a central orbit under inverse law is given by
2π a 3 / 2 4π 2 a 3
T= or T2 = ,

l
µ
sC µ
i.e., the square of periodic time varies as the cube of the major axes.
Hence from the third law we conclude that µ is same for all the planets, i.e., absolute
acceleration µ (acceleration at unit distance from the sun) is the same for all planets.
8.4 ACCURATE VALUE OF µ
es
Kepler’s third law is true only on the supposition that the
Sun is fixed and mass of the Sun is so great in comparison to
S β α P
that of the planet that the effect on its motion on account of its
attraction is negligible.
cc

A more accurate form is obtained below :


Let S be the mass of the Sun and P that of any planet and γ the constant of gravitation.
Now mutual attraction between them at any instant, when distance between them is r, is given
Su

SP
by γ 2 .
r
SP S
Therefore planet’s acceleration α = γ 2 = γ 2
r .P r
S .P P
and sun’s acceleration β = γ 2 = γ 2 .
r S r
and the relative acceleration of the planet with respect to the sun is α + β.
γ ( S + P) µ
∴ α+ β = = 2.
r2 r
Therefore here µ = γ ( S + P).
2 πa 3 / 2
Also periodic time is given by T = ;
µ

therefore exact periodic time T = a3 / 2 . ...(1)
[ γ ( S + P)]
SuccessClap: Best Coaching for UPSC Mathematics : For Info- 9346856874
Checkout ->22 Weeks Study Plan, Videos, Question Bank Solutions, Test Series

820 Dynamics

Now let T1 be the periodic time of another planet of mass P1 and a1 be semi-major axes of its
orbit; then

T1 = a13 / 2 . ...(2)
[ γ ( S + P1 )]
Therefore from (1) and (2), we get
T 2 S + P1 a3 S + P T 2 a3
= . or . = .
T12 S + P a13 S + P1 T12 a13
If D be the mean distance (distance between centres) of the planet from the Sun, then we can
also say

T= D3 / 2 . ...(3)
[ γ ( S + P)]
Similarly if p be the mass of satellite of planet P and the mean distance between their centres
be a, then the periodic time t of satellite is given by

ap

t= d 3/ 2. ...(4)
[ γ ( P + p)]
Therefore from (3) and (4), we get
T 2 P + p D3 S + P T 2 D3
= 3.

l
= . or .
t2 S + P d3 P + p t2 d
8.5.
sC
In solving questions in this chapter we will require help of properties of ellipse. So some of the
important properties are given below :
(1) The product of perpendiculars drawn from the foci
es
Z
on tangent at any point on ellipse is constant and is equal Y P
φ φ
to square of the semi-minor axis of the ellipse, i.e.,
SY .HZ = b 2
cc

(2) The sum of the focal distances of a point on an ellipse S C H


is 2a, where 2a is length of major axis of ellipse i.e.,
SP + HP = 2a.
b2
Su

(3) The length of latus rectum is 2 , where


a
b 2 = a 2 (1 − e 2 ),
e being the eccentricity of the ellipse.
(4) The tangent and normal at any point are each equally inclined to focal radii of that point.

EXAMPLES
µ
1. A particle moves with a central acceleration ; it is projected with velocity V
(distance) 2
at a distance R. Show that its path is a rectangular hyperbola if the angle of projection is
µ
sin −1 1/ 2
.
 2 2µ 
VR V − 
 R
Sol. We know that in the case of hyperbola
2 1
v2 = µ  +  .
r a
Therefore in this case
SuccessClap: Best Coaching for UPSC Mathematics : For Info- 9346856874
Checkout ->22 Weeks Study Plan, Videos, Question Bank Solutions, Test Series

Planetary Motion 821


 2 1
V2 = µ  +  as [v = V , r = R given]
R a
2 µ µ
or V2 − = ...(1)
R a
Also we know that h = pv.
Let p = p0 initially.
h = p0V = R sin α V (∵ p0 = R sin α ) ...(2)
 b  2
Also we know that h = (l µ ) = µ  .
 a 
Therefore (2) reduces to
(µa ) = R sin α . V or µa = R 2 sin 2 α . V 2
 
µ×µ  µ 

ap
or = R 2V 2 sin 2 α ∵ from (1), a = 2 µ 
 2 2µ   V2 − 
 V −   R 
R
µ2 µ
or sin 2 α = or sin α = 1/ 2
 2µ   2µ 

l
R 2V 2 V 2 −  RV  V 2 − 
 R
α = sin −1
sC

µ
R
.
or 1/ 2
 2 2µ 
V V − 
 R
es
µ
2. A particle describes an ellipse under a force towards the focus. If it was
[ distance]2
projected with velocity V from a point distant r from the centre of force, show that its periodic
cc

time is
−3/ 2
2π  2 V 2 
 − 
µ  r µ 
Sol. Since orbit described is an ellipse, therefore
Su

 2 1 V2 2 1
V 2 = µ  −  or = −
 r a µ r a
1  2 V2
or = − . ...(1)
a r µ 
3/ 2
Now periodic time is given by T = 2πa . ...(2)
µ
Therefore from (1) and (2), we get
−3/ 2
2π  2 V 2 
T=  − 
µ r µ 
3. If a planet were suddenly stopped in its orbit supposed circular, show that it would fall
2
into the Sun in a time which is times the period of the planet’s revolution.
8
Sol. Let a be the radius circular path and the particle be suddenly stopped at point P, i.e., its
velocity suddenly reduces to zero; then the planet will begin to move in the straight line PS towards
S under the acceleration
SuccessClap: Best Coaching for UPSC Mathematics : For Info- 9346856874
Checkout ->22 Weeks Study Plan, Videos, Question Bank Solutions, Test Series

822 Dynamics
µ
(distance) 2 P
Now equation of motion of the particle while falling from P to S is
dv µ µ
v = − 2 (r decreasing) or v dv = − 2 dr.
dr r r S
2 µ
On integrating, we get v2 = + A.
r

Now when r = a, v = 0; ∴ A= −
a
2  1 1  2µ ( a − r )
Therefore v = 2µ  −  = .
r a ar
dr  a − r
∴ v= = − (2µ )   (r decreasing).
dt ar 

ap
0  r   2µ  t
∴ ∫a 
a − r
 dr = −   ∫0 dt ,
 a 
where r is time of reaching the planet to the Sun.
 2µ  0  r 

l
∴ −  t=∫ sC   dr.
 a  a
a − r
Put r = a cos 2 θ, dr = − 2a cos θ sin θ d θ.
 2µ  π/2
∴ −   t =− ∫ 2a cos 2 θ d θ.
 a 0
es
 2µ  π/2
 a  t = 2a ∫0 cos θ d θ
2
or
 
 2µ  1 π πa 3 / 2
cc

or   t = 2a . or t=
a 2 2 2 (2µ )
If T be the periodic time of planet’s revolution, then.
2 πa 3 / 2 t 2 (2µ )
Su

T= ∴ =
µ T 2πa3 / 2
t 1 2 2
∴ = = or t= T.
T 4 2 8 8
4. If the velocity of a body in an elliptic orbit, major axis 2a, is the same at a certain point P,
whether the orbit is being described in a periodic time T about one focus S or in periodic time T ′
about the other focus S ′, prove that
2aT 2aT
SP = and S ′P = .
T + T′ T + T′
Sol. As we know that sum of focal distances of any point on the ellipse is equal to 2 a , so if
SP = r , then
S ′P = 2a − r.
Now when S is the focus, then
2 1 2 πa 3 / 2
v 2 = µ  −  and T = . ...(1)
r a µ
Now when S ′ is the focus, then
SuccessClap: Best Coaching for UPSC Mathematics : For Info- 9346856874
Checkout ->22 Weeks Study Plan, Videos, Question Bank Solutions, Test Series

Planetary Motion 823


 2 1 2 πa 3 / 2
v2 = µ ′  −  and T ′ = . ...(2)
 2a − r a  µ′
∴ From (1) and (2), we get
2 1  2 1
µ  −  = µ′  − . ...(3)
 r a   2 a − r a 
4πa 2 a3 4π 2 a 3
But µ = from (1) and µ ′ = from (2)
T2 T ′2
Therefore (3) reduces to
4π 2 a 3  2 1  4π 2 a 3  2 1
2 r
− = 2  − 
T  a T ′  2a − r a 
1 2a − r 1 r
or
2
. = 2. or T 2 r 2 = T ′ 2 (2a − r ) 2
T ar T′ a (2 a − r )

ap
or Tr = T ′ (2a − r ) or (T + T ′ ) r = 2aT ′
2aT ′ 2aT ′
or r= i.e., SP = .
T + T′ T + T′
2aT ′
S ′P = 2a −

l
Therefore [as S ′P = 2a − SP]
T +T′

or
sC
S ′P =
2aT
.
T + T′
5. A particle describes an ellipse about a centre of force at the focus; show that any point of
its path, the angular velocity about the other focus varies inversely as the square of the normal at
es
that point.
2 dθ dθ P Z
Sol. r = h = pv in a central orbit where is angular Y
dt dt φ φ
velcoity about the focus S . Therefore the angular velocity (say
cc

ω ) about the other focus H by above formula is given by


HP 2 . ω = HZ . v, [∵ p = HZ ] S GC H

h
or HP 2 ω = HZ . ,
Su

SY
 h h 
∵ h = pv, ∴ v = or v = .
 p SY 
h . HZ h . HZ
∴ ω= or ω= . ...(1)
SY . HP 2 SY . HP . HP
Now since the ∆ s HZP and SYP are similar,
HZ SY
∴ = .
HP SP
Therefore from (1),
h HZ h SY h
ω= . . . = . ...(2)
SY . HP HP SY . HP SP SP . HP
But SP = a + ex ′ = a + ae cos φ and HP = (a − ae ′ ) = a − ae cos φ.
∴ from (2),
h h
ω= =
(a + ae cos φ) ( a − ae cos φ) a 2 − a 2 e 2 cos 2 φ
SuccessClap: Best Coaching for UPSC Mathematics : For Info- 9346856874
Checkout ->22 Weeks Study Plan, Videos, Question Bank Solutions, Test Series

824 Dynamics
h h
or ω= 2 = 2 . ...(3)
a − (a − b ) cos φ b cos φ + a 2 sin 2 φ
2 2 2 2

Normal at P, whose co-ordinates are (a cos φ, b sin φ), is given by


ax by
− = a 2 − b2
cos φ sin φ
or ax sec φ − by cosec φ = a 2 − b 2 .
Now above normal meets the major axis at G, where y = 0.
 a 2 − b2 
Hence co-ordiantes of G are  cos φ, 0 .
 a 
2
2
 a 2 − b2 
Therefore PG =  a cos φ − cos φ  + b 2 sin 2 φ
 a 
b2

ap
= 2 cos2 φ + b2 sin 2 φ
a
b2
or PG 2 = 2 (b 2 cos 2 φ + a 2 sin 2 φ)
a

l
b2 1
or
sC
(b2 cos2 φ + a 2 sin 2 φ) = 2 .
a PG 2
h b2 1
∴ from (3), ω= 2 = h .
b cos 2 φ + a 2 sin 2 φ a 2 PG 2
es
6. A planet of mass M and periodic time T, when at its greatest distance from the Sun, comes
into collision with a meteor of mass m, moving in the same orbit in the opposite direction with
velocity v; if m/M be small, show that major axis of the planet’s path is reduced by
4m vT  1 − e 
cc

M π  1 + e  M
Sol. Let v be the velocity of the planet and meteor at A,
before collision. Since orbit is same for planet and meteor, hence
their velocities are equal at A. A
Su

S C
2  2 1
∴ v =µ − 
 a + ae a 
µ 1− e v
= . . m
a 1+ e ...(1)
[since SA = SC + CA = ae − a]
Let V be the velocity of combined body after the collision.
Since momentum is unaltered by collision, by the principle of conservation of momentum, we
get
( M + m) V = Mv − mv
−1
M −m  m  m
or V= v = 1 −  1 +  v
M +m  M  M
 m m  m 
= 1 −  1 −  v nearly as is small
 M M  M 
SuccessClap: Best Coaching for UPSC Mathematics : For Info- 9346856874
Checkout ->22 Weeks Study Plan, Videos, Question Bank Solutions, Test Series

Planetary Motion 825


  m  
2
 m m
= 1 − − v neglecting   
  M 
M M  
 2m 
or V = 1 − v ...(2)
 M
Hence from (2), we observe that
2µ  2 1 2µ
V 2 < v 2 or V 2 < as v2 = µ  −  or V2 < ,
a + ae  a + ae a  SA
Therefore the subsequent path of the combined body is ellipse. Let its major axis be 2a′.
Now for the new path i.e., ellipse at A,
 2 1
V2 = µ −  ...(3)
 a + ae a ′ 
2
 2m  2  2 1
1 −  v = µ  − 

ap
or [from (2)]
M  a + ae a ′ 
 2   m
2 
 4m  2 1
or  1 −  v =µ −   neglecting   and higher powers
M   a + ae a ′   M 

l
 4m  1 − e  2 1
or 1 − .
M  1+ e
sC
=µ −
 a + ae a ′ 
[from (1)]

 4m  1 1 − e 2 1
or 1 −  . = −
 M  a 1 + e a + ae a ′
1 1 2 1 − e 4m 1 − e  1  4m 1 − e 
es
=  − + . = 1 + M . 1 + e 
or a ′ a  1 + e 1 + e M 1 + e  a
−1
 4m 1 − e   4m 1 − e 
a ′ = a 1 + = a 1 − . nearly.
M 1 + e  M 1 + e 
or
 
cc

4m 1 − e
∴ 2a ′ − 2a = − . 2a.
M 1+ e
or major axis of subsequent path is reduced by
Su

 4m 1 − e 
 M . 1 + e . 2a .

2πa3 / 2  µ 1 − e  2πa3 / 2  2 µ 1 − e
Also T = ; ∴ vT =  .  as v = a . 1 + e 
µ  a 1 + e  µ
1 − e vT 1 + e
or vT = 2aπ  ; ∴ 2a =  1 − e 
 1 + e  π
4m 1 − e 4m 1 − e vT  1 + e
∴ . 2a = . .  1 − e  .
M 1+ e M 1+ e π
Hence major axis of subsequent path is reduced by

4m vT 1 − e
M π  1 + e  .
SuccessClap: Best Coaching for UPSC Mathematics : For Info- 9346856874
Checkout ->22 Weeks Study Plan, Videos, Question Bank Solutions, Test Series

826 Dynamics
µ
7. A particle is moving in ellipse of eccentricity e, under the acceleration 2 to a focus; when
r
the particle is nearest to a focus, the acceleration is suddenly replaced by an acceleration µ r
towards the centre of the ellipse. If the particle continues to move in the same ellipse, prove that
µ = µ ′ (1 − e2 ) a 2 .
Sol. Let v be the velocity of particle at point A which is nearest B
to the focus S; then
 2 1
v2 = µ  −  C
 AS a  A A'
S
 2 1 
= µ −
 a − ae a 
µ 1+ e B'
or v2 = . ...(1)
a 1− e

ap
Now the centre of force is shifted to centre i.e., at point C and new force is µ′r.
Now distance of point A from C is a and velocity of particle at A is v.
du
Also = 0 at point A, being an apse at A

l
The differential equation of the path is
sC
d 2u P µ′  µ′ 
+u= 2 2 = 2 2 as P = µ ′r =  .
dθ 2
h u h u  µ
Integrating above, we get
 du  2  2µ′
v 2 = h 2   + u 2  = − 2 + A, ...(2)
es
  d θ   2u
1 du 2 µ 1+ e
Now when u = , = 0 and v = from (1),
a dθ a 1− e
cc

µ 1 + e h2
∴ . = = − µ ′ a 2 + A.
a 1 − e a2
1+ e  du  2  µ′ µ 1 + e
  + u 2  = − + µ′a 2 .
Su

Therefore au . + .
1− e 
 d θ   u2 a 1 − e
Therefore equation (2) reduces to
1 + e  du  
2
µ′ µ 1 + e
au .   + u 2  = − + . + µ′a 2 . ...(3)
1 − e  d θ   u 2 a 1− e
 
Now as the particle continues to move in the same ellipse, the next apse will be at point B i.e.,
next apse be r = b.
1 du
Therefore when r = b or u = , = 0.
b dθ
1 du
Therefore in (3), putting u = and = 0, we get
b dθ
1+ e 1 µ 1+ e
aµ . 2 = − µ ′b 2 + . + µ ′a 2
1− e b b 1− e
µ 1 + e  a2 
or . .  2 − 1 = µ ′ (a 2 − b 2 )
a 1− e  b 
SuccessClap: Best Coaching for UPSC Mathematics : For Info- 9346856874
Checkout ->22 Weeks Study Plan, Videos, Question Bank Solutions, Test Series

Planetary Motion 827

µ 1 + e (a 2 − b 2 )
or . . = µ ′ (a 2 − b 2 )
a 1 − e a 2 (1 − e2 )
µ 1+ e 1
or . . = µ′ or µ = µ ′a3 (1 − e)2 .
a 1 − e a 2 (1 − e) (1 + e)

EXERCISES
1. Show that the velocity of a particle moving in an ellipse about a centre in the focus is
µ µe
compounded of two constant velocities, perpendicular to the radius and perpendicular
h h
to the major axis. Hence deduce that the radial velocity is given by the equation
2
 dr  µ
r 2   = {a (1 + e) − r}{r − a (1 − e)}
 dt  a
2. A planet is describing an ellipse about the Sun as focus. Show that its velocity away from
the Sun is greatest when the radius vector to the planet is at right angles to the major axis of

ap
2π ae
path and that is then , where 2a is the major axis, e the eccentricity and T the
T (1 − e2 )
periodic time.

l
3. Prove that the time taken by the earth to travel over half its orbit, remote from the Sun,
sC
separated by the minor axis is 2 days more than half the year. The eccentricity of the orbit is
1
.
60
4. A body is moving in an ellipse about a centre of force in the focus. When it arrives at P, the
direction of motion is turned through a right angle, the speed being unaltered. Show that
es
the body will describe an ellipse whose eccentricity varies as the distance of P from the
centre.
5. A comet is moving in a parabola about the Sun as focus. When at the end of its latus rec-
tum its velocity suddenly becomes altered in the ratio of n :1, where n < 1. Show that the
cc

comet will describe an ellipse whose eccentricity is (1 − 2n 2 + 2n 4 ) and whose major axis
l
is where 2l was the latus rectum of the parabolic path.
1 − n2
Su

6. A particle is describing a parabola under a force to the focus. It meets and coalesces with
another particle of n times its mass which was at rest before the impact. Show that the
composite body will describe an ellipse whose eccentricity is given by
4 n ( n + 2) θ
1 − e2 = cos 2 ,
4 2
( n + 1)
where θ is measured from the apse in the parabola.
7. A particle of mass m when at any point of the ellipse of semi-major axis a is split by an
explosion into two particles of masses m1 and m2 . If the particle m1 describes a parabola,
µ m2 a E
prove the semi-major axis of the orbit described by m2 is , where is the energy
µm − aE 2
generated by the explosion.
8. A body describes an ellipse under a force to the focus and when at the extremity of the minor
axis, moving towards the nearer apse, it receives a blow in the direction of the other focus
which cause it to move towards the centre of the ellipse. Show that the eccentricity of the new
orbit is (3e2 − 3 + e − 2 )1/ 2 and that the major axis is turned through angle whose tangent is
ab
.
a (2e 2 − 1)
SuccessClap: Best Coaching for UPSC Mathematics : For Info- 9346856874
Checkout ->22 Weeks Study Plan, Videos, Question Bank Solutions, Test Series

828 Dynamics
8.6
(a) Time to Describe a given arc of Parabolic Orbit Starting from the Vertex
We know that polar equation of parabola is
1 θ P
= 1 + cos θ = 2 cos 2
r 2
l θ θ
or r = sec2 = a sec2 θ
2 2 2 A S
[as l = 2a in parabola].
2 dθ
Also we know that r =h
dt
t θ
or h dt = r 2 d θ or h ∫ dt = ∫ r 2 d θ
0 0
θ θ  2 θ
or ht = ∫ a 2 sec2 dθ as r = a sec 
0 2  2

ap
θ θ θ  θ 1 θ
or ht a 2 ∫ sec2  1 + tan 2  d θ = 2a 2  tan + tan 3 
0 2  2   3 3 2
 θ 1 θ
or (µ 2a ) t = 2a 2  tan ± tan 3  [as g = (µl ), l = 2a ]
 2 3 2

l
or t= 
 µ
sC
 2a 2  
 
θ 1
2 3
3 θ
  tan + tan  .
2

(b) To Find the Time of Description of given arc P


es
of an Elliptic Orbit Starting from the Nearer
End of the Major Axis. θ
A
Polar eqaution of an ellipse with S as focus and SA as initial C S
line is
1
cc

= 1 + e cos θ, where e < 1.


r

Also r2 = h or h dt = r 2 d θ
dt
Su

t θ θ 1
or h ∫ dt = ∫ r 2 d θ or ht = l 2 ∫ d θ. ...(1)
0 0 0 (1 + e cos θ) 2

d  sin θ  (1 + e cos θ) cos θ + e sin 2 θ


Now  =
dθ 1 + e cos θ  (1 + e cos θ) 2
e + cos θ e2 + e cos θ
= =
(1 + e cos θ) 2 e (1 + e cos θ) 2
(1 + e cos θ) (1 − e 2 )
= [as e < 1]
e (1 + e cos θ) 2
d  sin θ  1 1 − e2 1
or  = −
d θ 1 + e cos θ  e (1 + e cos θ) e (1 + e cos θ)2

1 − e2 1 d  sin θ 
or 2
= −  
e (1 + e cos θ) e (1 + e cos θ) d θ 1 − e cos θ 
On integrating above, we get
SuccessClap: Best Coaching for UPSC Mathematics : For Info- 9346856874
Checkout ->22 Weeks Study Plan, Videos, Question Bank Solutions, Test Series

Planetary Motion 829


1 − e2 1 1 1 sin θ
e ∫ (1 + e cos θ)2 dθ = ∫
e 1 + e cos θ
dθ −
1 + e cos θ
1 1 1 e sin θ
or ∫ (1 + e cos θ)2 d θ = (1 − e2 ) ∫ (1 + e cos θ) d θ − 1 − e2 . 1 + e cos θ
Also we know that
 θ 
dθ 2 −1  1 − e 
∫ 1 + e cos θ (1 − e)
=  tan  1 + e  tan
2
 if e < 1.
 
1 2 
 −1  1 − e  θ  e sin θ
∴ ∫ (1 + e cos θ)2 d θ = 2 3/ 2  tan  1 + e  tan − 2
2  1 − e (1 + e cos θ)
(1 − e )  
Therefore from (1),
θ 1
ht = l 2 ∫ d θ.
0 (1 + e cos θ) 2

ap
 sin θ  
 2 −1  1 − e  e
or ht = l 2   tan   − 
  (1 − e2 )3 / 2  1 + e  1 − e2 1 + e cos θ  
 
Now as h = (µl ) = {µ .a (1 − e 2 )},

l
and l=
b 2 a 2 (1 − e2 )
a
=
a
sC = a (1 − e2 ).

2 2a 2 (1 − e 2 ) 2 −1  1 − e  θ ea 2 (1 − e2 )2 sin θ
∴ {µ a (1 − e )} t = tan  1 + e  tan −
2 3/ 2
(1 − e )  2 1− e 2 1 + e cos θ
es
a3 / 2  1 − e θ sin θ 
or t=  2 tan −1  tan − e (1 − e 2
) .
µ   1 + e  2 1 + e cos θ 

(c) To find the time of description of a given arc of a hyperbolic orbit.
cc

Polar equation of hyperbola is


l
= 1 + e cos θ, where e > 1.
r
Also we know that
Su


r2 = h or dt = r 2 d θ
dt
t θ 2
or ∫0 h dt = ∫0 r dθ
t θ l2
or ∫0 h dt = ∫
0 (1 + e cos θ) 2
d θ. ...(1)

d  sin θ  cos θ (1 + e cos θ) + e sin 2 θ


=
d θ  1 + e cos θ 
Now
(1 + e cos θ) 2
cos θ + e cos 2 θ + e sin 2 θ e + cos θ
= 2
=
(1 + e cos θ) (1 + e cos θ) 2
e cos θ + e 2 1 + e cos θ + e2 − 1
= =
e (1 + e cos θ) 2 e (1 + e cos θ) 2
d  sin θ  1 e4 − 1
= +
d θ  1 + e cos θ  e (1 + e cos θ) e (1 + e cos θ) 2
or (because e > 1)
SuccessClap: Best Coaching for UPSC Mathematics : For Info- 9346856874
Checkout ->22 Weeks Study Plan, Videos, Question Bank Solutions, Test Series

830 Dynamics
1 d  sin θ 
e 1 1
or =  1 + e cos θ  − 2
(1 + e cos θ) 2
e −1 d θ 
2
e −1 (1 + e cos θ)
Integrating both sides of above, we get
1 e sin θ 1 1
∫ (1 + e cos θ)2 d θ = e2 − 1 1 + e cos θ − e2 − 1 ∫ (1 + e cos θ) d θ.
Therefore (1) reduces to
t 2 e sin θ l2 θ 1
ht = 2 − 2 ∫
(e − 1) (1 + e cos θ) e − 1 0 (1 + e cos θ)

1
(e + 1) + (e − 1) tan θ
t 2 e sin θ l2 1 2
or ht = − . log
(e2 − 1) (1 + e cos θ) (e2 − 1) (e2 − 1)
1
(e + 1) − (e − 1) tan θ
2
as e > 1

ap
 1 
 (e − 1) + (e − 1) tan θ
l2 e sin θ 1 2 
or t=  2 − 2 3/ 2
log
1 .
h  (e − 1) (1 + e cos θ) (e − 1) (e + 1) − (e − 1) tan θ
 2 

l
l2 l2 l3 / 2 a3 / 2 (e2 − 1)3 / 2
Now = = =
h ( µl )

µ
sC µ
1 
(e + 1) + (e − 1) tan θ
a 2 / 3  sin θ 2 
∴ t= e (e − 1) − log
1 
µ  1 + e cos θ
(e + 1) − (e − 1) tan θ 
es
 2 

EXAMPLES
1. Find the time T of describing an arc C of a parbolic orbit Newtonian law; if C be bounded
cc

by a focal chord, prove that


T ∝ ∫ ( focal chord )3/ 2 . P
Sol. We have already found the time of description of given arc of
parabolic orbit as S θ
Su

π–θ A
 2a 3   1 1 31 
or    tan 2 θ + 2 tan 2 θ  .
 µ   
Q
Let there be a focal chord PQ inclined at an angle θ to the initial
line, so that the vectorial angle of Q will be (π − θ) below the initial line.
Therefore the time of describing the arc bounded by the focal chord PQ is
 2a 3   1 1 31 
θ
T=    tan θ + tan θ 
 µ   2 2 2  − ( π − θ)

 2a 3   1 1 31 1 1 31 
=    tan 2 θ + 3 tan 2 θ + cot 2 θ + 3 cot 2 θ 
 µ   
 

 2a 3   1 1 1 3 θ θ 
=    tan θ + cot θ +  tan + cot 3  
 µ   2 2 3 2 2 
SuccessClap: Best Coaching for UPSC Mathematics : For Info- 9346856874
Checkout ->22 Weeks Study Plan, Videos, Question Bank Solutions, Test Series

Planetary Motion 831


 21 2 1 6 1 2 1 
 2a 3  sin 2 θ + cos 2 θ 1 sin 2 θ cos 2 θ 
=   + 
 µ   sin 1 θ cos 1 θ 3 1 1
  sin 3 θ cos3 θ 
 2 2 2 2 
  21 1  1 1 1 1 
sin θ + cos 2 θ   sin 4 θ − sin 2 θ cos 2 θ + cos 4 θ  
 2a 3   2 
+ 
8 2 2  2 2 2 2 
=  
 µ   sin θ 3 sin 3
θ 
  
 
 2a3   2  2 

8 2 1 2 1  2 1 2 1 
=   +   sin θ + cos θ  − 3 sin θ cos θ 
 µ   sin θ 3 sin 3 θ  2 2  2 2 


 2a   2
3
8  2 1 1  
=    + 1 − 3 sin θ cos 2 θ 
 µ   sin θ 3 sin θ 3 2 2  

ap
  3  
3 sin θ + 4 1 − sin 2  
 2a3    4   2a   3 sin 2 θ + 4 − 3 sin 2 6 
= 2    = 2  µ   
 µ   3 sin 3 θ   3sin 3 θ 
 

l
or
 2a 3  4
T =2   cosec θ =
sC
1  2 2 3/ 2
 µ  [4a cosec θ]
 µ  3 3
1  2
= =   [focal chord]3 / 2 .
es
3  µ
l l
Since PQ = SP + SQ = +
1 + cos 1 + cos (π + θ)
[as vectorial angle of Q = π + θ measured anti-clockwise]
cc

l l
= + = 2l cosec2 θ
1 + cos θ 1 − cos θ
or PQ = 4a cosec2 θ (as l = 2a ).
Su

1  2
∴ T=   [focal chord]3 / 2
3  µ
or T ∝ (focal chord)3 / 2
2. Prove that time taken to describe two portions into which an ellipse is divided by the latus
rectum through the centre of the force are in a ratio
{cos −1 e − e (1 − e 2 )} :{π − cos −1 e + e (1 − e 2 )}.
Sol. We have already found the time t of describing an arc of elliptic orbit given by

θ
A' A
C S

L'
SuccessClap: Best Coaching for UPSC Mathematics : For Info- 9346856874
Checkout ->22 Weeks Study Plan, Videos, Question Bank Solutions, Test Series

832 Dynamics

a3 / 2  1 − e θ sin θ 
t=  2 tan −1  tan − e (1 − e 2
) 
µ   1 + e  2 1 + e cos θ 

Now in the above formula if we put θ = π / 2 it will give us time of description of arc AL : let it
be t1.

a3 / 2   1− e  π sin (π / 2) 
∴ t1 =  2 tan −1  tan − e (1 − e 2
) 
µ   1 + e  4 1 + e cos (π / 2) 

a3 / 2  1 − e 
t1 =  2 tan −1  − e (1 − e ) .
or µ   1 + e  

ap
1− e  z −1  1 − cos z 
Now 2 tan −1  −1
 = 2 tan tan = tan   , by putting e = cos z
 1 + e  2  1 + cos z 

l
1 − e
or 2 tan −1 
 1 + e 
z
sC
= 2, = z = cos −1 e.
2
Therefore (1) reduces to

a3 / 2
es
t1 = [cos −1 e − (1 − e2 ) ]
µ

The time of describing the arc LA′ L ′ = 2t1


cc

or time of describing the arc LAL′

2a 3 / 2
= [cos −1 e − e (1 − e2 ) ].
µ
Su

2 πa 3 / 2
Now since the period to describe an ellipse is
µ
Therefore the time to describe the arc LA′ L ′ is

2a 3 / 2 2a 3 / 2
− [cos −1 e − e (1 − e 2 ) ]
µ µ

2a 3 / 2
or [π − cos −1 e + e (1 − e 2 ) ].
µ

Hence the ratio of times of describing the arcs LA′ L and LA′ L ′ is

[cos −1 e − e (1 − e 2 ) ] : [π − cos −1 e + e (1 − e 2 ) ].
SuccessClap: Best Coaching for UPSC Mathematics : For Info- 9346856874
Checkout ->22 Weeks Study Plan, Videos, Question Bank Solutions, Test Series

Planetary Motion 833


EXERCISES
1. Prove that in a parabolic orbit the time taken to move from the vertex to a point distant r
from the focus is
1
(r + l ) (2r − l ),
3 µ
where 2l is latus rectum.
1
2. If the period of a planet be 365 days and the eccentricity e = , show that times of
60
describing the two halves of the orbit bounded by the latus rectum through the centre of
force are
365  1 
1 ±  nearly.
2  15 π 
1

ap
3. The perihelion distance of a planet describing a parabolic orbit is of the radius of the
n
earth’s path supposed circular, show that the time that the comet remain with the earth’s
orbit is
2 n + 2  n − 1
  of a year .

l
3π nsC 2n 
2
Also prove that the longest time that the comet will remain within the earth’s orbit is of

an year.
❑❑❑
es
cc
Su
SuccessClap: Best Coaching for UPSC Mathematics : For Info- 9346856874
Checkout ->22 Weeks Study Plan, Videos, Question Bank Solutions, Test Series

Motion in Three Dimensions


9.1 ACCELERATION OF A PARTICLE IN POLAR CO-ORDINATES
To find the acceleration of particle in terms of polar Z
coordinates. L
Let at any instant particle be at P whose coordinates be P
r, θ and φ , where r is the distance of a P from a fixed origin
r
O, θ is the angle that OP makes with z-axis and φ is the

ap
angle that the plane ZOP makes with a fixed plane ZOX (Z-X θ
O M
plane). φ X
Draw PN perpendicular to the plane XOY and let ON = ρ.
P
N K
d2x d2y d2z

l
Then the acceleration of P are , and , Y
sC dt 2 dt 2
where x, y and z are the coordinates of P.
dt 2

The polar coordinates of N, which in always is the plane XOY are ρ and φ. This point N will
have radial and transverse accelerations whose magnitudes are
FG dφ IJ
es
2
d 2ρ
dt 2 H dt K
−ρ along ON ,

1 d F dφ I
and G
H ρ 2
J
dt K
perpendicular to ON
cc

ρ dt

d2z
Also the acceleration of P relative to N is along NP.
dt 2
Su

Hence, the acceleration of P are


d 2ρ FG dφ IJ along LP,
2

dt 2
−ρ
H dt K
1 d F dφ I
ρ dt
G
H ρ 2
dt K
J perpendicular to the plane ZPK
d2z
and parallel to OZ .
dt 2
d2z d 2ρ
Since z = r cos θ and ρ = r sin θ, the acceleration and
, along and perpendicular to
dt 2 dt 2
d 2r dθ
2
FG IJ
OZ is the plane ZPK, are equivalent to
dt 2
−r
dt H K
along OP (radial acceleration) and

1 d FG
r2
dθ IJ
r dt H dt K
perpendicular to OP (transverse acceleration) in the plane ZPK.
SuccessClap: Best Coaching for UPSC Mathematics : For Info- 9346856874
Checkout ->22 Weeks Study Plan, Videos, Question Bank Solutions, Test Series

Motion in Three Dimensions 835


FG dφ IJ 2
FG dφ IJ 2
Also the acceleation − ρ
H dt K along LP is equivalent to − ρ sin θ
H dt K along OP and

FG dφ IJ 2
− ρ cos θ
H dt K perpendicular to OP.

Hence, if α, β, γ be the accelerations of P respectively along OP, perpendicular to OP in the


plane ZPK in the direction of θ increasing, and perpendicular to the plane ZPK in the direction of
φ increasing, we have
d 2r FG dφ IJ − ρ sin θ FG dφ IJ
2 2
α=
dt 2
−r
H dt K H dt K
d 2r F dφ I 2
− r G J − r sin 2 θ G J
F dφ I 2
=
dt 2 H dt K H dt K
1 d F 2 dθ I F dφ I
2

r dt H
G dt JK GH dt JK

ap
β= r − ρ cos θ

1 d F 2 dθ I F dφ I 2
=
r dt H
G r
dt K
J − r sin θ cos θ G J
H dt K ...(2)

1 d F dθ I d F IJ

l
and γ=
ρ dt H
sC G ρ dt JK = r sin θ dt GH r sin
2 1 dφ
dt
2 2
θ
...(3)K
Cor. Acceleration in Cylindrical Coordinates : Sometimes we consider the motion of P with
reference to the coordinates z, ρ and φ , called cylindrical coordinates.
As in 9.1, the accelerations are then
FG dφ IJ along LP,
es
2
d 2ρ
dt 2 H dt K
−ρ

1 d F dφ I
G
ρ dt H
ρ 2
J perpendicular to the plane ZPK
dt K
cc

d2z
and parallel to OZ.
dt 2
Su

9.2
A particle is attached to one end of a string of length l, the other end of which is tied to a
fixed point O. When the string in inclined at an acute angle α to the downward-drawn vertical
the particle is projected horizontally and perpendicular to the string with a velocity V; to find
the resulting motion.

N
φ

mg sinθ
mg cosθ
mg
SuccessClap: Best Coaching for UPSC Mathematics : For Info- 9346856874
Checkout ->22 Weeks Study Plan, Videos, Question Bank Solutions, Test Series

836 Dynamics
Taking O as origin, the components of weight mg of the particle are mg cos θ (along r),
− mg sin θ (perpendicular to OP) and 0 for P, we have r = l. Hence, the equations of motion will
LM
be as
dl d 2 l
= =0
OP
MN dt dt 2 PQ
FG dθ IJ 2
FG dφ IJ
− l sin 2 θ
2
T
−l
H dt K H dt K =
m
+ g cos θ ...(1)

d θ 2
F dφ I
− l cos θ sin θ G J
2
...(2)
l
dt 2 H dt K = − g sin θ

1 d F dφ I
and G sin θ dt JK = 0
sin θ dt H
2
...(3)
Equation (3) gives
LM OP

ap
dφ dφ
sin 2 θ = constant = sin 2 α
dt dt at 0 N Q
V sin α
= ...(4)
l

l
sC dφ V sin α
∴ =
dt l sin 2 θ

Substituting the value of in (2), we get
dt
d 2 θ V 2 sin 2 α cos θ g
es
− = − sin θ ...(5)
2 2 3 l
dt l sin θ

Multiplying by 2 , and integrating, we get
dt
cc

dθ d 2 θ 2V 2 sin 2 α cos θ
z 2
dt dt 2
dθ −
l2
z sin 3 θ
dθ = − z 2g
l
sin θ dθ

FG dθ IJ 2
V 2 sin 2 α 1 2g
i.e.,
H dt K + . = cos θ + A,
Su

2 sin θ l
l

at O, we have θ = α, = 0, we get
dt
V 2 sin 2 α 1 2g
0+ 2
. 2
= cos α + A ...(6)
l sin α l
Subtracting (6) from (5), we get
FG IJ

2
V 2 sin 2 α F 1 1 I
2g
H K
dt
=
l 2 GH 2

sin α sin θ 2
− JK
l
(cos α − cos θ)

V sin α F sin θ − sin α I 2 g


2 2 2 2
=
l
GH sin α sin α JK − l (cos α − cos θ)
2 2 2

V F cos α − cos θ I 2 g
2 2 2
=
l H
G sin θ JK − l (cos α − cos θ)
2 2

2g L V 2 F cos α − cos θ I − 1OP


(cos α − cos θ) M
=
l MN 2gl GH sin 2 θ JK PQ
SuccessClap: Best Coaching for UPSC Mathematics : For Info- 9346856874
Checkout ->22 Weeks Study Plan, Videos, Question Bank Solutions, Test Series

Motion in Three Dimensions 837


2g F cos α + cos θ I
=
l GH
(cos α − cos θ) 2n 2
sin 2 θ
JK
− 1 where V 2 = 4 lg n 2 .


Hence, is again zero-when
dt
2n 2 (cos α + cos θ) = sin 2 θ
i.e., when 2n 2 cos α + 2n 2 cos θ = 1 − cos 2 θ
i.e., when cos 2 θ + 2n 2 cos θ + 2n 2 cos α − 1 = 0
i.e., when cos θ = − n 2 ± (1 − 2 n 2 cos α + n 4 )

The negative sign gives inadmissisable value for θ. The only inclination at which again
dt
vanishes is when θ = θ1, when
cos θ1 = − n 2 + (1 − 2 n 2 cos α + n 4 )

ap
The motion is therefore confined between value α and θ1 of the θ.
The motion of the particle is along above or below the starting point, according as θ1 ≥ α,
i.e., according as cos θ1 ≤ cos α,

l
i.e., according as
sC
(1 − 2n 2 cos α + n 4 ) ≥ n 2 + cos α,
i.e., according as 1 − 2n 2 cos α ≥ cos α + 2n 2 cos α
2 sin 2 α
i.e., according as n ≥
es
4 cos α
V2 sin 2 α
i.e., according as ≥
4l g 4 cos α
cc

i.e., according as V 2 ≤ l g sin α tan α.


The tension of the string at any instant is now given by equation (1). Let T does not vanish
during the motion.
The square of the velocity at any instant
FG dθ IJ + FG l sin θ dφ IJ
Su

2 2

H dt K H
= l
dt K
LF dθ I 2 F dφ I 2 O
= l 2 MG J + G J sin 2 θP
MNH dt K H dt K PQ
Hence, the principle of conservation of energy gives
1 2 LF dθ I F dφ I 2 O 1
2
ml MG J + G J sin 2 θP = mV 2 − mgl (cos α − cos θ)
2 MNH dt K H dt K PQ 2
Hence, from (1), we get
T (velocity) 2 V 2 − 2 gl (cos α − cos θ)
= g cos θ + = g cos θ +
m l l
V2
= + g (3 cos θ − 2 cos α ).
l
SuccessClap: Best Coaching for UPSC Mathematics : For Info- 9346856874
Checkout ->22 Weeks Study Plan, Videos, Question Bank Solutions, Test Series

838 Dynamics
EXAMPLES
1. A heavy particle moves in a smooth sphere. Show that, if the velocity be that due to the
level of the centre, the reaction of the surface will vary as the depth below the centre.
Solution. At time t, let P be the position of the particle of mass
m, such that OP makes an angle θ with the vertical through O, the A
centre of the sphere. Let PM be the perpendicular from P on the
vertical line AA′. Then velocity of the particle at P
O
= ( 2 g . OM ) = (2 ga cos θ) θ
M P
where a is the radius of the the sphere.
θ
Let R be the reaction of the surface at P, then OP = a, the radius R
A'
of the sphere being constant. Hence, the equations of motion are mg

FG dθ IJ 2
FG dθ IJ = − R + g cos θ
− a sin 2 θ
2
−a
H dt K H dt K m ...(1)

ap
d θ 2
F dθ I
− a cos θ sin θ G J = − g sin θ
2
a
dt 2 H dt K ...(2)

d F dφ I
. G sin θ . J = 0
1

l
2
sin θ dt H dt K
and ...(3)
sC
Integration of equation (3), gives

sin 2 θ . = A (constant)
dt
dθ dr
es
Let when θ = α, = 0, = 0.
dt dt
LM
a sin θ
dφ OP
= V, i. e.,

=
V
∴ N dt θ = α Q dt a sin α
cc

V V sin α
so that A = sin 2 α . =
a sin α a
dφ V sin α
and hence sin 2 θ = ...(4)
dt a
Su


Substituting the value of from (4) to (2), we get
dt
d 2θ v 2 sin 2 α g
− cos θ sin θ . = − sin θ
2 2 4 a
dt a sin θ
d 2θ V 2 cos θ g
or − sin 2 α . = − sin θ
2 2 3 a
dt a sin θ

Multiplying the equation by 2 and integrating, we get
dt
FG IJ

2
V 2 sin 2 α 1 2g
H K
dt
+
a 2
.
2
sin θ
=
a
cos θ + B


when θ = α, = 0, then
dt
V 2 2g
= cos α + B
a2 a
SuccessClap: Best Coaching for UPSC Mathematics : For Info- 9346856874
Checkout ->22 Weeks Study Plan, Videos, Question Bank Solutions, Test Series

Motion in Three Dimensions 839


V2 2g
i.e., B= − cos α
2 a
a
FG dθ IJ 2 2
V sin α 2
1 2g V 2 2g
Thus, H dt K +
a2
.
sin 2 θ
=
a
cos θ +
a2

a
cos α

FG dθ IJ 2
V2 F sin 2 α − 1I − 2g (cos α − cos θ)
or H dt K =− G
a 2 H sin 2 θ
JK a
V2 2g
=− (sin 2 α − sin 2 θ) − (cos α − cos θ)
2 2 a
a sin θ
V2 2g
= (sin 2 θ − sin 2 α ) − (cos α − cos θ)
2 2 a
a sin θ
FG dθ IJ 2
FG dφ IJ 2
Substituting the values of
H dt K and
H dt K in (1), we get

ap
R V2
= (sin 2 θ − sin 2 α ) − 2 g (cos α − cos θ)
m a sin 2 θ
aV 2 sin 2 α sin 2 θ

l
+ + g cos θ

V2
sC a 2 sin 4 θ
V 2 sin 2 α
= (sin 2 θ − sin 2 α ) − 2 g (cos α − cos θ) + + g cos θ
a sin 2 θ a sin 2 θ
V2
es
= − 2 g cos α + 3g cos θ
a
But when θ = α, V = ( 2 ga cos α )
R 3g 3g
= 3g cos θ or R = ( a cos θ) =
cc

∴ . OM
m am am
i.e., R ∝ OM.
Hence, the reaction of the surface varies as the depth below the centre.
2. A particle moves on a smooth sphere under no force except the pressure of the surface.
Su

Show that the path is given by the equation cot θ = cot β cos φ, where θ and φ are its angular
coordinates.
Solution. Since there are no forces, except the pressure, the equations of motion are

FG dθ IJ 2
FG dφ IJ = R ,
+ a sin 2 θ
2
a
H dt K H dt K m ...(1)

d 2θ F dφ I 2
− a cos θ sin θ G J = 0
a
dt 2 H dt K
d 2θ F dφ I 2
= sin θ cos θ G J
i.e.,
dt 2 H dt K ...(2)

1 d F
and
sin θ dt H
G sin θ ddtφ IJK = 0 2
...(3)
Here a is the radius of the sphere.
SuccessClap: Best Coaching for UPSC Mathematics : For Info- 9346856874
Checkout ->22 Weeks Study Plan, Videos, Question Bank Solutions, Test Series

840 Dynamics
Integration of (3) gives

sin 2 θ = A (constant)
dt
dφ A
i.e., = ...(6)
dt sin 2 θ


By putting the value of from (4) in (2), we get
dt
d 2θ A2 A 2 cos θ
= sin θ cos θ . =
dt 2 sin 4 θ sin 3 θ

Multiplying both sides by 2 dt and integrating, we get
dt
FG IJ
2
A2

ap

dt H K
=−
sin 2 θ
+ B.


Initially, when θ = β, = 0.
dt

l
A2

sC B=
sin 2 β
,

FG dθ IJ 2 = A2 FG 1 − 1 IJ
Thus,
H dt K H sin 2 β sin 2 θ K
es
Dividing (4) by (5), we have
dφ sin β cosec 2 θ
= =
dt sin θ (sin 2 θ − sin 2 β) cot 2 β − cot 2 θ
cc

cot θ
Integrating, φ = cos −1 +C
cos β
If φ = 0 when θ = β, ∴ C = 0.
FG
cot θ IJ
Su

φ = cos −1
Hence,
cot βH K
cot θ
or cos φ = or cot θ = cot β cos φ
cot β
3. A smooth circular cone of angle 2α, has its axis vertical and its vertex, which is pierced
with a small hole, downwards. A mass M hangs at rest by a string which passes through the
vertex, and a mass m attached to the upper end describes a horizontal circle on the inner surface
of the cone. Find the time T of a complete revolution and show that small oscillations about the
steady motion take place in the time
FG M + m IJ .
T cosec α
H 3m K
Solution. Let T1 be the tension is the string and l the total length of the string. Let P be the
position of mass m in the initial state such that the length OP = r, O being the vertex of the cone.
Therefore, the length of the hanging portion of the string below O is (l − r ).
SuccessClap: Best Coaching for UPSC Mathematics : For Info- 9346856874
Checkout ->22 Weeks Study Plan, Videos, Question Bank Solutions, Test Series

Motion in Three Dimensions 841

dθ d 2θ
In this case, θ = α, hence, = 0, = 0. Hence, equations of motion will be
dt dt 2
LM d r F dφ I O 2
− r sin α G J P = − T − mg cos α
2
2
m
MN dt 2 H dt K PQ 1 ...(1)

d F dφ I
. G r 2 sin α J = 0
1
r sin α dt H dt K
...(2)

d2
and M (l − r ) = Mg − T1 ...(3)
dt 2
d 2r
The equation (3) gives −M = Mg − T1 ...(4)
dt 2
subtracting (4) from (1), we get
d 2r FG dφ IJ 2

ap
− mr sin 2 α
( M + m)
dt 2 H dt K = − mg cos α − Mg ...(5)
The equation (2) gives

r2 =A ...(6)

l
dt
For the steady motion,
sC d 2r
= 0, r = d
dt 2
dφ N φ
and therefore, (5) and (6), give on putting = ω,
es
dt P
mω 2 d sin 2 α = g ( M + m cos α ), ...(7) T1
2 α
and A=d ω mg ...(8)

cc

Also the time period T is given by T = . ...(9) O


ω T1
d 2 r d 2ρ dφ A d 2ω d 2ω ρ
−2 Mg FG IJ
If we put r = d + ρ,
dt 2
=
dt 2
and = =
dt r 2 ( d + ρ) 2
=
d2
1+
d H K
LM OP
Su


= ω 1−
Now we have from (5)
N d Q
neglecting squares of ρ as ρ is small

d 2ρ
= m ( d + ρ) sin 2 α ω 2 1 +

2
FG IJ
(m + M )
dt 2 d H
− g ( M + m cos α )
K
= mω 2 ( d + ρ) 1 −
4ρ FG IJ
sin 2 α − mω 2 d sin 2 α
d H K
from (7) and also neglecting the squares of ρ
= mω sin α ( d − 3ρ) − mω 2 d sin α, neglecting squares of ρ
2 2

= − 3mω 2 sin 2 α . ρ.
This represents a S.H.M. of time period
M+m 2π FG M + m IJ
= 2π
2 2
2 mω sin α
=
ω
cosec α
H 3m K
FG M + m IJ
= T cosec α
H 3m K [from (9)]
SuccessClap: Best Coaching for UPSC Mathematics : For Info- 9346856874
Checkout ->22 Weeks Study Plan, Videos, Question Bank Solutions, Test Series

842 Dynamics
EXERCISES
1. A heavy particle is projected horizontally along the inner surface of a smooth spherical shell
a 2a
of radius with velocity 2 ag at a depth below the centre. Show that it will rise to
2 3 3
a
a height above the centre, and that the pressure on the sphere just vanishes at the height
3
point of the path.
2. A heavy particle is projected with velocity V from the end of a horizontal diameter of a sphere
of radius a along the inner surface, the direction of projection making an angle β with the
FV I 2
2
equator. It the particle never leaves the surface prove that 3 sin 2 β < 2 + GH 3ga JK .

3. A particle constrained to move on a smooth spherical surface is projected horizontally from


a point at the level of the centre so that its angular velocity relative to the centre is ω. If

ap
ω 2 α be very great compared with g, show that its depth c below the level of the centre at
2g ωt
time t is sin 2
approximately..
2 2
ω
4. A Smooth hollow right circular cone is placed with its vertex downwards and axis vertical,

l
and at a point on its interior surface at a height h above the vertex a particle is projected
sC
horizontally along the surface with a velocity
2 gh
. Show that the lowest point of path
n2 + n
4
will be at a height above the vertex of the cone.
es
n
5. A particle is projected horizontally along the interior surface of a smooth hemisphere whose
axis is vertical and whose vertex is downwards. The point of projection being at the angular
distance β from the lowest point. Show that the particle may just ascend to the rim of the
cc

hemisphere is ( 2 ag sec β).


6. A particle moves on the inner surface of a smooth cone, of vertical angle 2α, being acted
on by a force towards the vertex of the cone, and its direction of motion always cuts the
generators at a constant angle β. Find the motion and the law of force.
Su

1
[Ans. F ∝ , r = r0e sin α cot β . φ ]
r3
9.3 ACCELERATIONS ALONG THE TANGENT, THE PRINCIPAL NORMAL
AND THE BINORMAL
A particle is moving along any curve in three dimensions; to find its accelerations along (i)
the tangent to the curve, (ii) the principal normal, and (iii) the binormal.
Let (x, y, z) be the coordinates of the point at time t. The accelerations parallel to the axes of
d2x d2y d2z
coordinates are , and .
dt 2 dt 2 dt 2
dx dx ds
Now, = .
dt ds dt
d2x FG IJ
dx d 2 s d 2 x ds
2

dt 2
= .
ds dt 2
+
H K
ds 2 dt
...(1)
SuccessClap: Best Coaching for UPSC Mathematics : For Info- 9346856874
Checkout ->22 Weeks Study Plan, Videos, Question Bank Solutions, Test Series

Motion in Three Dimensions 843

d2y FG IJ 2
dy d 2 s d 2 y ds
So,
dt 2
=
H K
.
ds dt 2
+
ds 2 dt
...(2)

d 2 z dz d 2 s d 2 z F ds I
2
and . + G
dt 2 ds dt 2 ds 2 H dt K
= J ...(3)

dx dy dz
(i) The direction cosines of the tangent are , and ,
ds ds ds
Hence, the acceleration along it
dx d 2 x dy d 2 y dz d 2 z
= . + . + .
ds dt 2 ds dt 2 ds dt 2
LM
dx dx d 2 s d 2 x ds
2
FG IJ OP
dy dy d 2 s d 2 y ds LM FG IJ OP 2
= .
ds ds dt 2 MN +
ds 2 dt
+ .
H K PQ
+
ds ds dt 2 ds 2 dt MN H K PQ
LM FG IJ OP

ap
2
dz dz d 2 s d 2 z dz
+
MN
. +
ds ds dt 2 ds 2 ds H K PQ
d 2s LMF dx I F dy I F dz I OP
2 2 2
= G J + GH ds JK + GH ds JK P
dt MNH ds K

l
2
Q
sC F ds I L dx d x + dy . d y + dz d z OP ...(4)
2 2 2 2
+G J M .
H dt K MN ds ds ds ds ds ds PQ 2 2 2

FG dx IJ + FG dy IJ + FG dz IJ = 1,
2 2 2
es
We have
H ds K H ds K H ds K
differentiating w.r.t. s, we get
dx d 2 x dy d 2 y dz d 2 z
+ . + . =0
cc

...(5)
ds ds 2 ds ds 2 ds ds
By putting these values in equation (4), we get
d 2s
Acceleration along the tangent = ...(6)
Su

dt 2
d2x d2y d2z
(ii) The direction cosines of the principal normal are ρ ,ρ and ρ , where ρ is
ds 2 ds 2 ds 2
the radius of curvature.
Hence, the acceleration along principal normal
d2x d2x d2y d2y d2z d2z
=ρ . +ρ . +ρ .
ds 2 dt 2 ds 2 dt 2 ds 2 dt 2
LM
d 2 x dx d 2 s d 2 x ds FG IJ OP + ρ d y LM dy . d s + d y FG ds IJ OP
2 2 2 2 2

ds 2 ds dt 2
+
MN
ds 2 dt H K PQ ds MN ds dt ds H dt K PQ
2 2 2

d z L dz d s d z F ds I O
2
M . + GH dt JK PP
2 2 2

ds MN ds dt ds 2
Q 2 2

[from (1), (2) and (3)]


SuccessClap: Best Coaching for UPSC Mathematics : For Info- 9346856874
Checkout ->22 Weeks Study Plan, Videos, Question Bank Solutions, Test Series

844 Dynamics


LM
d 2 s dx d 2 x dy d 2 y dz d 2 z
. + . + .
OP
MN
dt 2 ds ds 2 ds ds 2 ds ds 2 PQ
F ds I 2 LMF d x I F d y I F d z I
2
2
2
2
2
2 OP
+ρG J
H dt K MMGH ds JK + GH ds JK + GH dx JK
2 2 2 PP ...(7)
N Q
LF d x I F d y I F d z I
2 2 2 OP 1
We have MG
2 2 2

MMH dt JK + GH dt JK GH dt JK
2 2 2 PP = ρ .
2
N Q
and putting the value from (5), we get
Acceleration along the principal normal
FG ds IJ 2 × 1 = 1 FG ds IJ 2

H dt K ρ2 ρ H dt K ...(8)

(iii) The direction cosines of the binormal are proportional to

ap
dy d 2 z dz d 2 y dz d 2 z dx d 2 z dx d 2 y dy d 2 x
− . , . − . and . − . .
ds ds 2 ds ds 2 ds ds 2 ds ds 2 ds ds 2 ds ds 2
On multiplying (1), (2) and (3) in succession by these and adding, the result is zero, i.e., the

l
acceleration in the direction of the binormal vanishes
sC
Remark : Let (l1, m1, n1 ), (l2 , m2 , n2 ) and (l3 , m3 , n3 ) are the direction cosines of the tangent,
the principal normal, and the binormal, then equations (1), (2), (3) can be written as
d 2s R| 1 F ds I U|
d 2s
2
S| ρ GH dt JK V|
= l1 + l2
dt dt 2
T W2
es
2 R
| 1 F ds I U|
2 2
+m S G J V
d y d s
|T ρ H dt K |W
=m 1 2
2 2
dt dt
2 R| 1 F ds I U|2 2
cc

+n S G J V
d z d s
|T ρ H dt K |W
and =n 1 2
2 2
dt dt
These equations show that the accelerations along the axes are the components of an
along the tangent, an acceleration 1 FG ds IJ along the principal normal, and
d 2s 2
Su

acceleration
dt 2 ρ H dt K
nothing is the direction of the binomal.
dv v2
Hence, for a particle describing a v , along the tangent and along the principal normal,
dt ρ
which lies in the osculating plane of the curve.
9.4
A particle moves in a curve, there being no friction, under force such as occur in nature.
Show that the change in its kinetic energy as it passes from one position to the other is independent
of the path pursued and depends on its initial and final positions.
Proof : Let X, Y, Z be the components of the forces. Resolving the forces along the tangent to
the path, we have
d 2s dx dy dz
m =X +Y +Z
2 ds ds ds
dt
ds
Multiplying by 2 and integrating, we get
dt
SuccessClap: Best Coaching for UPSC Mathematics : For Info- 9346856874
Checkout ->22 Weeks Study Plan, Videos, Question Bank Solutions, Test Series

Motion in Three Dimensions 845

FG ds IJ = 2 z ( X dx + Y dy + Z dz)
2

H dt K
m

1 F ds I
2
m G J = z ( X dx + Y dy + Z dz )
2 H dt K

Since the forces are such as occur in nature, hence the components are one-valued functions
of distances from fixed points. So, the quantity X dx + Y dy + Z dz is the differential of some
function φ ( x, y, z), so that
1 1
mv 2 = m
dx FG IJ 2

2 2 dt H K = φ ( x, y, z ) + C

1
where mv02 = φ ( x 0 , y0 , z0 ) + C.
2
Here ( x 0 , y0 , z0 ) is the starting point an v0 the initial velocity.

ap
1 1
Hence mv 2 − mv02 = φ ( x, y, z ) − φ ( x 0 , y0 , z0 )
2 2
The right-hand member of this equation depends only on the position of the initial point and
on that of the point of the path under consideration, and is quite independent of the path pursued.

l
9.5 MOTION ON A SMOOTH SURFACE
sC
If the particle moves on a surface whose equation in f ( x, y, z) = 0, let the direction-cosines of
the normal point (x, y, z) of its path be (l1, m1 , n1 ) so that
l1 m1 n1
es
= =
df / dx df / dy df / dz
(l12 + m12 + n12 )
=
[( df / dx ) 2 + ( df / dy) 2 + ( df / dz ) 2 ]
cc

1
=
...(1)
[( df / dx ) 2 + ( df / dy) 2 + ( df / dz ) 2 ]
Now, if R be the normal reaction, we have
Su

d2x d2y d2z


m 2 = X + Rl1 , m 2 = Y + Rm1 and m 2 = Z + Rn1 ,
dt dt dt
where X, Y, Z are the components of the impressed forces.
dx dy dz
Multiplying these equations by , , and adding, we have
dt dt dt
LMF I F I F I OP
2 2 2

MNGH JK GH JK GH JK PQ
1 d dx dy dz dx dy dz
m + + =X +Y +Z
2 dt dt dt dt dt dt dt
for the coefficient of R
dx dy dz
= l1 + m1 + n1
dt dt dt
FG dx dy dz ds IJ
= l1
H ds
+ m1
ds
+ n1
ds dt K
ds
= (the cosines of the angle between a tangent line to the surface and
dt
the normal)
= 0.
SuccessClap: Best Coaching for UPSC Mathematics : For Info- 9346856874
Checkout ->22 Weeks Study Plan, Videos, Question Bank Solutions, Test Series

846 Dynamics
Hence, on integration,
1
2
z
mv 2 = ( X dx + Y dy + Z dz )
Also, on eliminating R, the path on the surface is given by
d2x d2y d2z
m −X m −Y n −Z
dt 2 = dt 2 = dt 2
l1 m1 n1

EXAMPLES
1. A smooth helix is placed with its axis vertical and a small bead slides down it under gravity.
FG πa IJ
Show that it makes its first revolution from rest in time 2
H g sin α cos α K where α is angle of

the helix.

ap
Solution. Equations of a helix are
x = a cos θ, y = a sin θ, z = a θ tan α,
dx dθ dy dθ dz dθ
so that = − a sin θ , = a cos θ , = tan α .
dt dt dt dt dt dt
FG IJ FG IJ FG IJ FG IJ

l
2 2 2 2
ds dx dy dz
H K H K H K H K
∴ dt
=
dt
sC
+
dt
+
dt

= [a 2 sin 2 θ + a 2 cos 2 θ + a 2 tan 2 α ]


FG dθ IJ 2

H dt K
FG dθ IJ 2
es
= a 2 (1 + tan 2 α )
H dt K
= a 2 sec 2
F dθ I
αG J .
2

H dt K
cc

Here a is the radius and α the angle of the helix.


Now, if z0 be initial value of z, then by the principle of energy
FG ds IJ 2
= a 2 sec 2 α
FG dθ IJ 2
2 ( z − z0 ) g =
H dt K H dt K
Su

FG dθ IJ 2
2g
or H dt K =
a sec 2 α
( z − z0 )
2

we have z = a θ tan α and z0 = a θ 0 tan α


FG dθ IJ = 2ga tan α (θ − θ ) = 2g sin α cos α (θ − θ )
2
∴ H dt K a sec α 2 2 a
0 0

FG 2g sin α cos α IJ dt = dθ
∴ H a K (θ − θ ) 0
Integrating,
FG 2g sin α cos α IJ = zθ + α dθ = 2 [ 2
(θ − θ 0 )]θθ + 2 π = 2 (2 π)
t
H a K θ 0
(θ − θ )
0
0
0

t=2 G
F πa IJ .
Hence,
H g sin α cos α K
SuccessClap: Best Coaching for UPSC Mathematics : For Info- 9346856874
Checkout ->22 Weeks Study Plan, Videos, Question Bank Solutions, Test Series

Motion in Three Dimensions 847


2. A particle moving on a paraboloid of revolution under a force parallel to the axis crosses
the meridians at a constant angle. Show that the force varies inversely as the fourth power of the
distance from the axis.
Solution. Let the equation to the paraboloid in cylindrical coordinates be
ρ2 = 4 az ...(1)
The equation of motion is
1 d
ρ2
dφ FG IJ
ρ dt dt H
=0
K

i.e., ρ2 = constant = A (say) ...(2)
dt
Let the force F parallel to axis cross the meridians at a constant angle β then

r sin θ
dt = tan β,

r

ap
dt
dθ dφ
i.e., r = r sin cot β,
dt dt
But we know that
ds dθ dθ

l
=rsC = r sin θ cot β
dt dt dt

= ρ cos β . [∵ ρ = r sin θ] ...(3)
dt
If z0 be the initial value of z, then by the principle of energy

z 1 2 dφFG IJ 2
1 2 FG IJ

2
es
[1 + cot 2 β] = ρ cosec 2 β
z0z F dz =
2
ρ
dt H K 2 H K
dt
A2
= from (2)
2 sin 2 . ρ 2
cc

A2
= , from (1)
8a sin 2 β. z
A2 2 aA 2 2
Integrating, F=− =− . , from (1)
Su

2 2
8a sin β. z sin β ρ 4
2

1
∴ F∝ , i. e., the force varies inversely as the fourth power of the distance from the axis.
ρ2
EXERCISES
1. A particle, without weight, slides on a smooth helix of angle α and radius a under a force to
a fixed point on the axis equal to m µ (distance). Show that the reaction of the curve can
not vanish unless the greatest velocity of the particle is a µ sec α.
2. A smooth paraboloid is placed with its axis vertical and vertex downwards, the latus-rectum
of the generating parabola being 4a. A heavy particle is projected horizontally with velocity
V at a height h above the lowest point. Show that the particle is again moving horizontally

V2
when its height is . Show also that the reaction of the paraboloid at any point is inversely
2g
proportional to the corresponding radius of curvature of the generating parabola.
✰✰✰

You might also like